Criminal Law 1 Review

You might also like

Download as doc, pdf, or txt
Download as doc, pdf, or txt
You are on page 1of 85

Table of Contents

Art.3: Felonies...................................................................................................................................................6
Classification of Felonies According to the Means of Commission.............................................................6
Calimutan v. People (G.R. No. 152133)...................................................................................................6
Manuel v. People (G.R. No. 165842).......................................................................................................9
Mistake of Fact...........................................................................................................................................12
U.S. v. Ah Chong (15 Phil. 488)..............................................................................................................12
Diego v. Castillo (A.M. No. RTJ-02-1673)...............................................................................................14
People v. Fernando (G.R. No. L-24978).................................................................................................16
Mala in se v. Mala prohibita.......................................................................................................................18
Estrada v. Sandiganbayan (G.R. No. 148560)........................................................................................18
People v. Go Shiu Ling (G.R. No. 115156)..............................................................................................19
Art. 4: Criminal Liability..................................................................................................................................20
Wrongful Act Done be Different from That Intended...............................................................................20
Quinto v. Andres (453 SCRA 511)..........................................................................................................20
People v. Ortega (276 SCRA 166)...........................................................................................................21
People v. Pilda (405 SCRA 134)..............................................................................................................22
Impossible Crimes......................................................................................................................................23
People v. Domasian (219 SCRA 245)......................................................................................................23
Intod v. CA (215 SCRA 52)......................................................................................................................25
Art. 6: Stages of Execution..............................................................................................................................27
People v. Campuhan (G.R. No. 129433)................................................................................................27
Valenzuela v. People (G.R. No. 160188)................................................................................................29
People v. Orita (G.R. No. 88724)............................................................................................................31
Art. 8: Conspiracy...........................................................................................................................................33
People v. Quirol (473 SCRA 509)............................................................................................................33
People v. Comadre (431 SCRA 366).......................................................................................................35
Sim v. CA (428 SCRA 459)......................................................................................................................37
Art. 11: Justifying Circumstances...................................................................................................................40
Self-defense................................................................................................................................................40
People v. Sanchez (G.R. No. 161007).....................................................................................................40
Soplente v. People (G.R. No. 152715)...................................................................................................42
Urbano v. People (G.R. No. 182750)......................................................................................................44
Defense of Relatives...................................................................................................................................46
Balunueco v. CA and People (G.R. No. 126968)...................................................................................46
Fulfilment of a Duty...................................................................................................................................48
Mamangun v. People (GR 149152)........................................................................................................48
Baxinela v. People (G.R. No. 149652)....................................................................................................50
Angcaco v. People (G.R. No. 146664)....................................................................................................51
Obedience to an Order..............................................................................................................................53
Tabuena v. Sandiganbayan (G.R. Nos. 103501-03)...............................................................................53
Art.12: Exempting Circumstances..................................................................................................................55
1|Page
Insanity/Imbecility.....................................................................................................................................55
People v. Rubiños (G.R. No. 138453).....................................................................................................55
People v. Valledor (G.R. No. 129291)....................................................................................................58
Minority......................................................................................................................................................61
Llave v. People (G.R. No. 166040).........................................................................................................61
Jose v. People (G.R. No. 162052)...........................................................................................................63
Declarador v. Judge Gubaton (G.R. No. 159208)...................................................................................65
Accident......................................................................................................................................................67
Toledo v. People (439 SCRA 94).............................................................................................................67
People v. Concepcion (386 SCRA 74).....................................................................................................68
Irresistible Force/Uncontrollable Fear.......................................................................................................70
Ty v. People (G.R. No. 149275)..............................................................................................................70
Entrapment v. Instigation..........................................................................................................................72
People v. Sta. Maria (G.R. No. 171019).................................................................................................72
People v. Pacis (G.R. No. 146309)..........................................................................................................74
Chang v. People (G.R. No. 165111)........................................................................................................76
Art. 13: Mitigating Circumstances..................................................................................................................78
Incomplete Justifying or Exempting Circumstances..................................................................................78
People v. CA and Tangan (G.R. No. 103613).........................................................................................78
No Intention to Commit so Grave a Wrong...............................................................................................81
People v. Callet (G.R. No. 135701).........................................................................................................81
Vindication of a Grave Offense..................................................................................................................82
People v. Torpio (G.R. No. 138984).......................................................................................................82
Passion or Obfuscation..............................................................................................................................84
People v. Lab-eo (G.R. No. 133438).......................................................................................................84
People v. Bates (G.R. No. 139907).........................................................................................................86
People v. Malejana (G.R. No. 145002)...................................................................................................88
Voluntary Surrender..................................................................................................................................89
People v. Beltran (G.R. No. 168051)......................................................................................................89
Andrada v. People (GR No. 135222)......................................................................................................91
People v. Quimzon (G.R. No. 133541)...................................................................................................93
Confession of Guilt.....................................................................................................................................95
People v. Montinola (G.R. Nos. 131856-57)..........................................................................................95
People v. Dawaton (G.R. No. 146247)...................................................................................................97
Similar and Analogous Circumstances.......................................................................................................98
Canta v. People (G.R. No. 140937)........................................................................................................98
Art. 14: Aggravating Circumstances.............................................................................................................100
Classes of Aggravating Circumstances.....................................................................................................100
People v. Evina (405 SCRA 152)...........................................................................................................100
People v. Palaganas (501 SCRA 533)...................................................................................................101
People v. Mendoza (327 SCRA 695)....................................................................................................102
In Contempt or With Insult to the Public Authorities..............................................................................103
People v. De Mesa (G.R. No. 137036).................................................................................................103

2|Page
People v. Tac-an (G.R. No. 76338-39)..................................................................................................104
Abuse of Public Position...........................................................................................................................106
Fortuna v. People (G.R. No. 135784)...................................................................................................106
People v. Villamor (G.R. Nos. 140407-08)...........................................................................................107
People v. Magayac (G.R. No. 126043).................................................................................................108
Nighttime, Uninhabited or Obvious Place or Band..................................................................................109
People v. Villanueva (G. R. No. 135330)..............................................................................................109
People v. Ancheta (G.R. No. 70222)....................................................................................................112
People v. Baroy (G.R. Nos. 137520-22)...............................................................................................114
Recidivism................................................................................................................................................116
People v. Dacillo (G.R. No. 149368).....................................................................................................116
Reiteracion...............................................................................................................................................118
People v. Cajara (G.R. No. 122498).....................................................................................................118
By Means of Inundation, fire, etc.............................................................................................................121
People v. Malngan (G.R. No. 170470).................................................................................................121
People v. Comadre (G.R. No. 153559).................................................................................................123
Craft, Fraud or Disguise............................................................................................................................125
People v. Labuguen (G.R. No. 127849)................................................................................................125
Abuse of Superior Strength......................................................................................................................127
People v. Amodio (G.R. No. 177356)...................................................................................................127
People v. Jamon (413 SCRA 282).........................................................................................................129
People v. Calpito (416 SCRA 491)........................................................................................................131
Treachery.................................................................................................................................................133
People v. Piedad (393 SCRA 488).........................................................................................................133
People v. Piliin (515 SCRA 207)............................................................................................................135
People v. Ilo (392 SCRA 326)................................................................................................................137
Ignominy...................................................................................................................................................139
People v. Salazar (G.R. Nos. 148712-15).............................................................................................139
People v. Bumidang (G.R. No. 130630)...............................................................................................141
People v. Siao (G.R. No. 126021).........................................................................................................144
Aid of Minor or By Means of Motor Vehicles..........................................................................................146
People v. Mallari (404 SCRA 170)........................................................................................................146
People v. Enguito (326 SCRA 508).......................................................................................................148
Cruelty......................................................................................................................................................151
People v. Guerrero (389 SCRA 389).....................................................................................................151
Simangan v. People (434 SCRA 38)......................................................................................................153
People v. Catian (374 SCRA 514).........................................................................................................154
Art. 15: Alternative Circumstances...............................................................................................................155
Relationship..............................................................................................................................................155
People v. Calongui (G.R. No. 170566)..................................................................................................155
People v. Marcos (G.R. No. 132392)....................................................................................................156
Intoxication..............................................................................................................................................157
People v. Marquita (G.R. No. 137050).................................................................................................157

3|Page
People v. Mondigo (G.R. No. 167954).................................................................................................159
Arts. 16-20: Persons Criminally Liable for Felonies......................................................................................160
Principals..................................................................................................................................................160
People v. Batin (GR No. 177223).........................................................................................................160
People v. Vasquez (G.R. No. 123939)..................................................................................................162
People v. Dacillo (G.R. No. 149368).....................................................................................................165
Accomplices.............................................................................................................................................167
People v. Roche (G.R. No. 115182)......................................................................................................167
Abarquez v. People (G.R. No. 150762)................................................................................................169
People v. Compo (G.R. No. 112990)....................................................................................................171
Accessories...............................................................................................................................................173
People v. Tolentino (G.R. No. 139179)................................................................................................173
People v. Cui (G.R. No. 121982)...........................................................................................................176
People v. Verzola (G.R. No. L-35022)...................................................................................................179
Accessories Exempt from Criminal Liability.............................................................................................181
People v. Mariano (G.R. No. L-40527).................................................................................................181
Arts. 21-24: Penalties in General..................................................................................................................184
Retroactive Effect of Penal Laws..............................................................................................................184
People v. Evina (G.R. No. 124830-310)................................................................................................184
People v. Lazaro (G.R. No. 112090).....................................................................................................185
People v. Pacifador (G.R. No. 139405)................................................................................................186
Pardon by Offended Party........................................................................................................................187
Sta. Catalina v. People (G.R. No. 167805)............................................................................................187
Balderama v. People (G.R. No. 147578-85).........................................................................................189
People v. Dimaano (G.R. No. 168168).................................................................................................191
Arts. 25-45: Penalties...................................................................................................................................193
Reclusion Perpetua..................................................................................................................................193
People v. Novio (G.R. No. 139332)......................................................................................................193
People v. Zacarias (G.R. No. 138990)...................................................................................................195
People v. Ramirez (G.R. No. 138261)...................................................................................................196
Arts. 46-77: Application of Penalties............................................................................................................197
Complex Crime.........................................................................................................................................197
People v. Latupan (G.R. Nos. 112453-56)............................................................................................197
People v. Pineda (G.R. No. L-26222)....................................................................................................200
People v. Sanidad (G.R. No. 146099)...................................................................................................202
Delito Continuado....................................................................................................................................204
Ramiscal v. Sandiganbayan (G.R. Nos. 169727-28).............................................................................204
Santiago v. Garchitorena (G.R. No. 109266)........................................................................................206
Habitual Delinquency...............................................................................................................................208
People v. Espina (G.R. No. 43556).......................................................................................................208
People v. De Jesus (G.R. No. 45198)....................................................................................................209
Arts. 89-93: Total Extinction of Criminal Liability.........................................................................................211
Death of the Accused...............................................................................................................................211

4|Page
De Guzman v. People (G.R. No. 154579).............................................................................................211
People v. Bayotas (G.R. No. 102007)...................................................................................................212
People v. Abungan (G.R. No. 136843).................................................................................................214
Prescription of Offenses...........................................................................................................................215
Panaguiton v. DOJ (G.R. No. 167571)..................................................................................................215
Recebido v. People (346 SCRA 881).....................................................................................................217
Caniza v. People (159 SCRA 16)...........................................................................................................219
Amnesty...................................................................................................................................................221
People v. Patriarcha (G.R. No. 135457)...............................................................................................221
Arts. 100-103: Civil Liability..........................................................................................................................224
Nuguid v. Nicdao (G.R. No. 150785)....................................................................................................224
Subsidiary Civil Liability of Other Persons................................................................................................227
Nueva Espana v. People (460 SCRA 547).............................................................................................227
Pangonorom v. People (455 SCRA 211)...............................................................................................230
Quinto v. Andres (453 SCRA 511)........................................................................................................232
Probation Law (P.D. No. 968).......................................................................................................................234
Francisco v. CA (G.R. No. 108747).......................................................................................................234
Lagrosa v. People (G.R. No. 152044)...................................................................................................236
Vicoy v. People (G.R. No. 138203).......................................................................................................238
Anti-Fencing Law (P.D. No. 1612).................................................................................................................239
Francisco v. People (434 SCRA 122)....................................................................................................239
Tan v. People (313 SCRA 220)..............................................................................................................241

Art.3: Felonies

Classification of Felonies According to the Means


of Commission

Calimutan v. People (G.R. No. 152133)

Facts:

Victim Cantre and Sañano, together with two


other companions had a drinking spree in a videoke
bar at ten o’clock in the morning of February 4, 1996.
Thereafter, they decided to part ways and went to their
respective houses. On their way home, Cantre and
Sanano met the petitioner and Michael Bulalacao.
Cantre suddenly punched Bulalacao because he is
suspecting the latter as the one responsible for
throwing stones at his house on previous night. After
being hit, bulalacao ran away. Petitioner picked-up a
stone which is as big as man’s fist, ran toward Cantre,
and threw it to the latter, hitting him at the left side of
his back. When Cantre turned his attention to the
petitioner, Sanano tried pacify the two. Both Cantre
and petitioner calmed down and went to their houses.
When Cantre arrived at his house, he complained of
the pain in the left side of his back which was hit by the
stone. At that night, he again complained of backache
and also of stomachache. He’s condition immediately

5|Page
became worst, and at around three o’clock in the another. In culpable felonies, the act or omission of the
following morning, Cantre died. offender is not malicious. The injury caused by the
offender to another person is "unintentional, it being
Right after his death, Cantre was examined by simply the incident of another act
Dr. Conchita S. Ulanday, the Municipal Health Officer performed without  malice." (People vs. Sara, 55 Phil.
and made a findings that the cause of death was 939). As stated in Art. 3, the wrongful act results from
cardio-respiratory arrest due to suspected food imprudence, negligence, lack of foresight or lack of
poisoning. Unsatisfied, the Cantre family requested for skill.
an exhumation and autopsy of the body of the victim In the Petition at bar, this Court cannot, in
by the NBI. Dr. Mendez conducted an exhumation and good conscience, attribute to petitioner any malicious
autopsy and reported that the cause of the death was intent to injure, much less to kill, the victim Cantre; and
traumatic injury of the abdomen. The victim suffered in the absence of such intent, this Court cannot sustain
from an internal hemorrhage and there was massive the conviction of petitioner Calimutan for the intentional
accumulation of blood in his abdominal cavity due to crime of homicide, as rendered by the RTC and
his lacerated spleen caused by any blunt instrument, affirmed by the Court of Appeals. Instead, this Court
such as a stone. finds petitioner Calimutan guilty beyond reasonable
doubt of the culpable felony of reckless imprudence
Petitioner alleged that he only attempted to resulting in homicide  under Article 365 of the Revised
pacify the victim but the latter refused and pulled out Penal Code. The prosecution did not establish that
eight-inch Balisong. When he saw the victim was petitioner Calimutan threw the stone at the victim
about to stab Bulalacao, he picked up a stone and Cantre with the specific intent of killing, or at the very
threw it at the victim Cantre. He was able to hit the least, of harming the victim Cantre. What is obvious to
victim. He contended that the throwing of the stone this Court was petitioner Calimutan’s intention to drive
was in defense of his companion. away the attacker who was, at that point, the victim
Cantre, and to protect his helper Bulalacao who was,
The RTC rendered a decision, which was later as earlier described, much younger and smaller in built
affirmed by the CA, holding that petitioner was than the victim Cantre.
criminally liable for homicide and that the act of
throwing a stone from behind was a treacherous one Manuel v. People (G.R. No. 165842)
and the accused committed a felony which caused the
death of the victim and held that the accused is
criminally liable for all the direct and natural Facts: This is a case filed against Eduardo Manuel for
consequences of this unlawful act even if the ultimate bigamy by Tina B. Gandalera. Complainant allege that
result had not been intended. Hence, these case. she met the petitioner in Dagupan City sometime in
January 1996. When he visited her in Baguio, as one
Issue: thing led to another, they went to a motel where,
Eduardo succeeded in having his way with her.
Whether or not the petitioner has the intent to Petitioner proposed marriage and even brought his
kill the victim and thus liable for homicide? parents to assure that he is single. Tina finally
accepted the marriage proposal and they were married
Decision: on April 22, 1996. In their marriage contract, it
appeared that Eduardo is “single”. However, their
While the Supreme Court is in accord with the happy relationship turns into a disaster, Manuel started
factual findings of the RTC and the CA and affirms that making himself scarce and went to their house only
there is ample evidence proving that the death of the twice or thrice a year. One day, petitioner took all of his
victim Cantre was caused by his lacerated spleen cloths, left and never returned. Out of curiousity, Tina
which is the result by the stone thrown at him by went to NSO in Manila where she found out that
petitioner Calimutan, it nonetheless, is at variance with petitioner had been previously married to Rubylus
the RTC and the CA as to the determination of the Gaña. She was so embarrassed and humiliated when
appropriate crime or offense for which the petitioner she learned that Eduardo was in fact already married
should have been convicted for. when they exchanged their own vows.

Article 3 of the Revised Penal Code classifies For his part, Eduardo testified that he informed
felonies according to the means by which they are Tina of his previous marriage, but she nevertheless
committed, in particular: (1) intentional felonies, and agreed to marry him. He abandoned her when he
(2) culpable felonies. These two types of felonies are noticed that she had a "love-bite" on her neck,
distinguished from each other by the existence or suspecting it that it come from another man. Eduardo
absence of malicious intent of the offender. further testified that he declared he was "single" in his
marriage contract with Tina because he believed in
In intentional felonies, the act or omission of good faith that his first marriage was invalid. He did not
the offender is malicious. In the language of Art. 3, the know that he had to go to court to seek for the
act is performed with deliberate intent (with malice). nullification of his first marriage before marrying Tina..
The offender, in performing the act or in incurring the Rubylus was charged with estafa in 1975 and
omission, has the intention to cause an injury to thereafter imprisoned. He visited her in jail after three

6|Page
months and never saw her again. He insisted that he Malice is a mental state or condition prompting
married Tina believing that his first marriage was no the doing of an overt act without legal excuse or
longer valid because he had not heard from Rubylus justification from which another suffers injury. When
for more than 20 years. After trial, the court rendered the act or omission defined by law as a felony is
judgment finding Eduardo guilty beyond reasonable proved to have been done or committed by the
doubt of bigamy. It declared that Eduardo’s belief, that accused, the law presumes it to have been
his first marriage had been dissolved because of his intentional. Indeed, it is a legal presumption of law that
first wife’s 20-year absence, even if true, did not every man intends the natural or probable
exculpate him from liability for bigamy and that even if consequence of his voluntary act in the absence of
the private complainant had known that Eduardo had proof to the contrary, and such presumption must
been previously married, the latter would still be prevail unless a reasonable doubt exists from a
criminally liable for bigamy. Eduardo appealed the consideration of the whole evidence.
decision to the CA maintaining his contentions. He
insisted that conformably to Article 3 of the Revised
Penal Code, there must be malice for one to be Mistake of Fact
criminally liable for a felony. He was not motivated by
malice in marrying the private complainant because he U.S. v. Ah Chong (15 Phil. 488)
did so only out of his overwhelming desire to have a
fruitful marriage. Hence, these case.
Facts:
Issue: Whether or not the petitioner has criminal
intent to contract on the second marriage to be liable Defendant herein a chinese man named Ah
for bigamy? Chong is employed us a cook at Fort Mckinley. At that
time there were rumours and accounts of frequent
Decision: The Supreme Court ruled that the robbing of homes in the area.
prosecution proved that the petitioner was married to
Gaña in 1975, and such marriage was not judicially On the night of the killing, Ah chong before
declared a nullity; hence, the marriage is presumed to going to bed, and afraid of the rumoured robberies
subsist. The prosecution also proved that the petitioner taking place in the vicinity locked himself in their room
married the private complainant in 1996, long after the by placing wooden blocks and chairs for the purpose
effectivity of the Family Code. The petitioner is of thwarting robbers in case they tried to rob him.
presumed to have acted with malice or evil intent when
he married the private complainant. As a general rule, After having gone to bed, he was awakened
mistake of fact or good faith of the accused is a valid by the noise of someone trying to open the door. Ah
defense in a prosecution for a felony by dolo; such Chong for his part called out twice, “Who is there, but
defense negates malice or criminal intent. However, to no avail”. Fearing that the person trying to enter was
ignorance of the law is not an excuse because robber Ah Chong leaped from his bed and shouted “If
everyone is presumed to know the law. Ignorantia you enter the room I will kill you”. But at that precise
legis neminem excusat. It was the burden of the moment, he was suddenly struck by the chair that he
petitioner to prove his defense that when he married had placed in the door, and believing that he was
the private complainant in 1996, he was of the well- being attacked he seized a knife and struck it on the
grounded belief  that his first wife was already dead, as supposed assailant/robber, who was killed by the blow.
he had not heard from her for more than 20 years However the deceased was not a robber not intruder it
since 1975. He should have adduced in evidence a turned out that the person was his roommate, trying to
decision of a competent court declaring the enter their room.
presumptive death of his first wife as required by
Article 349 of the Revised Penal Code, in relation to Issue:
Article 41 of the Family Code. Such judicial declaration
also constitutes proof that the petitioner acted in good Whether or not Ah Chong is criminally liable?
faith, and would negate criminal intent on his part
when he married the private complainant and, as a Decision:
consequence, he could not be held guilty of bigamy in
such case. The petitioner, however, failed to discharge NO. Ah Chong must be acquitted on the basis
his burden. of honest mistake of fact. Where the facts been as Ah
Article 3, paragraph 2 of the Revised Penal Chong perceived them to be, he would have been
Code provides that there is deceit when the act is justified in killing the intruder under Article 11 of the
performed with deliberate intent. Indeed, a felony Revised, par. 1 of the Revised Penal Code, which
cannot exist without intent. Since a felony by dolo is provides for a valid self-defense of his person. If the
classified as an intentional felony, it is deemed intruder was indeed a robber, forcing his way to enter
voluntary. Although the words "with malice" do not the room, unlawful aggression would be present. Also
appear in Article 3 of the Revised Penal Code, such the necessity means to avoid or to repel the attack
phrase is included in the word "voluntary." would be reasonable. Using the knife to defend
himself. And lastly Ah Chong gave no provocation at
all to warrant such aggression. The Supreme Court

7|Page
Held that there is nothing unlawful in the intention as People v. Fernando (G.R. No. L-24978)
well in the act of Ah Chong, his act would not have
been a felony if the real scenario was the facts he
believed them to be. Facts:

Diego v. Castillo (A.M. No. RTJ-02-1673) The residents of Barrio of Municahan of the
Municipality of Zamboanga were alarmed by the
presence of 3 suspicious looking persons prowling
Facts: around the town, suspecting them as moro prisoners
who recently escaped from Jail.
This is an administrative complaint filed
against herein respondent for Gross ignorance of the Fernando the accused herein was a
law in rendering his decision in a criminal complaint for policeman, when passing in front of the house of
bigamy. Remigio Delgado he was called by the latters daughter
and said to him that her father wanted to talk to him.
On 1965 Lucena Escoto contracted marriage Remigio told Fernando that 3 unknown and suspicious
with Jorge de Perio Jr. Both of which were Filipino looking fellows were prowling around the house,
Citizens. However on February 15, 1978 the two dressed in blue same as those purportedly worn by the
acquired a Decree of Divorce in Texas, USA. escapees. Fernando stayed in the house talking to the
daughter of Remigio,, both seated in a bench near the
On June 4, 1987 the same Lucena Escoto window. At about 7 o’clock in the evening , there
contracted marriage with herein complainants brother appeared a figure in the dark about 4 meters from the
Manule P. Diego, celebrated at Dagupan. stairs, a person in dark clothes, calling Mang Miong.
Fernando and the daughter of Remegio had no idea
Judge Castillo held in this case the acquittal of who was calling. Fernando asked the man what he
Ms. Escoto on the basis of good faith on her part. That wanted but instead of answering the question the man
Ms. Escoto believing that her previous marriage had continued to the walk with bolo in hand. Fernando
been validly dissolved by the divorce decree acquired upon seeing this took out his revolver and fired a
in a foreign country and that she was legally free to warning shot. Thereafter having fired a shot into the air
contract the second marriage. That according to Judge the man continued his ascend to the stairs, Fernando
Castillo as an ordinary laywoman , she entertains the took a shot at him. However it was found out that the
impression that she can contract a subsequent unknown man was Buenaventura Paulino, nephew of
marriage. Furthermore Judge Castillo stressed that Remigio.
knowledge of the law should not be exacted strictly
from her since she is a lay person, and that ineptitude The trial court held that Fernando was guilty of
should not be confused with criminal intent. the crime of murder. Hence this appeal.

Issue: Issue:

Whether or not mistake of fact to cut-off the Whether or not Fernando is criminally liable for
criminal liability of Ms. Escoto was validly taken up by his acts?
Judge Castillo?
Decision:

Decision: Yes. But not for the crime of murder. The


accused being agent of the law, to whom notice was
No. As carefully distinguished by the Supreme given of the presence of the suspicious looking
Court in its previous decisions that mistake of fact, persons who might be the escapees. The appearance
which would could be a valid defense of good faith in a of a man unknown to him, dressed in clothes as that of
bigamy case, from mistake of law, which does not the escaped convicts, and calling to the owner of the
excuse a person, even a lay person, from liability. In house, of which the daughter of the owner of the
People vs. Bidtu the Supreme Court held that even if house did not also recognized, caused the accused to
the accused, who had obtained a divorce decree under suspect that the unknown man was one of the
Mohammedan custom, honestly believed that in escaped convicts, and after firing a warning shot, the
contracting her second marriage she was not man still did not halt his advance with bolo in hand. In
committing any violation of law, and that she had no the midst of the circumstances and believing that the
criminal intent, the same does not justify the her act. man was a wrongdoer he tried to perform his duty and
The Court further that it is sufficient to say that first fired into the air and then at the allege intruder. At
everyone is presumed to know the law, and the fact that psychological moment when the forces of far and
that one doe not know that his act constitutes a the sense of duty were at odds, the accused was not
violation of law does not exempt him from the able to take full account of the true situation. However,
consequence thereof. a circumstance that should have made him suspect
that the man was not only a friend but a relative when
the man called “Nong Miong, and in not asking the

8|Page
daughter of the owner of the house who was it who Even granting that Comia acted in good faith,
was calling to her father with such familiarity, he did he cannot escape criminal responsibility. The crime
not use the ordinary precaution that he should have with which he is charged is a malum prohibitum. Lack
used before talking such fatal action. Hence he is of criminal intent and good faith are not exempting
liable for homicide through reckless negligence. circumstances. As held inPeople v. Lo Ho Wing:
Moreover, the act of
transporting a prohibited drug
is a "malum prohibitum"
because it is punished as an
Mala in se v. Mala prohibita offense under a special law. It
is a wrong because it is
Estrada v. Sandiganbayan (G.R. No. 148560) prohibited by law. Without the
law punishing the act, it
cannot be considered a
Facts:
wrong. As such, the mere
commission of said act is what
Petitioner Former President Joseph Estrada
constitutes the offense
was prosecuted for a crime of violation of RA 7080 (An
punished and suffices to
Act Defining and Penalizing the Crime of Plunder), as
validly charge and convict an
amended by RA 7659. Thus, he questions the
individual caught committing
constitutionality of the said Law. One of the issues that
the act so punished,
was raised in the petition is whether Plunder as
regardless of criminal intent.
defined in RA 7080 is a malum prohibitum,  and if so,
Likewise, in People v. Bayona, it was held:
whether it is within the power of Congress to so
The rule is that in acts mala in
classify it.
se  there must be a criminal
intent, but in those mala
Issue:
prohibita it is sufficient if the
prohibited act was
Whether or not Plunder is a crime malum
intentionally done. "Care must
prohibitum?
be exercised in distinguishing
the difference between the
Decision:
intent to commit the crime and
the intent to perpetrate to act."
The legislative declaration in R.A. No.7659
that plunder is a heinous offense implies that it is a
malum in se.  For when the acts punished are
inherently immoral or inherently wrong, they are mala
Art. 4: Criminal Liability
in se  and it does not matter that such acts are
punished in a special law, especially since in the case Wrongful Act Done be Different from That Intended
of plunder the predicate crimes are mainly mala in
se. Indeed, it would be absurd to treat prosecutions for Quinto v. Andres (453 SCRA 511)
plunder as though they are mere prosecutions for
violations of the Bouncing Check Law (B.P. BIg. 22) or
of an ordinance against jaywalking, without regard to Facts:
the inherent wrongness of the acts.
On Nov. 13,1995, Dante Andres and
People v. Go Shiu Ling (G.R. No. 115156) Randyven Pacheco invited Wilson Quinto and Edison
Garcia to go fishing with them inside a drainage
culvert. However, only Quinto joined the two, Garcia
Facts: remain in a grassy area about two meters from the
entrance of the drainage system. After a while,
The Regional Trial Court of Pasay City finds Pacheco came out, went back again, and emerged
accused-appellant Antonio Comia guilty of conspiring again carrying Wilson who was already dead. He laid
with four others to import regulated drugs in violation of the boy’s lifeless body down in the grassy area and
Art. III, Section 14 in relation to Article IV, Section 21 of went to the house of Wilson’s mother and informed her
the Dangerous Drugs Act (Rep. Act No. 6425, as that her son had died. After more than three months,
amended). the cadaver of Wilson was exhumed and the NBI
performed an autopsy thereon. An information was
Issue: later filed with the RTC changing Andres and Pacheco
with homicide.
Whether or not a crime for violation of
Dangerous Drugs Act is a crime malum prohibitum? Issue:
Decision:

9|Page
Whether or not the accused has criminal People v. Pilda (405 SCRA 134)
liability for the death of the victim?

Facts:
Decision:
Accused Edman Aguilos, Odilon Lagliba and
The prosecution failed to prove the guilt of the Rene Pilola were changed with murder for the death of
accused beyond reasonable doubt. It failed to prove Joselito Capa. Rene Pilola devised stabbing the victim
the guilt of the accused is criminality liable although and interposed the defense of alibi. The trial count
the wrongful act done be different from that which he found all the accused guilty and sentenced them to
intended. The Supreme Court agreed with the trial and reclusion perpetua. Rene Pilola appealed the decision
appellate courts that the proximate cause of the death by contending that there was no conspiracy and he
of the victim was not cause by any wrongful act of the may not be held criminally liable as principal by direct
accused. It is the burden of the prosecution to prove participation. He argued that the prosecution failed to
the corps delicti which consists of criminal act and the prove that he conspined with the others in stabbing the
defendant’s agency in the commission of the act. This, victim to death. He asserts that he is merely an
the prosecution failed to do. accomplice.

Issue:

People v. Ortega (276 SCRA 166) Whether or not the appellant may be held
criminally liable as principal by direct participation in
the absence of proof of conspiracy?
Facts :
Decision:
In 1992, Benjamin Ortega, Jr., Manuel Garcia
and a certain John Doe were changed with murder for The court in applying paragraph 1, Article 4 of
the killing Andre Man Masangkay. As narrated by a the Revised Penal Code ruled that even if two or more
witness, the victim answered the called of nature and offenders do not conspire to commit homicide or
went to the back portion of the house where they were murder, they may be held criminally liable as principals
having a drinking spree. Accused Ortega followed him by direct participation if they perform overt acts which
and later they heard the victim shouting for help and immediately cause or accelerate the death of the
when they ran towards the scene he saw the accused victim. They are all criminally liable although the
on top of the victim and stabbing the latter with a long wrongful act done be different from that which he
bladed weapon. Thereafter, Ortega and Garcia intended by reason of their individual and separate
brought the victim to a well and dropped him and overt criminal acts.
placed stones into the well. The trial court found the
accused guilty beyond reasonable doubt. The accused Impossible Crimes
appealed averring that the trial court erred in holding
them criminally liable because at the time the victim People v. Domasian (219 SCRA 245)
was dropped into the well, he was still alive.

Issue: Facts:

Whether or not the accused may be held In the morning of March 11, 1982, while Enrico
criminally liable for the death of the victim which is not was walking with a classmate along Roque Street in
attributable to the stab wounds but due to drowning? Lopez, Quezon, he was approached by a man
(Domasian) who requested his assistance in getting
Decision: his father's signature on a medical certificate.Enrico
agreed to help and rode with Domasian in a tricycle to
A person who commits a felony is criminally Calantipayan. Enrico became apprehensive and
liable for the direct natural and logical consequences started to cry when, instead of taking him to the
of his wrongful act even where the resulting crime is hospital, Domasian flagged a minibus and forced him
more serious than that intended. The essential inside, holding him firmly all the while. Domasian told
requisites for this criminal liability to attach are as him to stop crying or he would not be returned to his
follows : father.

1. the intended act is felonious ; Domasian talked to a jeepney driver and


2. the resulting act is likewise a felony; handed him an envelope addressed to Dr. Enrique
and Agra, the boy's father. Then they rode a tricycle, the
3. the unintended graven wrong was driver got suspicious and reported the matter to two
primarily caused by the actor’s barangay tanods. The tanods went after the two,
wrongful acts. Somehow, Domasian managed to escape, leaving
Enrico behind. Enrico was on his way home in a

10 | P a g e
passenger jeep when he met his parents, who were companions, Mandaya pointed the location of
riding in the hospital ambulance and already looking Palangpangan's bedroom. Thereafter, Petitioner,
for him. Pangasian, Tubio and Daligdig fired at said room.It
turned out, however, that Palangpangan was in
At about 1:45 in the afternoon of the same another City and her home was then occupied by her
day, after Enrico's return, Agra received an envelope son-in-law and his family. No one was in the room
containing a ransom note. The note demanded P1 when the accused fired the shots. No one was hit by
million for the release of Enrico and warned that the gun fire.
otherwise the boy would be killed. Agra thought the
handwriting in the note was familiar. After comparing it Issue:
with some records in the hospital, he gave the note to
the police, which referred it to the NBI for Whether or not said act constitutes an
examination.The test showed that it bad been written impossible crime?
by Dr. Samson Tan. Domasian and Tan were
subsequently charged with the crime of kidnapping Decision:
with serious illegal detention in the Regional Trial
Court of Quezon. Yes. The factual situation in the case at bar
present a physical impossibility which rendered the
Issue: intended crime impossible of accomplishment and
under Article 4, paragraph 2 of the Revised Penal
Whether or not the sending of the ransom note Code, such is sufficient to make the act an impossible
was an impossible crime? crime.

Decision: To be impossible under this clause, the act


intended by the offender must be by its nature one
No. Even before the ransom note was impossible of accomplishment. There must be either
received, the crime of kidnapping with serious illegal impossibility of accomplishing the intended act in order
detention had already been committed. The act cannot to qualify the act an impossible crime. Legal
be considered an impossible crime because there was impossibility occurs where the intended acts, even if
no inherent improbability of its accomplishment or the completed, would not amount to a crime.Factual
employment of inadequate or ineffective means. The impossibility occurs when extraneous circumstances
delivery of the ransom note after the rescue of the unknown to the actor or beyond his control prevent the
victim did not extinguish the offense, which had consummation of the intended crime. The case at bar
already been consummated when Domasian deprived belongs to this category.
Enrico of his liberty.
In our jurisdiction, impossible crimes are
Moreover the trial court correctly held that recognized. The impossibility of accomplishing the
conspiracy was proved by the act of Domasian in criminal intent is not merely a defense, but an act
detaining Enrico; the writing of the ransom note by penalized by itself. Furthermore, the phrase "inherent
Tan; and its delivery by Domasian to Agra. These acts impossibility" that is found in Article 4(2) of the Revised
were complementary to each other and geared toward Penal Code makes no distinction between factual or
the attainment of the common ultimate objective physical impossibility and legal impossibility.

Intod v. CA (215 SCRA 52) To uphold the contention of respondent that


the offense was Attempted Murder because the
absence of Palangpangan was a supervening cause
Facts: independent of the actor's will, will render useless the
provision in Article 4, which makes a person criminally
In the morning of February 4, 1979, Intod, liable for an act "which would be an offense against
Pangasian, Tubio and Daligdig went to Mandaya's persons or property, were it not for the inherent
house in Lopez Jaena, Misamis Occidental and asked impossibility of its accomplishment . . ." In that case all
him to go with them to the house of Palangpangan. circumstances which prevented the consummation of
Thereafter, Mandaya and Intod, Pangasian, Tubio and the offense will be treated as an accident independent
Daligdig had a meeting with Aniceto Dumalagan. He of the actor's will which is an element of attempted and
told Mandaya that he wanted Palangpangan to be frustrated felonies.
killed because of a land dispute between them and
that Mandaya should accompany the four men,
otherwise, he would also be killed. Art. 6: Stages of Execution
At about 10:00 o'clock in the evening of the People v. Campuhan(G.R. No. 129433)
same day, Petitioner, Mandaya, Pangasian, Tubio and
Daligdig, all armed with firearms, arrived at Facts:
Palangpangan's house in Katugasan, Lopez Jaena,
Misamis Occidental. At the instance of his

11 | P a g e
On April 25, 1996 at around 4pm while Ma. Judicial depiction of consummated rape has
Corazon Pamintuan was downstairs busy preparing not been confined to the oft-quoted “touching of the
drinks for her two daughters, she heard Crysthel, one female organ,” but has also progressed into being
of her daughters crying, “Ayo’ko, Ayo’ko” prompting described as “the introduction of the male organ into
her to rush upstairs. Thereupon, she saw Primo inside the labia of the pudendum,” or “the bombardment of
her children’s room kneeling in front of her four-year the drawbridge.” But to the SC's mind, the case at bar
old daughter, whose pajamas were already removed, merely constitutes a "shelling of the castle of orgasmic
while his short pants were down to his knees. potency," or a "strafing of the citadel of passion."

Primo was apprehended and was charged Under Art.6, in relation to Art. 335, of the
with statutory rape. Revised Penal Code, rape is attempted when the
offender commences the commission of rape directly
The trial court found him guilty and sentenced by overt acts, and does not perform all the acts of
him to the extreme penalty of death. In convicting the execution which should produce the crime of rape by
accused, the trial court relied quite heavily on the reason of some cause or accident other than his own
testimony of Corazon that she saw Primo with his short spontaneous desistance. All the elements of attempted
pants down to his knees kneeling before Crysthel rape – and only of attempted rape – are present in the
whose pajamas and panty were supposedly “already instant case; hence, the accused should be punished
removed" and that Primo was “forcing his penis into only for it.
Crysthel’s vagina.”

Issue: Valenzuela v. People (G.R. No. 160188)

Is Primo guilty of Consummated rape? Facts:

Decision: Aristotel Valenzuela and Jovy Calderon were


charged with the crime of theft.
Judgment modified into attempted rape.
On 19 May1994, Valenzuela and Calderon
In People vs. Dela Pena, the SC clarified that were seen outside the Super Sale Club inside the SM
the decisions finding a case for rape even if the Complex along North Edsa by Lorenzo Lago, SM
attacker’s penis merely touched the external portions Security Guard. Valenzuela was hauling a push cart
of the female genitalia were made in the context of the with cases of Tide detergent and unloaded
presence or existence of an erect penis capable of full these cases of Tide in an open parking space
penetration. Where the accused failed to achieve an where Calderon was waiting. Valenzuela went inside
erection, had a limp of flaccid penis, or an oversized the supermarket again and came back with more
penis which could not fit into the victim’s vagina, the cases of detergent.
Court nonetheless held that rape was consummated
on the basis of the victim’s testimony that the accused Thereafter, Valenzuela left the parking lot and
repeatedly tried, but in vain, to insert his penis into her haled a taxi. He boarded the cab and told the driver to
vagina and in all likelihood reached the labia of her go to the area where Calderon was waiting. Calderon
pudendum as the victim felt his organ on the lips of her loaded the cases of Tide and boarded the taxi.
vulva, or that the penis of the accused touched the Lago saw all of this and proceeded to stop the
middle part of her vagina. taxi. When Lago asked for a receipt of the
merchandise, the two accused reacted by fleeing on
Thus, touching when applied to rape cases foot. Valenzuela and Calderon were apprehended at
does not simply mean mere epidermal contact, the scene.
stroking or grazing of organs, a slight brush or a
scrape of the penis on the external layer of the victim's Valenzuela and Calderon were both
vagina, or the mons pubis, as in this case. convicted by the trial court of consummated
theft.
There must be sufficient and convincing proof
that the penis indeed touched the labias or slid into the It was only Valenzuela who filed an appeal
female organ, and not merely stroked the external with the Court of Appeals. Petitioner contends that he
surface thereof, for an accused to be convicted of should only be convicted of frustrated theft since at the
consummated rape. As the labias, which are required time he was apprehended, he was never placed in a
to be “touched” by the penis, are by their natural situs position to freely dispose of the articles stolen. The
or location beneath the mons pubis or the vaginal Court of Appeals rejected this contention, hence, this
surface, to touch them with the penis is to attain some Petition for Review.
degree of penetration beneath the surface, hence, the
conclusion that touching the labia majora or the labia Issue:
minora of the pudendum constitutes consummated
rape. Whether or not petitioner Valenzuela is guilty
only of frustrated theft?

12 | P a g e
Upon changing positions, the complainant
Decision: managed to dash into the next room. However, the
appellant chased her until she jumped out of a window
Petition dismissed. Under the statutory and ran to a nearby municipal building where two
definition of theft, free disposal of the stolen items is policemen were on duty. Upon being told what
not a constitutive element of theft. happened, the policemen rushed to the boarding
house, however, they failed to apprehend appellant.
Under Article 308 of the Revised Penal Code,
the crime of theft is defined as follows. “Theft is The complainant was brought to a hospital
committed by any person who, with intent to gain but where she was examined. The medical Certificate
without force or violence against or intimidation of stated Dr. Ma. Luisa Abude findings: “No visible
persons nor force upon things, shall take the property abrasions or marks at the perineal area or over the
of another without the latter’s consent xxx” vulva, errythematous (sic) areas noted surrounding
vaginal orifice, tender, hymen intact;”
On the face of the definition, there is only one
operative act of execution by the actor involved in theft
The trial court convicted the accused of
– the taking of personal property of another. It is also
frustrated rape.
clear from the definition that in order such taking may
be qualified as theft, there must further be present the
descriptive circumstances that the taking was with The accused contends that there is no crime
intent to gain; without force upon things or violence of frustrated rape and the Solicitor General shares the
against or intimidation of persons; and is was without same view.
the consent of the owner of the property.
Issue:
For the purpose of ascertaining whether theft
is susceptible of commissions in the frustrated stage, Whether or not the trial court erred in declaring
the question is again, when is the crime of theft that the crime of frustrated rape was committed by the
produced? There would be all but certain unanimity in accused.
the position that theft is produced when there is
deprivation of personal property due to to its taking by Decision:
one with intent to gain. Viewed from that perspective, it
is immaterial to the product of the felony that the The decision of the Regional Trial Court is
offender, once having committed all the acts of hereby MODIFIED. The accused Ceilito Orita is hereby
execution for theft, is able or unable to freely disposed found guilty beyond reasonable doubt of the crime of
of the property stolen since the deprivation from the rapeand sentenced to reclusion perpetua.There is no
owner alone has already ensued from such acts of debate that rape can be attempted and consummated.
execution. Our concern now is whether or not the crime of rape
can be frustrated. The requisites of a frustrated felony
Indeed the SC, after all, held that unlawful are: (1) that the offender has performed all the acts of
taking is deemed complete from the moment of the execution which would produce the felony and (2) that
offender gains possession of the thing even if he has the felony is not produced due to causes independent
no opportunity to dispose of the same. of the perpetrator's will.

In the crime of rape, from the moment the


People v. Orita (G.R. No. 88724) offender has carnal knowledge of his victim he actually
attains his purpose and, from that moment also all the
essential elements of the offense have been
Facts:
accomplished (consummated). In the consummation of
rape, perfect penetration is not essential. Any
In the early morning of March 20, 1983, penetration of the female organ by the male organ is
Cristina S. Abayan, a 19-year old freshman student at sufficient. Entry of the labia or lips of the female organ,
the St. Joseph's College, arrived at her boarding house without rupture of the hymen or laceration of the
from a party. All of a sudden, appellant held her and vagina is sufficient to warrant conviction. 
poked a knife to her neck. Appellant dragged
complainant up the stairs and with the Batangas knife
In the case of People v. Eriña, We found the
still poked to her neck, they entered complainant's
offender guilty of frustrated rape there being no
room.
conclusive evidence of penetration of the genital organ
of the offended party. However, it appears that this is a
After removing his clothes and ordering her to "stray" decision inasmuch as it has not been reiterated
remove hers, the appellant began to rape the in Our subsequent decisions. Article 335 of the
complainant. However as they were in a position Revised Penal Code, which provides, in its penultimate
where he continued to poke the knife to her, appellant paragraph, for the penalty of death when the rape is
could not fully penetrate her. attempted or frustrated and a homicide is committed
13 | P a g e
by reason or on the occasion thereof. We are of the
opinion that this particular provision on frustrated rape
is a dead provision.  Issue:
Can there be a conspiracy based on the
There was no conclusive evidence of foregoing facts?
penetration of the genital organ of the victim, but
neither was it ruled out. However the medical Decision:
certificate stated that the vulva was erythematous
(which means marked by abnormal redness of the skin Conspiracy need not be proven by direct
due to capillary congestion, as in inflammation) and evidence of prior agreement to commit the
tender. However, Dr. Zamora’s (in place of Dr Abude) crime.Neither it is necessary to show that all the
testimony is merely corroborative and is not an conspirators actually hit and killed the victim. What has
indispensable element in the prosecution of this case. to be shown is that all the participants performed
In a prosecution for rape, the accused may be specific acts with such closeness and coordination as
convicted even on the sole basis of the victim's to unmistakably indicate a common purpose and
testimony if credible.  design. The conspiracy in the instant case was
sufficiently proven by Jed meeting with appellants at
the old airport tower and walking together with them
Art. 8: Conspiracy towards the runway where appellants and Jed
performed acts in unison with each other as to
People v. Quirol (473 SCRA 509) unmistakably reveal a common purpose and design. 

Anent Mario’s defense of alibi, despite


Facts: corroboration from Exequiel Aranas, it is still an
inherently weak defense and cannot prevail over a
On December 4, 1993, in celebration of a fiesta in positive identification from a witness found credible by
Apas, Lahug, Cebu City, a “benefit disco dance” was the trial court. Absent arbitrariness or oversight of
held at the local UCMA Village. Appellants, Juanito some fact or circumstance of significance and
and Mario Quirol, and the two victims, Benjamin Silva influence, we will not interfere with the credence given
and Roel Ngujo, attended. to the testimony of Wilson over that of Mario and that
of Exequiel, as assessments of credibility are generally
 At the dance, Juanito, Mario and Jed were left to the trial court whose proximate contact with
together and drank all through the night with some those who take the witness stand places it in a more
friends. The dance ended just prior to 4 a.m. and competent position to discriminate between true and
prosecution principal witness Wilson Cruz testified that false testimony. Moreover, as correctly discussed by
it was about that time when he was asked by Benjamin the Court of Appeals, the distance between the scene
and Roel to accompany them in escorting some ladies of the crimes and where Mario claims he passed out is
home. not so far away as to prevent him from being physically
present at the place of the crimes or its immediate
Wilson told them to go ahead and that he would vicinity at the time the crimes were committed.
just follow. Wilson was behind them at a distance of 7
to 10 fathoms when the group passed by the house of People v. Comadre (431 SCRA 366)
Jed. From his vantage point, Wilson saw Jed stop the
two victims in front of his house and frisk them.
Facts:
Thereafter, Wilson saw Jed bind Benjamin and
Roel together with a pair of handcuffs and lead them At around 7:00 in the evening of August 6, 1995,
towards the control tower of the old airport of Lahug, Robert Agbanlog, Jimmy Wabe, Gerry Bullanday,Rey
Cebu City. There, the three were met by Juanito and Camat and Lorenzo Eugenio were having a drinking
Mario and together they proceeded to the airport spree on the terrace of the house of Robert’s father,
runway. Barangay Councilman Jaime Agbanlog, situated in
Barangay San Pedro, Lupao, Nueva Ecija. Jaime
Wilson, hidden behind a bush, said he could hear Agbanlog was seated on the banister of the terrace
Benjamin plead for his life. A few seconds later, Jed listening to the conversation of the companions of his
took out his .38 caliber service revolver and shot son.
Benjamin at point-blank range on the head. As
Benjamin fell, Roel was dragged down to his knees As the drinking session went on, Robert and the
since he was handcuffed to Benjamin. Mario then held others noticed appellants Antonio Comadre, George
Roel while Juanito started stabbing him using a Comadre and Danilo Lozano walking. The three
Batangas knife. Jed finished it by shooting Roel. stopped in front of the house. While his companions
looked on, Antonio suddenly lobbed an object which
The lower court and Court of Appeals find that fell on the roof of the terrace. Appellants immediately
there was conspiracy and convicting them despite their fled by scaling the fence of a nearby school.
defense of alibi.
14 | P a g e
The object, which turned out to be a hand able to find a 1997 Nissan Pathfinder. They agreed to
grenade, exploded ripping a hole in the roof of the inspect the vehicle together.
house. Robert Agbanlog, Jimmy Wabe, Gerry
Bullanday, Rey Camat and Lorenzo Eugenio were hit On April 30, 1998, only Elison went to
by shrapnel and slumped unconscious on the floor.  Dagupan City to get the Nissan Pathfinder from his
friend, petitioner Augusto Sim, Jr. Petitioner told
The undisputed facts show that when Antonio Elison that the Nissan Pathfinder was given to him by
Comadre was in the act of throwing the hand grenade, a customer in payment of a debt and had been used
George Comadre and Danilo Lozano merely looked on only for a year.
without uttering a single word of encouragement or
performed any act to assist him. The trial court held Elison brought the 1997 Nissan Pathfinder to
that the mere presence of George Comadre and San Pablo City. Private complainant at first did not like
Danilo Lozano provided encouragement and a sense the vehicle since it was not the brand he was looking
of security to Antonio Comadre, thus proving the for. Elison said that his kumpadre would look at the
existence of conspiracy. vehicle as the latter was also interested in it.

Issue: Private complainant decided to buy the 1997


Nissan Pathfinder at the agreed price of P480,000.00.
Can there be a conspiracy based on the foregoing The amount was paid in five checks issued by Fe
facts? Ilagan. One check was dated May 6, 1998 in the sum
of P350,000.00, and four checks in the sum of
Decision: Similar to the physical act constituting the P32,500.00 each was dated June 6, July 6, August 6
crime itself, the elements of conspiracy must be and September 6, all in 1998.
proven beyond reasonable doubt. Settled is the rule
that to establish conspiracy, evidence of actual Elison gave private complainant photocopies
cooperation rather than mere cognizance or approval of the Certificate of Registration (C.R.) and Official
of an illegal act is required. Receipt (O.R.) issued by the Land Transportation
Office (LTO) showing the name of the owner as one
A conspiracy must be established by positive and Henry Austria. After a week, Elison brought the deed
conclusive evidence. It must be shown to exist as of sale which private complainant signed without the
clearly and convincingly as the commission of the signature of the owner, Henry Austria. After private
crime itself. Mere presence of a person at the scene of complainant signed the deed of sale, he gave it back
the crime does not make him a conspirator for to Elison to be brought back to Dagupan City for
conspiracy transcends companionship. signing by the owner/vendor and transfer of
registration in the name of private complainant.
The evidence shows that George Comadre and
Danilo Lozano did not have any participation in the On June 7, 1998, Elison returned and
commission of the crime and must therefore be set delivered to private complainant the deed of sale
free. Their mere presence at the scene of the crime as signed by the owner/vendor, together with the new
well as their close relationship with Antonio are C.R. and O.R. issued by the LTO of Lingayen,
insufficient to establish conspiracy considering that Pangasinan in the name of private complainant. The
they performed no positive act in furtherance of the checks given by private complainant in payment of the
crime. vehicle were deposited by petitioner in his name at
Solidbank-Dagupan Branch. All five checks were
debited in favor of petitioner. After receiving the
Sim v. CA (428 SCRA 459) registration papers from Elison, private complainant
was eventually able to use the Nissan Pathfinder.

Facts: On October 28, 1998, private complainants


vehicle was apprehended by Anti-Carnapping
Private complainant Jay Byron Ilagan is a tire operatives of the Philippine National Police (ANCAR
supplier. He had been dealing with accused Elison NCRTMO). The vehicle and its registration papers
Villaflor for twenty years, as the latter is engaged in the were inspected and thereafter brought to Camp
same business of selling tires and rims. Crame. It turned out that the vehicle was a "hot car."

In March 1998, private complainant talked to Thereafter, Elison and petitioner were charged
Elison and expressed his interest in buying a vehicle. and convicted with estafa. Further the court also ruled
Elison told him that he knew someone who sells that conspiracy is present. With this, the accused
vehicles at a cheap price, and that he had bought a appealed.
Toyota Tamaraw FX at lower than the market price.
Private complainant then asked Elison to ask if there Issue:
was an Isuzu pick-up for sale. A month later, Elison
called private complainant to inform him that he was

15 | P a g e
Whether there was conspiracy between Facts:
petitioner and Elison Villaflor in defrauding private
complainant Jay Byron Ilagan? Sanchez's account of the facts shows that he
and Jamero were tenants of adjacent lots located in
Decision: San Jose, Mahayag, Zamboanga del Sur. At about
7:00 o'clock in the morning of September 4, 1993,
Even in the absence of direct evidence of prior Sanchez saw Jamero destroying the dike which served
agreement to commit the crime, conspiracy may be as the boundary between the two lots. Sanchez
deduced from the acts of the perpetrators before, confronted Jamero and told the latter that he was
during and after the commission of the crime, which encroaching on his land. Jamero struck him with a
are indicative of a common design, concerted action shovel. The shovel got stuck in the mud so Jamero
and concurrence of sentiments. Conspiracy is deemed resorted to throwing mud at Sanchez. Fighting back,
implied when the malefactors have a common purpose Sanchez hacked Jamero with a bolo, resulting in the
and were united in its execution. Spontaneous latter's death. Sanchez then proceeded to the
agreement or active cooperation by all perpetrators at municipal building to surrender upon the advice of his
the moment of the commission of the crime is sufficient son-in-law.
to create joint criminal responsibility.
In Erquiaga v. Court of Appeals, we ruled that According to the OSG, Jamero's attack on
conspiracy, as a rule, has to be established with the Sanchez was unsuccessful because the latter was
same quantum of proof as the crime itself and shown able to evade it and Jamero's shovel got stuck in the
as clearly as the commission of the crime. However, mud. Jamero fled toward the ricefield when Sanchez
conspiracy need not be shown by direct evidence, but unsheathed his bolo. Sanchez pursued him and struck
may take the form of circumstances which, if taken his head with a bolo. Jamero fell down but was able to
together, would conclusively show that the accused stand up again. He ran away but after a short distance,
came to an agreement to commit a crime and decided fell down again. Sanchez approached him and stabbed
to carry it out with their full cooperation and him several times. Not satisfied, Sanchez pushed
participation. Jamero's face down into the knee-deep mud. After
Jamero's aggression ceased when he fled and left his
As correctly pointed out by the appellate court, shovel stuck in the mud, there was no longer any
petitioners actions in relation to the fraudulent sale of justification for Sanchez to go after him and hack him
the Nissan Pathfinder to private complainant clearly to death.
established conspiracy as alleged in the information,
which acts transcend mere knowledge or friendship Issue: Whether or not unlawful aggression, if not
with co-accused Elison. Notwithstanding the fact that it continuous, does not constitute aggression warranting
was only Elison who dealt with or personally self-defense?
transacted with private complainant until the time the
sale was consummated, by his own testimony Decision:
petitioner admitted all the acts by which he actively
cooperated and not merely acquiesced in perpetrating There
the fraud upon private complainant. That petitioner is a can be no self-defense, complete or incomplete,
conspirator having joint criminal design with Elison is unless the accused proves the first essential requisite
evident from the fact that as between them, both knew —unlawful aggression on the part of the victim.
that petitioner was the person selling the vehicle under Unlawful aggression presupposes an actual, sudden
the false pretense that a certain Henry Austria was the and unexpected or imminent danger on the life and
registered owner.Petitioner, together with Elison, limb of a person – a mere threatening or intimidating
clearly deceived private complainant in order to attitude is not sufficient. There must be actual physical
defraud him in the amount of P480,000.00, to the force or a threat to inflict physical injury. In case of a
latter’s damage and prejudice. In addition, the acts of threat, it must be offensive and positively strong so as
petitioner in deliberately misrepresenting himself to to display a real, not imagined, intent to cause injury.
private complainant as having the necessary authority Aggression, if not continuous, does not constitute
to possess and sell to the latter the vehicle so that he aggression warranting self-defense.
could collect from him P480,000.00 only to renege on
that promise and for failure to reimburse the said In this case, the twin circumstances of
amount he collected from private complainant, despite Jamero's shovel getting stuck in the mud and his
demand, amount to estafa punishable under Art. 315, running away from Sanchez convincingly indicate that
par. 2 (a). there was no longer any danger to the latter's life and
limb which could have justified his pursuit of Jamero
and subsequent hacking and killing of the latter.
Art. 11: Justifying Circumstances

Self-defense Sanchez's failure to prove unlawful aggression


by Jamero and the prosecution's evidence conclusively
People v. Sanchez (G.R. No. 161007) showing that it was Sanchez who was the unlawful
aggressor completely discounts Sanchez's claim of
16 | P a g e
self-defense. Even incomplete self-defense by its very of seconds or even simultaneously. Rogelio bore no
nature and essence would always require the superhuman power to slow down time or to prevent the
attendance of unlawful aggression initiated by the events from unfolding at virtual warp speed, to be able
victim which must clearly be shown. to assess with measured certainty the appropriate
commensurate response due to each of his
Soplente v. People (G.R. No. 152715) aggressors. Even those schooled in the legal doctrines
of self-defense would, under those dire circumstances,
be barely able to discern the legally defensible
Facts: response and immediately employ the same. Our laws
on self-defense are supposed to approximate the
natural human responses to danger, and not serve as
The cousins, Rogelio and Nicanor, watched our inconvenient rulebook based on which we should
the amateur singing contest being held near the Sta. acclimatize our impulses in the face of peril.
Cruz Chapel. While engrossed with the singing
contest, they were approached by two persons from It would be wrong to compel Rogelio to have
the group of Leyson who then tapped Nicanor’s discerned the appropriate calibrated response to
shoulder. They insisted on bringing Nicanor along with Notarte’s kicking when he himself was staring at the
them so Nicanor called for Rogelio’s help. The latter evil eye of danger. That would be a gargantuan
immediately intervened to stop the two from harassing demand even for the coolest under pressure.
Nicanor.
Urbano v. People (G.R. No. 182750)
At about past eleven o’clock in the evening
(11:00 p.m.), before the conclusion of the amateur
singing contest, Rogelio and Nicanor decided to go Facts:
home. At past midnight, Bukay (their cousin’s wife)
asked Rogelio and Nicanor to accompany her in Petitioner
looking for her children who had watched the singing Urbano testified being, in the afternoon of September
contest. They obliged but before they had gone about 28, 1993, in the nearby town of Bugallon for a picnic.
three hundred meters, Nicanor separated from them to He was with Tomelden and several others, including
buy cigarettes from a nearby store. Rogelio and Bukay Dominador Navarro, Chairperson of Lingayen Water
went onwards but at a distance of about fifty meters District. At a restaurant in Bugallon, the group ordered
from the stage, Rogelio stopped and Bukay proceeded goat’s meat and drank beer. When it was time to
alone to look for her children. A few minutes later, depart, Navarro asked petitioner to inform Tomelden,
Bukay appeared with the children and they all headed then seated in another table, to prepare to leave.
home.
While on the way home, Rogelio suddenly When so informed, Tomelden insulted
found himself surrounded by around ten persons led petitioner, telling the latter he had no business
by Leyson. He shouted at Nicanor to run and the latter stopping him from further drinking as he was paying for
immediately scampered away. Leyson drew his gun his share of the bill. Chastised, petitioner returned to
and fired at Rogelio but the latter was able to parry it his table to report to Navarro. At that time, petitioner
by tapping the base of Leyson’s hand holding the gun. saw that Tomelden had already consumed 17 bottles
Forthwith, Rogelio stabbed Leyson once. He was of beer. In all, the group stayed at the picnic place for
kicked by Notarte immediately after he stabbed three and a half hours before returning to the LIWAD.
Leyson. Rogelio also stabbed Notarte. Rogelio Upon reaching the LIWAD compound, Tomelden
managed to escape after that and he sought refuge in allegedly slapped and hurled insults at him, calling him
the house of Susing (their cousin). "sipsip" just to maintain his employment as Navarro’s
Before dawn, a policeman arrived at Susing’s tricycle driver. Tomelden allegedly then delivered
house and Rogelio voluntarily gave himself up. The several fist and kick blows at petitioner, a couple of
knife he used was also turned over to the police. He which hit him despite his evasive actions. Petitioner
was brought to the police substation at Lagao. maintained that he only boxed the victim in retaliation,
landing that lucky punch in the course of parrying the
Issue: latter’s blows.
Thereafter, Tomelden went to the hospital
Whether or not our laws on self-defense are several times complaining of dizziness, headache, and
suppose to approximate the natural human responses other pains. The last time he went to the hospital,
to danger. things turned for the worst. Tomelden died due, per Dr.
Arellano, to "cardio-respiratory arrest secondary to
Decision: cerebral concussion with resultant cerebral
hemorrhage due to mauling incident."
At the commencement of the attack, Rogelio
could not have been obliged to view Notarte, or any
other member of the posse for that matter, as a less Orje Salazar, their co-worker, attests to the
menacing threat than Leyson. We have to understand provocative acts of Tomelden and to his being the
that these events occurred spontaneously in a matter aggressor.

17 | P a g e
embracing Senando who was continuously hacking
Issue: Reynaldo. Thereafter, Senando shoved Ramon to the
ground and as if further enraged by the intrusion, he
turned his bolo on the fallen Ramon. Ricardo
Whether or not the victim’s insulting remarks screamed, "tama na yan, mga kapatid ko ‘yan." But the
directed at the accused, and uttered immediately assailant would not be pacified as he hacked Ramon
before the fist fight constitute sufficient provocation? on the chest. At this point, Servando, the brother of
Senando, threw an axe at him but Reynaldo picked it
Decision: up and smashed Senando with it.

When The trial court found the accused guilty of


the law speaks of provocation either as a mitigating homicide and frustrated homicide. According to the
circumstance or as an essential element of self- trial court, the denial of Ricardo was self-serving and
defense, the reference is to an unjust or improper calculated to extricate himself from the predicament he
conduct of the offended party capable of exciting, was in. Further, the trial court added that the wounds
inciting, or irritating anyone; it is not enough that the allegedly received by Ricardo in the hands of the
provocative act be unreasonable or annoying; the victim, Senando Iguico, if at all there were any, did not
provocation must be sufficient to excite one to commit prove that Senando was the aggressor for the wounds
the wrongful act and should immediately precede the were inflicted while Senando was in the act of
act. This third requisite of self-defense is present: (1) defending himself from the aggression of Ricardo and
when no provocation at all was given to the aggressor; his co-conspirators.
(2) when, even if provocation was given, it was not The Court of Appeals sustained the conviction
sufficient; (3) when even if the provocation was of accused Ricardo with modification that his
sufficient, it was not given by the person defending conviction for the wounding of Amelia Iguico, should
himself; or (4) when even if a provocation was given by be for attempted homicide only.
the person defending himself, it was not proximate and
immediate to the act of aggression. Issue:

In the instant case, Tomelden’s insulting Whether or not there was a valid defense of
remarks directed at petitioner and uttered immediately relatives?
before the fist fight constituted sufficient provocation.
This is not to mention other irritating statements made Decision:
by the deceased while they were having beer in
Bugallon. Petitioner was the one provoked and Petitioner invokes the justifying circumstance
challenged to a fist fight. of defense of relatives under Art. 11, par. (2), of The
Revised Penal Code. The essential elements of this
Defense of Relatives justifying circumstance are the following: (a) unlawful
aggression; (b) reasonable necessity of the means
Balunuecov. CA and People (G.R. No. 126968) employed to prevent or repel it; and, (c) in case the
provocation was given by the person attacked, the one
making the defense had no part therein.
Facts:
Of the three (3) requisites of defense of
On May 2, 1982 at around 6:00 o’clock in the relatives, unlawful aggression is a condition sine qua
evening Amelia Iguico saw accused Reynaldo, his non, for without it any defense is not possible or
father Juanito and brothers Ricardo and Ramon, all justified. In order to consider that an unlawful
surnamed Balunueco, and one Armando Flores aggression was actually committed, it is necessary that
chasing her brother-in-law Servando Iguico. With the an attack or material aggression, an offensive act
five (5) individuals in hot pursuit, Servando scampered positively determining the intent of the aggressor to
into the safety of Amelia’s house. Meanwhile Senando cause an injury shall have been made; a mere
went out of the house fully unaware of the commotion threatening or intimidating attitude is not sufficient to
going on outside. Upon seeing Senando, Reynaldo justify the commission of an act which is punishable
turned his attention on him and gave chase. Senando per se, and allow a claim of exemption from liability on
instinctively fled towards the fields but Reynaldo, the ground that it was committed in self-defense or
Ricardo, and Armando cornered him and ganged up defense of a relative. It has always been so recognized
on him. To shield him from further violence, Amelia put in the decisions of the courts, in accordance with the
her arms around her husband but it was not enough to provisions of the Penal Code.
detract Ricardo from his murderous frenzy. Amelia was
also hit on the leg. Having admitted the killing of the victim, petitioner has
the burden of proving these elements by clear and
In his defense, accused Ricardo invoke convincing evidence. He must rely on the strength of
defense of relatives. He testified that at that time he his own evidence and not on the weakness of that of
was fetching water he heard somebody shout. When the prosecution, for even if the prosecution evidence is
he hurried to the place, he saw his brother Ramon
18 | P a g e
weak it cannot be disbelieved if the accused has death of Contreras was a necessary consequence of
admitted the killing. the due performance of his duty as a policeman is
essential to exempt him from criminal liability.
In the case at bar, petitioner Ricardo utterly
failed to adduce sufficient proof of the existence of a To be sure, acts in the fulfillment of a duty,
positively strong act of real aggression on the part of without more, do not completely justify the petitioner’s
the deceased Senando. With the exception of his self- firing the fatal gunshot at the victim. True, petitioner, as
serving allegations, there is nothing on record that one of the policemen responding to a reported robbery
would justify his killing of Senando. then in progress, was performing his duty as a police
officer as well as when he was trying to effect the
Fulfilment of a Duty arrest of the suspected robber and in the process,
fatally shoot said suspect, albeit the wrong man.
Mamangun v. People (GR 149152) However, in the absence of the equally necessary
justifying circumstance that the injury or offense
Facts: committed be the necessary consequence of the due
performance of such duty, there can only be
On July 31, 1992, at about 8:00 in the evening, incomplete justification, a privileged mitigating
a certain Liberty Contreras was heard shouting, circumstance under Articles 13 and 69 of the Revised
“Magnanakaw…Magnanakaw.” Several residents Penal Code.
responded and thereupon chased the suspect who
entered the yard of Antonio Abacan and proceeded to Baxinela v. People (G.R. No. 149652)
the rooftop of Abacan’s house.

At about 9:00 o’clock that same evening, the Facts:


desk officer of the Meycauayan PNP Police Station,
upon receiving a telephone call that a robbery-holdup Petitioner SPO2 Eduardo L. Baxinela was in a
was in progress in Brgy. Calvario, immediately pub drinking with two other policemen in as early as
contacted and dispatched to the scene the crew 11:00 p.m. of October 18, 1996. At around 12:00 a.m.
including herein petitioner PO2 Rufino S. Mamangun. to 12:30 a.m. there was a minor altercation between
With the permission of Abacan, petitioner Mamangun, the deceased Sgt. Lajo and another customer at the
and two others went to the rooftop of the house pub but eventually the two were able to patch things
whereat the suspect was allegedly taking refuge. up. While on his way out, Lajo was followed by
Braxinela with a gun already drawn out. From behind,
The three policemen, each armed with a Baxinela held Lajo’s left arm and asked why he was
drawn handgun, searched the rooftop. There, they saw carrying a gun. Thereafter an explosion coming from
a man whom they thought was the robbery suspect. At Baxinela’s gun was heard. Lajo, still standing, took two
that instance, petitioner Mamangun, who was walking steps and then fell down.
ahead of the group, fired his handgun once, hitting the
man. The man turned out to be Gener Contreras Issue:
(Contreras) who was not the robbery suspect.
Whether or not fulfilment of duty may validly
Contreras died from the gunshot wound. be invoked by the petitioner?

Issue: Decision:

Whether or not the shooting in question was No. In order to avail of this justifying
done in the performance of a duty or in the lawful circumstance it must be shown that: 1) the accused
exercise of a right or office? acted in the performance of a duty or in the lawful
exercise of a right or office; and 2) the injury caused or
Decision: the offense committed is the necessary consequence
of the due performance of duty or the lawful exercise
No. The justifying circumstance of fulfillment of of a right or office. While the first condition is present,
duty under paragraph 5, Article II, of the Revised Penal the second is clearly lacking. Baxinela’s duty was to
Code may be invoked only after the defense investigate the reason why Lajo had a gun tucked
successfully proves that: (1) the accused acted in the behind his waist in a public place. This was what
performance of a duty; and (2) the injury inflicted or Baxinela was doing when he confronted Lajo at the
offense committed is the necessary consequence of entrance, but perhaps through anxiety, edginess or the
the due performance or lawful exercise of such duty. desire to take no chances, Baxinela exceeded his duty
by firing upon Lajo who was not at all resisting. The
Concededly, the first requisite is present in this shooting of Lajo cannot be considered due
case. Petitioner, a police officer, was responding to a performance of a duty if at that time Lajo posed no
robbery-holdup incident. His presence at the situs of serious threat or harm to Baxinela or to the civilians in
the crime was in accordance with the performance of the pub.The Court will, however, attribute to Baxinela
his duty. However, proof that the shooting and ultimate the incomplete defense of fulfillment of a duty as a

19 | P a g e
privileged mitigating circumstance. In Lacanilao v. person was pointless as Restituto Bergante was not in
Court of Appeals, it was held that if the first condition is his house. As regards the second requisite, there can
fulfilled but the second is wanting, Article 69 of the be no question that the killing of Freddie Ganancial
Revised Penal Code is applicable so that the penalty was not a necessary consequence of the arrest to be
lower than one or two degrees than that prescribed by made on Restituto Bergante.
law shall be imposed.
Obedience to an Order
Angcaco v. People (G.R. No. 146664)
Tabuena v. Sandiganbayan (G.R. Nos. 103501-03)

Facts:
Facts:
At around 4 o'clock in the morning of
September 25, 1980, Noe Bergante and his brother Luis A. Tabuena and Adolfo M. Peralta were
Noel Bergante and his cousin Freddie Ganancial were convicted by the Sandiganbayan of malversation under
awakened by the sound of gunfire while they were Article 217 of the Revised Penal Code in the total
asleep in their house. Petitioner John Angcaco and his amount of P55 Million of the Manila International
co-accused were serving a warrant of arrest issued on Airport Authority (MIAA) funds during their incumbency
Restituto Bergante, who was wanted in connection as General Manager and Acting Finance Services
with a robbery case. Noel informed the policemen that Manager, respectively, of MIAA.
his father was not in the house, having gone to Puerto
Princesa. One of them ordered the men in the house Then President Marcos instructed Tabuena
to come out. Noel accordingly went to the gate and over the phone to pay directly to the president's office
later called Noe to also come out of the house. Noe and in cash what the MIAA owes the Philippine
and his cousin, Freddie Ganancial, did as bidden. National Construction Corporation (PNCC), to which
Tabuena replied, "Yes, sir, I will do it." About a week
Once they were outside the house, Noe and later, Tabuena received from Mrs. Fe Roa-Gimenez,
Freddie were flanked by petitioner Angcaco on the then private secretary of Marcos, a Presidential
right side and accused Ramon Decosto on the left Memorandum dated January 8, 1986 reiterating in
side. Decosto pointed an armalite at the two and black and white such verbal instruction, directed to pay
warned them not to run. Noe and Freddie joined Noel immediately the Philippine National Construction
Bergante. Protacio Edep approached Freddie saying, Corporation, thru this Office, the sum of FIFTY FIVE
"You are tough," and pushed him. Then, shots rang MILLION (P55,000,000.00) PESOS in cash as partial
out from the armalite and short firearm of Decosto and payment of MIAA's account with said Company signed
Edep, as a result of which Freddie Ganancial turned by the then President Marcos.
around and dropped to the ground face down.
In obedience to President Marcos' verbal
Issue: instruction and memorandum, Tabuena, with the help
of Dabao and Peralta, caused the release of P55
Whether or not the justifying circumstance of Million of MIAA funds
fulfilment of duty is applicable in this case?
The disbursement of the P55 Million was, as
Decision: described by Tabuena and Peralta themselves, "out of
the ordinary" and "not based on the normal
No. For this justifying circumstance to be procedure". Not only were there no vouchers prepared
appreciated, the following must be established: (1) that to support the disbursement, the P55 Million was paid
the offender acted in the lawful exercise of a right or a in cold cash. Also, no PNCC receipt for the P55 Million
duty; and (b) that the injury or offense committed be was presented.
the necessary consequence of the due performance of
such right or office. Issue:
In this case, the mission of petitioner and his
colleagues was to effect the arrest of Restituto Whether or not the petitioners’ defense of
Bergante. The standard procedure in making an arrest good faith is tenable?
was, first, to identify themselves as police officers and Decision:
to show the warrant to the arrestee and to inform him
of the charge against him, and, second, to take the Yes. It is settled that this is a valid defense in a
arrestee under custody. But, it was not shown here prosecution for malversation for it would negate
that the killing of Ganancial was in furtherance of such criminal intent on the part of the accused.To constitute
duty. No evidence was presented by the defense to a crime, the act must, except in certain crimes made
prove that Ganancial attempted to prevent petitioner such by statute, be accompanied by a criminal intent,
and his fellow officers from arresting Restituto or by such negligence or indifference to duty or to
Bergante. There was in fact no clear evidence as to consequences as, in law, is equivalent to criminal
how Freddie Ganancial was shot. Indeed, as already intent. The maxim is actus non facit reum, nisi mens sit
stated, any attempt by the victim to arrest the wanted rea — a crime is not committed if the mind of the

20 | P a g e
person performing the act complained of is wife, but seeks exoneration from criminal liability by
innocent.Ordinarily, evil intent must unite with an interposing the defense of insanity.
unlawful act for there to be a crime. Actus non facit
reum, nisi mens sit rea. There can be no crime when Issue:
the criminal mind is wanting.
Whether or not the court a quo erred in not
Tabuena had no other choice but to make the giving probative weight to the testimony and
withdrawals, for that was what the MARCOS psychiatric evaluation of Dr. Maria Mercedita Mendoza
Memorandum required him to do. He could not be finding the accused-appellant to be suffering from
faulted if he had to obey and strictly comply with the psychosis or insanity classified under schizophrenia,
presidential directive, and to argue otherwise is paranoid type?
something easier said than done. Marcos was
undeniably Tabuena's superior — the former being Whether or not the court a quo erred in
then the President of the Republic who unquestionably disregarding accused-appellant's defense of insanity?
exercised control over government agencies such as
the MIAA and PNCC. In other words, Marcos had a Decision:
say in matters involving inter-government agency
affairs and transactions, such as for instance, directing The Supreme Court ruled that insanity
payment of liability of one entity to another and the presupposes that the accused was completely
manner in which it should be carried out. And as a deprived of reason or discernment and freedom of will
recipient of such kind of a directive coming from the at the time of the commission of the crime. A
highest official of the land no less, good faith should be defendant in a criminal case who relies on the defense
read on Tabuena's compliance, without hesitation nor of mental incapacity has the burden of establishing the
any question, with the MARCOS Memorandum. fact of insanity at the very moment when the crime was
Tabuena therefore is entitled to the justifying committed. Only when there is a complete deprivation
circumstance of "Any person who acts in obedience to of intelligence at the time of the commission of the
an order issued by a superior for some lawful crime should the exempting circumstance of insanity
purpose." The subordinate-superior relationship be considered.
between Tabuena and Marcos is clear. And so too, is
the lawfulness of the order contained in the MARCOS The presumption of law always lies in favor of
Memorandum, as it has for its purpose partial payment sanity and, in the absence of proof to the contrary,
of the liability of one government agency (MIAA) to every person is presumed to be of sound mind.
another (PNCC). Accordingly, one who pleads the exempting
circumstance of insanity has the burden of proving it.
Good faith in the payment of public funds relieves a Failing this, one will be presumed to be sane when the
public officer from the crime of malversation. crime was committed.

A perusal of the records of the case reveals


that appellant's claim of insanity is unsubstantiated and
Art.12: Exempting Circumstances wanting in material proof. Testimonies from both
prosecution and defense witnesses show no
Insanity/Imbecility substantial evidence that appellant was completely
deprived of reason or discernment when he
People v. Rubiños (G.R. No. 138453) perpetrated the brutal killing of his wife.

As can be gleaned from the testimonies of the


Facts: prosecution witnesses, a domestic altercation
preceded the fatal stabbing. Thus, it cannot be said
Where the law prescribes a penalty consisting that appellant attacked his wife for no reason at all and
of two indivisible penalties, as in the present case for without knowledge of the nature of his action. To be
parricide with unintentional abortion, the lesser one sure, his act of stabbing her was a deliberate and
shall be applied in the absence of any aggravating conscious reaction to the insulting remarks she had
circumstances. Hence, the imposable penalty here hurled at him as attested to by their 15-year-old son
is reclusion perpetua, not death. Lorenzo Robiños.

In an Information dated May 31, 1995, Furthermore, appellant was obviously aware of
appellant was accused of killing his pregnant wife and what he had done to his wife. He was even bragging to
the fetus inside her. When arraigned on July 27, 1995, her brother, Benjamin Bueno, how he had just killed
appellant, with the assistance of his counsel, pleaded her.
not guilty. After due trial, the RTC convicted him.
Finally, the fact that appellant admitted to
Appellant does not refute the factual responding law enforcers how he had just killed his
allegations of the prosecution that he indeed killed his wife may have been a manifestation of repentance and
remorse -- a natural sentiment of a husband who had
21 | P a g e
realized the wrongfulness of his act. His behavior at Antonio was on the left side, while Simplicio was
the time of the killing and immediately thereafter is seated near the door, on the right side of Roger.
inconsistent with his claim that he had no knowledge of
what he had just done. Barangay Kagawad Rolando All of a sudden, accused-appellant entered the
Valdez validated the clarity of mind of appellant when room; uttered Roger's nickname ("Jer") and
the latter confessed to the former and to the police immediately attacked him with a knife, but Roger was
officers, and even showed to them the knife used to able to parry the thrust and was stabbed instead on
stab the victim. Clearly, the assault of appellant on his the right forearm. Accused-appellant then stabbed
wife was not undertaken without his awareness of the Elsa Rodriguez on the chest and said, "Ako akabales
atrocity of his act. den, Elsa." (I had my revenge, Elsa). Thereafter,
Similarly, an evaluation of the testimonies of accused-appellant fled, leaving the stunned Simplicio
the defense witnesses hardly supports his claim of and Antonio unharmed.
insanity. The bulk of the defense evidence points to his
allegedly unsound mental condition after the Prosecution witness Roger Cabiguen testified
commission of the crime. Except for appellant's 19- that sometime in 1980, accused-appellant suspected
year-old son Federico Robiños, all the other defense him of killing his pet dog. In 1989, accused-appellant
witnesses testified on the supposed manifestations of courted Elsa but she jilted him. On one occasion, Elsa
his insanity after he had already been detained in spat on and slapped accused-appellant.
prison.
Accused-appellant's defense of insanity was
To repeat, insanity must have existed at the anchored on the following facts:
time of the commission of the offense, or the accused
must have been deranged even prior thereto. On March 11, 1991, Dr. Manuel Bilog, City
Otherwise he would still be criminally Health Officer I of Puerto Princesa City interviewed
responsible. Verily, his alleged insanity should have accused-appellant and thereafter made the following
pertained to the period prior to or at the precise conclusions and recommendation, that subject patient
moment when the criminal act was committed, not at be committed to the National Mental Hospital, Metro
any time thereafter. Manila for proper medical care and evaluation soonest.

Indeed, when insanity is alleged as a ground The defense offered in evidence the April 27,
for exemption from criminal responsibility, the evidence 1992 medical findings on accused-appellant by Dr.
must refer to the time preceding the act under Guia Melendres of the National Center for Mental
prosecution or to the very moment of its execution. If Health, pertinent portion of which reads: In view of the
the evidence points to insanity subsequent to the foregoing history, observations, physical mental and
commission of the crime, the accused cannot be psychological examinations the patient Enrico Valledor
acquitted. y Andusay is found suffering from Psychosis or
Insanity classified under Schizophrenia. This is a
Hence, appellant who invoked insanity should thought disorder characterized by deterioration from
have proven that he had already been completely previous level of functioning, auditory hallucination,
deprived of reason when he killed the victim. Verily, ideas of reference, delusion of control, suspiciousness,
the evidence proffered by the defense did not indicate poor judgment and absence of insight. Likewise, he is
that he had been completely deprived of intelligence or found to be suffering from Psychoactive Substance
freedom of will when he stabbed his wife to death. Use Disorder, Alcohol, abuse. This is characterized by
Insanity is a defense in the nature of a confession or a maladaptive pattern of psychoactive substance use
avoidance and, as such, clear and convincing proof is indicated by continued use despite knowledge of
required to establish its existence. Indubitably, the having a persistent or recurrent social, occupational,
defense failed to meet the quantum of proof required psychological or physical problems.
to overthrow the presumption of sanity.
Issue:

People v. Valledor (G.R. No. 129291) Whether or not the lower court erred in
convicting the accused despite the fact that when he
allegedly committed the offense charged he was
Facts: mentally ill, out of his mind or insane?

On March 6, 1991, at around 1:45 in the Decision:


afternoon, Roger Cabiguen was in his house at Burgos
Street, Barangay Tagumpay, Puerto Princesa City. He The Supreme Court ruled that in considering a
was working on a lettering job inside his bedroom plea of insanity as a defense, the starting premise is
together with his first cousin, then 25-year old Elsa that the law presumes all persons to be of sound mind.
Rodriguez, and his friends, Simplicio Yayen and Otherwise stated, the law presumes all acts to be
Antonio Magbanua. Roger was working at his table voluntary, and it is improper to presume that acts were
and seated on his bed while Elsa was across the table. done unconsciously.

22 | P a g e
Since the presumption is always in favor of that accused-appellant was suffering from a mental
sanity, he who invokes insanity as an exempting disorder after the commission of the crime, has no
circumstance must prove it by clear and positive bearing on his liability. What is decisive is his mental
evidence. And the evidence on this point must refer to condition at the time of the perpetration of the offense.
the time preceding the act under prosecution or to the Failing to discharge the burden of proving that he was
very moment of its execution. legally insane when he stabbed the victims, he should
be held liable for his felonious acts.
Insanity is evinced by a deranged and
perverted condition of the mental faculties which is Minority
manifested in language and conduct. An insane
person has no full and clear understanding of the Llave v.People (G.R. No. 166040)
nature and consequences of his acts. Hence, insanity
may be shown by the surrounding circumstances fairly Facts:
throwing light on the subject, such as evidence of the
alleged deranged person's general conduct and On Sept. 24, 2002, on an errand from her
appearance, his acts and conduct consistent with his mother, the victim, who was only 7 years old at that
previous character and habits, his irrational acts and time, proceeded to their house, where the accused
beliefs, as well as his improvident bargains. The waited for her, and accosted her; he proceeded to
vagaries of the mind can only be known by outward sexually abuse her, while the victim cried for help.
acts, by means of which we read thoughts, motives
and emotions of a person, and through which we A barbecue vendor nearby heard her cries and
determine whether the acts conform to the practice of came to the scene; the accused fled, and the vendor
people of sound mind. told the victim to tell her parents what happened.
In the case at bar, accused-appellant failed to
discharge the burden of overcoming the presumption Together with her parents, the victim went to
of sanity at the time of the commission of the crime. the police and reported the incident; the vendor also
The following circumstances clearly and unmistakably testified to what he saw during that time.
show that accused-appellant was not legally insane
when he perpetrated the acts for which he was The medical examiner found no injury on the
charged: 1) Simplicio Yayen was positioned nearest to hymen and perineum, but found scanty yellowish
accused-appellant but the latter chose to stab Roger discharge between the labia minora; there was also
and Elsa; 2) Accused-appellant called out the fresh abrasion of the perennial skin at 1
nickname of Roger before stabbing him; 3) Simplicio o’clock position near the anal opening.
Yayen and Antonio Magbanua who were likewise
inside the room were left unharmed; 4) Accused- The trial court found the victim guilty, declaring
appellant, a spurned suitor of Elsa, uttered the words, that he acted with discernment, but crediting him with
"Ako akabales den, Elsa." (I had my revenge, Elsa) the special mitigating circumstance of minority.
after stabbing her; and 5) Accused-appellant hurriedly
left the room after stabbing the victims. Issue:

Evidently, the foregoing acts could hardly be W/N accused had carnal knowledge of the
said to be performed by one who was in a state of a victim, and if yes, whether he acted with discernment,
complete absence of the power to discern. Judging being a minor of age more than 9 years old but less
from his acts, accused-appellant was clearly aware than 15?
and in control of what he was doing as he in fact
purposely chose to stab only the two victims. Two Decision: YES
other people were also inside the room, one of them
was nearest to the door where accused-appellant Penetration, no matter how slight, or the mere
emerged, but the latter went for the victims. His introduction of the male organ into the labia of the
obvious motive of revenge against the victims was pudendum, constitutes carnal knowledge.  Hence,
accentuated by calling out their names and uttering the even if the penetration is only slight, the fact that the
words, "I had my revenge" after stabbing them. Finally, private complainant felt pains, points to the conclusion
his act of immediately fleeing from the scene after the that the rape was consummated.
incident indicates that he was aware of the wrong he
has done and the consequence thereof. While it is true that medical examiner did not find
any abrasion or laceration in the private complainant’s
Accused-appellant's acts prior to the stabbing genitalia, such fact does not negate the latter’s
incident to wit: crying; swimming in the river with his testimony the petitioner had carnal knowledge of her.
clothes on; and jumping off the jeepney; were not The absence of abrasions and lacerations does not
sufficient to prove that he was indeed insane at the disprove sexual abuses, especially when the victim is
time of the commission of the crime. As consistently a young girl as in this case.
held by this Court, "A man may act crazy but it does The court have held that when the offended party is
not necessarily and conclusively prove that he is young and immature, from the age of thirteen to
legally so." Then, too, the medical findings showing sixteen, courts are inclined to give credence to their
23 | P a g e
account of what transpired, considering not only their
relative vulnerability but also the shame and Conspiracy is defined as an agreement between
embarrassment to which they would be exposed if the two or more persons to commit a crime and decide to
matter to which they testified is not true. commit it.  Conspiracy presupposes capacity of the
parties to such conspiracy to discern what is right from
Discernment is the mental capacity to understand what is wrong.  Since the prosecution failed to prove
the difference between right and wrong. that the petitioner acted with discernment, it cannot
thereby be concluded that he conspired with his co-
The accused, with methodical fashion, dragged the accused.
resisting victim behind the pile of hollow blocks near
the vacant house to insure that passers-by would not
be able to discover his dastardly acts. Declarador v. Judge Gubaton (G.R. No. 159208)

Jose v. People (G.R. No. 162052) Facts:

Facts: The accused, a 17-year old, minor, stabbed a


teacher, wife of the complainant, 15 times; he was
Accused was arrested in a drug buy-bust operation charged with murder aggravated by evident
conducted by the police; accused was a passenger in premeditation and abuse of superior strength, to which
the car of Zarraga, whom allegedly made the deal with the accused plead guilty to the crime charged.
the undercover in the said operation. They claimed
that they were kidnapped by the police and asked In view of the accused’s plea, RTC rendered
ransom for their release from one of the accused’s judgment finding the accused guilty, but the sentence
wife. was suspended by the judge automatically because
the accused was a minor at that time; he was ordered
The trial court found them guilty, and credited in to be committed to the Regional Rehabilitation for
their favour the preventive imprisonment they had Youth.
undergone.CA reduced the penalty on petitioner since
he was 13 years old at the time of the commission of The husband of the victim, herein petitioner,
the offense. claimed that in Art. 192 of P.D. No. 603, the benefit of
a suspended sentence does not apply to a juvenile
Issue: who was convicted of a crime punishable by death,
reclusion perpetua or life imprisonment.
W/N petitioner acted with discernment and that
prosecution failed to allege in the information that he Issues:
acted with discernment?
Whether or not respondent Judge committed
Decision: NO grave abuse of discretion amounting to lack of or
excess in jurisdiction in ordering the suspension of the
For a minor at such an age to be criminally liable, sentence of the accused?
the prosecution is burdened to prove beyond
reasonable doubt, by direct or circumstantial evidence, Decision: YES
that he acted with discernment, meaning that he knew
what he was doing and that it was wrong. The benefits of P.D. No. 603 shall not apply to
a youthful offender who has once enjoyed suspension
Such circumstantial evidence may include the of sentence under its provisions or to one who is
utterances of the minor; his overt acts before, during convicted for an offense punishable by death or life
and after the commission of the crime relative thereto; imprisonment or to one who is convicted for an offense
the nature of the weapon used in the commission of by the Military Tribunals.
the crime; his attempt to silence a witness; his disposal
of evidence or his hiding the corpus delicti. Furthermore, it shall not apply to a juvenile in
conflict with law who has once enjoyed suspension of
 The only evidence of the prosecution against the sentence or, when at the time of the promulgation of
petitioner is that he was in a car with his cousin, co- judgment, the juvenile is already 18 yrs. old.
accused, when the latter inquired from the poseur-
buyer, if he could afford to buy shabu. The disqualification is based on the nature of
the crime charged and the imposable penalty therefor,
There is no evidence that the petitioner knew what and not on the penalty imposed by the court after trial.
was inside the plastic and soft white paper before and It is not the actual penalty imposed but the possible
at the time he handed over the same to his cousin.  one which determines the disqualification of a juvenile.
Indeed, the poseur-buyer did not bother to ask the
petitioner his age because he knew that pushers used Rep. Act No. 9344 only amended the
young boys in their transactions for illegal drugs. disqualification of those juveniles in conflict with law,

24 | P a g e
who at the time of the promulgation of judgment, was
already 18 years old, and allowed the benefits to apply
to them. The other disqualification in P.D. No. 603 People v. Concepcion (386 SCRA 74)
remains unchanged. Hence, the accused is still
disqualified under law to benefit from such suspension
of sentence. Facts:

Case law has it that statutes in pari materia The accused Concepcion is a police officer
should be read and construed together because charged with murder for the killing of one Lorenzo
enactments of the same legislature on the same Galang. According to testimonies of both parties’
subject are supposed to form part of one uniform witnesses, Lorenzo Galang was brought to the
system; later statutes are supplementary or barangay hall because he was so drunk and unruly at
complimentary to the earlier enactments and in the the town plaza and was continually disturbing the
passage of its acts the legislature is supposed to have peace.
in mind the existing legislations on the subject and to
have enacted the new act with reference thereto. The accused then came to the barangay hall
apparently to question Galang. But herein lies the
differences in the testimonies. The prosecution
Accident witnesses (2 of them) testified that while interrogating
Galang, Concepcion suddenly fired two shots past the
Toledo v. People (439 SCRA 94) ear of the victim without injuring him. But later on, he
hit the victim in the abdomen and fired a shot which
wounded Galang in the thigh and then Concepcion
Facts: fired three more shots which hit the victim in the chest
and killed him.
The accused Toledo was charged with
homicide for the killing of one Ricky Duarte. Toledo But according to the accused, he was merely
insisted that when he killed the victim, the same was pacifying Galang when the victim became so unruly
purely accidental. He claimed that the victim was so that the accused fired two warning shots. But instead
drunk that the same charged at the door of his house. of scaring Galang, the latter tried to grab the gun from
This prompted the accused to get his bolo and when the accused. Two shots were accidentally fired which
he tried to prevent Ricky from entering, he accidentally hit Galang thus causing his death.
hit the latter whereby killing him.
The accused claims that he should be
But still the RTC and the Ca found him guilty. exempted because he was just performing his lawful
duty as a police officer and that the shooting was
And so, the accused goes to the SC wherein purely accidental.
this time, he claims that his actions were purely on
self-defense. It was done when the victim attacked him The trial court found Concepcion guilty.
and in trying to defend himself, he accidentally killed
Duarte. Issue:

Issue: Should the accused be exempted from


criminal liability due to accident?
Should the Court find his actions exempting
and/or justifying? Decision:

Decision: Well settled is the rule in criminal cases, that


the prosecution has theburden of proof to establish the
The Sc ruled that there is no such thing as guilt of the accused. However, once the defendant
accidental self-defense. The accused cannot claim the admits the commission of the offense charged, but
death purely accidental and when the findings of the raises an exempting circumstance as a defense, the
lower courts were unfavorable, later on change his burden of proof is shifted to him.  By invoking mere
defense by alleging that what happened was purely accident as a defense, appellant now has the burden
self-defense. of proving that he is entitled to that exempting
circumstance under Article 12 (4) of the Code.
The two defenses perpetuated by the accused
are totally inconsistent with each other. Although in the Unfortunately for the accused, his testimony
justifying circumstance of self-defense, an accused is was too full of inconsistencies which failed to
excused because of DELIBERATELY trying to repel an discharge the burden . For one, Concepcion claims
unlawful aggression which could have killed or injure that when the victim tried to grab his gun, said rifle was
him. And so, such acts are not in tune with ACCIDENT hanging on his shoulder on a swivel. But then he
which presupposes an act which was not even claimed that Galang tried to rest the rifle away by
contemplated or planned but purely accidental. grabbing the BARREL OF THE GUN”. This was very

25 | P a g e
inconceivable. Furthermore, it was not believable that Uncontrollable fear - For this exempting
a person so drunk would try to take away a rifle from a circumstance to be invoked successfully, the following
police officer who also had a handgun tucked by his requisites must concur: (1) existence of an
waist. uncontrollable fear; (2) the fear must be real and
imminent; and (3) the fear of an injury is greater than
Lastly, the prosecution witness categorically or at least equal to that committed.
testified that he saw Concepcion shoot the victim with
the M-16 rifle. In the instant case, the evil sought to be
avoided is merely expected or anticipated.  If the evil
And so, the finding of guilt by the lower court sought to be avoided is merely expected or anticipated
was proper. or may happen in the future, this defense is not
applicable

Irresistible Force/Uncontrollable Fear It must appear that the threat that caused the
uncontrollable fear is of such gravity and imminence
Ty v. People (G.R. No. 149275) that the ordinary man would have succumbed to it. It
should be based on a real, imminent or reasonable
fear for one’s life or limb. A mere threat of a future
Facts: injury is not enough.  It should not be speculative,
fanciful, or remote. A person invoking uncontrollable
This case stemmed from the filing of fear must show therefore that the compulsion was
7 Informations for violation of B.P. 22 against Ty such that it reduced him to a mere instrument acting
before the RTC of Manila. The said accused drew and not only without will but against his will as well. It must
issue to Manila Doctors’ Hospital to apply on account be of such character as to leave no opportunity to the
or for value to Editha L. Vecino several post-dated accused for escape.
checks. The said accused well knowing that at the time
of issue she did not have sufficient funds in or credit The fear harbored by Ty was not real and
with the drawee bank for payment of such checks in imminent.  Ty claims that she was compelled to issue
full upon its presentment, which check when presented the checks, a condition the hospital allegedly
for payment within ninety (90) days from the date demanded of her before her mother could be
hereof, was subsequently dishonored by the drawee discharged, for fear that her mother’s health might
bank for “Account Closed” and despite receipt of notice deteriorate further due to the inhumane treatment of
of such dishonor, said accused failed to pay said the hospital or worse, her mother might commit
Manila Doctors Hospital the amount of the checks or to suicide.  This is speculative fear; it is not the
make arrangement for full payment of the same within uncontrollable fear contemplated by law.
five (5) banking days after receiving said notice.  
Entrapment v. Instigation
Ty claimed that she issued the checks
because of “an uncontrollable fear of a greater People v. Sta. Maria (G.R. No. 171019)
injury.”  She claims that she was forced to issue the
checks to obtain release of her mother whom the
hospital inhumanely and harshly treated, and would Facts:
not discharge unless the hospital bills are paid.
On November 27, 2002, at around 10:00
The trial court rendered judgment against Ty. o’clock in the morning, P/Chief Insp. Noli Pacheco,
Ty interposed an appeal with the CA and reiterated her Chief of the Provincial Drug Enforcement Group of the
defense that she issued the checks “under the impulse Bulacan Provincial Office based at Camp Alejo Santos,
of an uncontrollable fear of a greater injury or in Malolos, Bulacan received an intelligence report about
avoidance of a greater evil or injury.”   The appellate the illegal drug activities in Sitio Gulod, Barangay
court affirmed the judgment of the trial court with Pantubig, San Rafael, Bulacan of a certain "Fael," who
modification.  It set aside the penalty of imprisonment later turned out to be appellant Rafael Sta. Maria.
and instead sentenced Ty to pay a fine of sixty P/Chief Insp. Pacheco formed a surveillance team to
thousand pesos P 60,000.00 equivalent to double the look for a police asset to negotiate a drug deal with
amount of the check, in each case. appellant. In the morning of November 29, 2002, the
surveillance team reported to P/Chief Insp. Pacheco
Issue: that a confidential asset found by the team had already
negotiated a drug deal for the purchase of P200 worth
Whether or not the defense of uncontrollable fear is of shabu from appellant at the latter’s house at No. 123
tenable to warrant her exemption from criminal Sitio Gulod, Barangay Pantubig, San Rafael, Bulacan
liability?   between 7:00 and 7:30 in the evening of November 29,
2002. The surveillance team then prepared for a buy-
Decision:No. bust operation, with PO3 Enrique Rullan as team
leader, and PO1 Rhoel Ventura, who was provided
with two (2) marked P100-bills, as poseur-buyer. At the

26 | P a g e
appointed time and place, PO1 Ventura and the with the appellant. There was no showing that the
confidential informant proceeded to appellant’s house informant induced appellant to sell illegal drugs to him.
and knocked at the door. Appellant opened the door
and the confidential informant introduced to him PO1
Ventura as a prospective buyer. PO1 Ventura later People v. Pacis (G.R. No. 146309)
handed the two (2) marked P100-bills to appellant
who, in turn, gave him a plastic sachet of shabu.
Thereupon, PO1 Ventura sparked his cigarette lighter, Facts: On April 6, 1998, Atty. Yap supervising agent
which was the pre-arranged signal to the other of the Dangeroud Drugs Division of the NBI, received
members of the buy-bust team that the sale was an information that certain Roberto Pacis was offering
consummated. Appellant was arrested and the two to sell ½ kilo of shabu for the amount of P950 per gram
marked P100-bills recovered from him. Also arrested or a total of P475,000.00. The NBI Chief of Dangerous
on that occasion was one Zedric dela Cruz who was Drugs Division approved the buy-bust operation. Atty.
allegedly sniffing shabu inside appellant’s house and Yap and Sr. Agent Congzon, Jr. were assigned to
from whom drug paraphernalia were recovered. Upon handle the case. The two officer and an informant went
laboratory examination of the item bought from to the house of the appellant at 375 Caimito Ville,
appellant, the same yielded positive for Caimito Street, Valle Verde II, Pasig City. They
methylampetamine hydrochloride or shabu weighing negotiated the sale of ½ kilo of shabu. The total price
0.041 gram. was reduced to P450,000.00. It was agreed that the
payment and delivery of shabu would be made the
The accused was charged of violation of next day at same place.
Section 5, Article II of R.A. No. 9165, otherwise known
as the Comprehensive Dangerous Drugs Act of 2002. On April 17, 1998, NBI agents and the
The trial court found appellant guilty beyond informant went to appellant’s house. Appellant handed
reasonable doubt of the offense charged. The Court of to Atty. Yap a paper bag, the latter saw a transparent
Appeals promulgated the assailed decision denying plastic with white crystalline substance inside.
the appeal. Appellant asked for the payment. Atty. Yap introduced
Congzon to get the money from the car. When
Issue: Whether or not instigation was the act which Congzon returned, he gave the “boodle money” to
preceded Sta. Maria’s arrest? Atty. Yap who handed to the appellant. Upon receipt of
payment, the officers identified themselves as NBI
Decision: agents and arrested him.

In entrapment, the entrapper resorts to ways The trial gave full credence to the testimonies
and means to trap and capture a lawbreaker while of the prosecution witnesses. Hence, this appeal.
executing his criminal plan. In instigation, the instigator
practically induces the would-be-defendant into Issue:
committing the offense, and himself becomes a co-
principal. In entrapment, the means originates from the Whether or not the “buy-bust” operation that
mind of the criminal. The idea and the resolve to led to the appellant’s arrest was valid?
commit the crime come from him. In instigation, the
law enforcer conceives the commission of the crime Decision:
and suggests to the accused who adopts the idea and
carries it into execution. The legal effects of In entrapment, ways and means are resorted
entrapment do not exempt the criminal from liability. to for the purpose of trapping and capturing
Instigation does. lawbreakers in the execution of their criminal plan. In
instigation on the other hand, instigators practically
It is no defense to the perpetrator of a crime induce the would- be defendant into the commission of
that facilities for its commission were purposely placed the offense and become co- principals themselves. It
in his way, or that the criminal act was done at the has been held in numerous cases by this Court that
"decoy solicitation" of persons seeking to expose the entrapment is sanctioned by law as legitimate method
criminal, or that detectives feigning complicity in the of apprehending criminal elements engage in the sale
act were present and apparently assisting its and distribution of illegal drugs.
commission. Especially is this true in that class of
cases where the offense is one habitually committed, The records show that the operation that led to
and the solicitation merely furnishes evidence of a the arrest of the appellant was indeed an entrapment,
course of conduct. not instigation. Courts generally give full faith and
credit to officers of the law, for they are presumed to
The solicitation of drugs from appellant by the have performed their duties in the regular manner. In
informant utilized by the police merely furnishes entrapment cases, credence is given to the narration
evidence of the course of conduct. The police received of an incident by the prosecution witnesses who are
an intelligence report that appellant has been officers of the law.
habitually dealing in illegal drugs. They duly acted on it
by utilizing an informant to effect a drug transaction
27 | P a g e
Jurisprudence has firmly entrenched the The Sandiganbayan convicted San Mateo and
following as elements in the crime of illegal sale of Chang of violation of sec 3(b) of R.A. No. 3019,
prohibited drugs: (1) the accused sold and delivered a otherwise known as Anti- Graft and Corrupt Practices
prohibited drug to another, and (2) he knew that what Act. Hence, this appeal.
he had sold was a dangerous drug. The elements
were duly proven in the case herein. The record shows Issue:
that the appellant sold and delivered the shabu to NBI
agents posing as buyers. Whether or not there was a valid entrapment
operation?
Chang v. People (G.R. No. 165111)
Facts: Decision:

Chang was the Municipal Treasurer of Makati Petitioners were undisputedly public officers at
who was tasked to examine or investigate tax returns the time of the commission of the offense. The
of private corporations and companies operating within prosecution, not only established creditably how the
Makati and determine the sufficiency and insufficiency offense charged was committed. It is established just
of the income tax assessed on them and collect as creditably how petitioners conspired to commit the
payments, San Mateo was the Chief Operations, crime.
Business revenue Examination, Audit division, Makati
Treasurer’s office. There is entrapment when law officers employ
ruses and schemes to ensure the apprehension of the
The examiners found that Group Developers, criminal while in the actual commission of the crime.
Inc. (GDI) incurred a tax deficiency inclusive of penalty There is instigation when the accused is induced to
in the amount of P494,601.11. The assessment notice commit the crime. The difference in the nature of the
was received by Mario Magat, Chief Operating Officer two lies in the origin of the criminal intent. In
of GDI. Magat was later able to talk to San Mateo via entrapment, the mens reoriginates from the mind of
phone. On May 15, 1991, Magat and San Mateo met the criminal. The idea and the resolve to commit the
for lunch at the Makati Sports Club. Chang later joined crime comes from him. In instigation, the law officer
the two, the three agreed that if GDI could pay conceives the commission of the crime and suggests
P125,000 by the end of May 1991, the assessment to the accused who adopts the idea and carries it into
would be ‘resolved’. execution.

On June 6, 1991, Magat met again for lunch From the evidence of the prosecution, it was
with San Mateo and Chang at the Makati Sports Club. clearly established that the criminal intent originated
Magat tried to convince the two that GDI wanted to pay from the mind of the petitioner. Even before the June
the correct amount of tax to the municipality. He was 19, 1991 meeting took place, petitioners already made
advised by San Mateo and Chang, however, that GDI known to Magat that GDI only had two options to
had only two options: Pay the P494,601.11 to the prevent the closure of the company, either to pay the
municipality or P125,000 to them. assessed amount of P494,601.11 to the Municipality or
to pay the amount of P125,000 to them.
On June 12, 1991, Magat met with the NBI
Deputy Director Epimaco Velasco who advised him to
file a complaint with the NBI. Magat thus gave a sworn Art. 13: Mitigating Circumstances
statement. After several days, Magat contacted San
Mateo and asked him if their position was still the Incomplete Justifying or Exempting
same to which the latter said yes. Magat thereafter told Circumstances
San Mateo that he would deliver the P125,000 on June
19,1991 at the Makati Sports Club. People v. CA and Tangan (G.R. No. 103613)

On June 19, 1991, Magat informed the NBI


that payment was to be made that day around Facts:
lunchtime. The NBI formed a team to conduct an
entrapment. The genuine money as well as the boodle On December 1, 1984, Navy Captain Eladio C.
money and the envelope where the money was placed Tangan was driving alone on Roxas Boulevard
were then laced with fluorescent powder. heading south and Generoso Miranda was driving his
San Mateo arrived and joined Magat at his car in the same direction with his uncle, Manuel
table. Chang arrived and joined the two. Magat told Miranda. Generoso was moving ahead of Tangan.
Chang and San Mateo that GDI was ready to pay and Suddenly, firecrackers were thrown in Generoso's way,
asked them if they could give him the Certificate of causing him to swerve to the right and cut Tangan's
Examination showing that GDI has no more tax path. Tangan blew his horn several times. Generoso,
liability. Chang handed the Certificate while Magat slowed down to let Tangan pass. Tangan accelerated
gave the brown envelope. At that instant, the NBI and overtook Generoso, but when he got in front,
agents announced that they were being arrested. Tangan reduced speed. Generoso tried four or five
times to overtake on the right lane but Tangan kept

28 | P a g e
blocking his lane. When Tangan slowed down to make self-defense and the ordinary mitigating circumstances
a U-turn, Generoso passed him, pulled over and got of sufficient provocation on the part of the offended
out of the car with his uncle. Tangan also stopped his party and of passion and obfuscation were appreciated
car and got out. Generoso and Tangan then in his favor; Tangan was released from detention after
exchanged expletives. Then Tangan went to his car the promulgation of judgment and was allowed bail in
and got his .38 caliber handgun on the front seat. the homicide case.

According to the prosecution witnesses, Mary Tangan appealed to the Court of Appeals, which
Ann Borromeo, Rosalia Cruz and Manuel Miranda, the affirmed the judgment of the trial court but increased
accused pointed his gun at Generoso Miranda and the award of civil indemnity to P50,000.00. His
when Manuel Miranda tried to intervene, the accused subsequent motion for reconsideration and a motion to
pointed his gun at Manuel Miranda, and after that the cite the Solicitor General in contempt were denied by
accused pointed again the gun to Generoso Miranda, the Court of Appeals.
the accused shot Generoso Miranda at a distance of
about a meter. The shot hit the stomach of Generoso The Solicitor General, on behalf of the prosecution,
Miranda causing the latter to fall. Manuel Miranda alleging grave abuse of discretion, filed a petition for
grappled for the possession of the gun and during their certiorari under Rule 65, naming as respondents the
grappling, Rosalia Cruz intervened and took hold of Court of Appeals and Tangan, where it prayed that the
the gun and after Rosalia Cruz has taken hold of the appellate court's judgment be modified by convicting
gun, a man wearing a red T-shirt took the gun from accused-appellant of homicide without appreciating in
her. The man in T-shirt was chased by Manuel his favor any mitigating circumstance. 
Miranda who was able to get the gun where the man in
red T-shirt placed it. Issue:

On the other hand, the defense, particularly Whether or not Tangan acted in incomplete
the accused and his witness by the name of Nelson self-defense?
Pante claimed that after the gun was taken by the
accused from inside his car, the Mirandas started to Decision:
grapple for possession of the gun and during the
grappling, and while the two Mirandas were trying to Incomplete self-defense is not considered as a
wrest away the gun from the accused, they fell down at justifying act, but merely a mitigating circumstance;
the back of the car of the accused. The accused lost hence, the burden of proving the crime charged in the
the possession of the gun after falling at the back of information is not shifted to the accused. In order that it
his car and as soon as they hit the ground, the gun fell, may be successfully appreciated, however, it is
and it exploded hitting Generoso Miranda. necessary that a majority of the requirements of self-
defense be present, particularly the requisite of
Tangan ran away while Generoso lay on the ground unlawful aggression on the part of the victim. Unlawful
bloodied. Manuel looked for the gun and ran after aggression by itself or in combination with either of the
Tangan. Tangan found a policeman who allowed him other two requisite suffices to establish incomplete
to enter his patrol car. Manuel arrived and told the self-defense. Absent the unlawful aggression, there
policeman that Tangan had just shot his nephew. can never be self-defense, complete or
Manuel went back to where Generoso lay and there incomplete, because if there is nothing to prevent or
found two ladies, Mary Ann Borromeo and Rosalina repel, the other two requisites of defense will have no
Cruz, helping his nephew board a taxi. Manuel basis.
suggested that Generoso be brought to the hospital in
his car. He was rushed to the Philippine General The element of unlawful aggression in self-
Hospital but he expired on the way. defense must not come from the person defending
himself but from the victim.
Tangan was charged with the crime of murder with the
use of an unlicensed firearm. However, the information A mere threatening or intimidating attitude is
was amended to homicide with the use of a licensed not sufficient. The exchange of insulting words and
firearm, and he was separately charged with illegal invectives between Tangan and Generoso Miranda, no
possession of unlicensed firearm. Tangan entered a matter how objectionable, could not be considered as
plea of not guilty in the homicide case, but moved to unlawful aggression, except when coupled with
quash the information for illegal possession of physical assault. There being no lawful aggression on
unlicensed firearm on various grounds. The motion to the part of either antagonists, the claim of incomplete
quash was denied, whereupon he filed a petition for self-defense falls.
certiorari with this Court. On November 5, 1987, said
petition was dismissed and the joint trial of the two No Intention to Commit so Grave a Wrong
cases was ordered.
People v. Callet (G.R. No. 135701)
After trial, the lower court acquitted Tangan of illegal
possession of firearm, but convicted him of homicide. Facts:
The privileged mitigating circumstance of incomplete
29 | P a g e
Elbert S. Callet was charged and found guilty
of the crime of Murder in the death of Alfredo Senador. The trial court rendered a judgment convicting
Callet used a 9-inch hunting knife in stabbing the latter Dennis for the crime of Murder qualified by treachery
on the left shoulder near the base of the neck causing or evident premeditation and appreciating three
Senador’s death shortly thereafter. Callet appealed mitigating circumstances. His father Manuel was
his conviction claiming that the Regional Trial Court of acquitted. Not satisfied with the judgment, Dennis
Negros Oriental, Dumaguete City (Branch 30) gravely appealed his case.
erred in failing to consider the mitigating circumstance
of the fact that he had no intention to commit so grave Issue:
a wrong thereforehis liability should be mitigated.
Whether or not the mitigating circumstance of
Issue: having acted in the immediate vindication of a grave
offense is appreciated?
Whether or not the criminal liability of Callet be
mitigated in that he had no intention to commit so Decision:
grave a wrong?
The Supreme held that the mitigating
Decision: circumstance of having acted in the immediate
vindication of a grave offense was properly
The Supreme Court ruled in the negative. The appreciated. Dennis was humiliated, mauled and
lack of “intent” to commit a wrong so grave is an almost stabbed by the Anthony. Although the unlawful
internal state. It is weighed based on the weapon aggression had ceased when Dennis stabbed
used, the part of the body injured, the injury inflicted Anthony, it was nonetheless a grave offense for which
and the manner it is inflicted. The fact that the the Dennis may be given the benefit of a mitigating
accused used a 9-inch hunting knife in attacking the circumstance.
victim from behind, without giving him an opportunity to
defend himself, clearly shows that he intended to do However, the mitigating circumstance of
what he actually did, and he must be held responsible sufficient provocation cannot be considered apart from
therefore, without the benefit of this mitigating the circumstance of vindication of a grave offense.
circumstance. These two circumstances arose from one and the
same incident, i.e., the attack on the
Vindication of a Grave Offense appellant by Anthony, so that they should be
considered as only one mitigating circumstance.
People v. Torpio (G.R. No. 138984)
Passion or Obfuscation

Facts: People v. Lab-eo (G.R. No. 133438)

In the evening of October 11, 1997, Anthony


went to the house of Dennis and invited the latter for a Facts:
drinking spree. Afterwards both left the house of
Dennis and went to a nearby store and started drinking Segundina Cayno was engaged in the
with a companion named Porboy Perez. The three business of selling rummage goods. Early in the
proceeded to Shoreline. In a cottage, Anthony tried to morning of October 21, 1996 her son, Jerry Cayno
let Dennis drink gin and as the latter refused, Anthony went to the “dap-ayan” or barangay hall, in front of the
bathed Dennis with gin and mauled him several times. Tadian Public Market to display the goods for sale.
Dennis crawled beneath the table and Anthony tried to After displaying the goods, Segundina arrived and took
stab him with a 22 fan knife but did not hit him. Dennis over. Before noontime, while Nancy and Julie were
got up and ran towards their home. plucking the white hair strands of Segundina, appellant
Wilson Lab-eo arrived and approached his aunt,
Upon reaching home, he got a knife. Alarmed Segundina. Upon seeing him, Nancy went to a
by the action of Dennis, his mother shouted. Manuel, distance of about two meters while Julie was still near
his father, tried to scold his son and confiscate from Segundina. Appellant sat down in front of his aunt and
him the knife but failed to do so, resulting to Manuel’s uttered something to her in a very soft voice. Nancy
incurring a wound in his hand. He went back to the did not hear what he said because of her distance from
cottage. Upon seeing Dennis, Anthony ran towards the them while Julie could not make out the conversation
creek but Dennis blocked him and stabbed him. When because of the sound coming from a running motor
he was hit, Anthony ran but got entangled with fishing engine. What they only heard was Segundina’s
net and fell on his back. Dennis then mounted on him answer which was uttered in a loud angry voice “koma-
and continued stabbing him resulting to the latter’s an ka tay baka mahigh bloodac” (“you get out because
death. After stabbing, Dennis left and went to Camp I might suffer high blood”). They saw appellant leave.
Downes and slept there. The next morning, Dennis When appellant returned about 3 to 5 minutes after,
voluntarily surrendered himself to Boy Estrera, a well- Segundina was sitting on a low rattan stool. In front of
known police officer. her were Nancy and Julie, they did not notice

30 | P a g e
appellant’s return, especially Segundina who had her emerged from the banana plantation each brandishing
back to appellant. When Julie saw appellant approach a bolo. They immediately attacked Jose hacking him
Segundina from the back, Julie thought that he would several times. Jose fell to the ground and rolled but
just box his aunt because she did not see the knife, Marcelo and his son kept on hacking him. Marcelo,
which was wrapped in his blue jacket. Then appellant then, turned to Simon and Edgar and shouted “huwes
suddenly made a thrusting motion and he stabbed de kutsilyo”. Upon hearing the same, Simon and
Segundina on the left portion of her back. He then ran Edgar ran.
away leaving the knife at the victim’s back with the
jacket he had covered it with, hanging by the knife’s Upholding the prosecution evidence, the trial
handle. Appellant surrendered to the police court rendered its Judgment, finding Marcelo Bates
authorities. The appellant was indicted for murder. guilty beyond reasonable doubt of the crime of Murder.
The appellant does not deny stabbing Cayno.
However, he maintains that neither treachery nor Issue:
evident premeditation attended the commission of the
crime. The Trial Court found the appellant guilty of the Whether or not Marcelo could validly invoke
crime of murder and sentenced him to suffer the the mitigating circumstance of passion and
penalty of reclusion perpetua. obfuscation?

Issue: Whether or not the actuation of the accused Decision:


can be properly appreciated as passion or obfuscation
in his favour? Passion and obfuscation may not be properly
appreciated in favor of appellant. To be considered as
Decision: a mitigating circumstance, passion or obfuscation must
arise from lawful sentiments and not from a spirit of
For a person to be motivated by passion and lawlessness or revenge or from anger and resentment.
obfuscation, there must first exist an unlawful act that In the present case, clearly, Marcelo was infuriated
would naturally produce an impulse sufficient to upon seeing is brother, Carlito, shot by Jose.
overcome reason and self-control. There is passional However, a distinction must be made between the first
obfuscation when the crime is committed due to an time that Marcelo hacked Jose and the second time
uncontrollable burst of passion provoked by prior that the former hacked the latter. When Marcelo
unjust or improper acts, or due to a legitimate stimulus hacked Jose right after seeing the latter shoot at
so powerful as to overcome reason. In asking the Carlito, and if appellant refrained from doing anything
appellant to leave, the victim did not do anything else after that, he could have validly invoked the
unlawful. There is an absolute lack of proof that the mitigating circumstance of passion and obfuscation.
appellant was utterly humiliated by the victim’s But when, upon seeing his brother Carlito dead,
utterance. Nor was it shown that the victim made that Marcelo went back to Jose, who by then was already
remark in an insulting and repugnant manner. The prostrate on the ground and hardly moving, hacking
victim’s utterance was not the stimulus required by Jose again was a clear case of someone acting out of
jurisprudence to be so overwhelming as to overcome anger in the spirit of revenge.
reason and self-restraint.
People v. Malejana (G.R. No. 145002)

People v. Bates (G.R. No. 139907)


Facts:
Prosecution’s witness Andres Madrid
Facts: narrated that on July 28, 1990 at around 7:15 p.m.,
while he was seated in front of his jeep parked at the
Around 2:00 in the afternoon of November 28, side of the road at Marisfoque, Pilar, Sorsogon in the
1995, Edgar Fuentes, Simon Fuentes and Jose company of Janus Roces, Antonio Sy, Samuel
Boholst left Barangay Esperanza, Ormoc City to Andrade, Bernarda Sy, Jose Belmonte and Ernesto
deliver copra to a certain Fely Rodado at Barangay Francisco, he saw appellant at about 30 meters away
Green Valley, Ormoc City. After delivering copra heading towards their direction. Upon reaching their
around 5:00 in the afternoon, the three men headed group, appellant asked them where Roces was. When
back to Barangay Esperanza. While they were along a he noticed Roces who was sitting at a distance of 1
trail leading to the house of Carlito Bates, the latter meter beside Madrid, appellant brandished an armalite
suddenly emerged from the thick banana plantation rifle and fired a shot into the air. Then he pointed the
surrounding the trail, aiming his firearm at Jose barrel of the gun at Roces and fired five (5) times,
Boholst who was then walking ahead of his hitting Roces thrice. After the victim fell to the ground,
companions. Jose grabbed Carlito’s right hand and appellant left the scene of the incident and went ot his
elbow and tried to wrest possession of the firearm. house about 150 meters away. Madrid and his other
While the two were grappling for possession, the gun companions tried to assist Roces but discovered that
fired, hitting Carlito who immediately fell to the ground. the latter was already dead, presumably from the
At that instant, Marcelo Bates and his son Marcelo gunshot wounds that were inflicted upon him by
Bates, Jr., brother and nephew of Carlito, respectively, appellant.

31 | P a g e
The RTC rendered its Decision finding the
Issue: accused Honorato Beltran, Jr. guilty beyond
Whether or not the defense was able to reasonable doubt of the crime of murder. On appeal,
establish that accused was entitled to the mitigating the Court of Appeals affirmed the RTC’s Decision.
circumstance of passion and obfuscation? Hence, this petition.

Decision: Issue:
Passion and obfuscation similarly cannot be
appreciated in favor of the appellant. To be entitled to Whether or not the appellant Honorato Beltran,
this mitigating circumstance, the following elements Jr. is entitled to the mitigating circumstance of
must be present: 1) There should be an act both voluntary surrender?
unlawful and sufficient to produce such condition of
mind, 2) The act that produced the obfuscation was
not far removed from the commission of the crime by a
considerable length of time, during which the Decision:
perpetrator might recover his normal equanimity. The
bare assertion that the victim and appellant had an Appellant is not entitled to the mitigating
argument does not provide justifiable basis for circumstance of voluntary surrender. Article 13,
applying to him this mitigating circumstance. The paragraph (7) of the Revised Penal Code states that
cause that produced the passion and obfuscation has the offender’s criminal liability may be mitigated if he
not been established nor proven by clear and voluntarily surrendered to a person in authority or his
convincing evidence. The defense advance mere agents. Accordingly, the essential elements of
speculations and conjectures to gloss over the fact that voluntary surrender are: (1) that the offender had not
there is lack of proof of the cause. Courts are not been actually arrested or apprehended; (2) that the
permitted to render judgments upon guesses or surrender was voluntary and spontaneous; and (3) that
surmises. Suspicion, it has been said, cannot give the offender surrendered himself to a person in
probative force to testimony which in itself is authority or his agent.
insufficient
Appellant was already apprehended for the
hacking incident by the barangay officials of Lipa City
Voluntary Surrender just before he was turned over to the police by a
certain Tomas Dimacuha. Assuming that appellant
People v. Beltran (G.R. No. 168051) had indeed surrendered to the authorities, the same
was not made spontaneously. Immediately after the
hacking incident, appellant, instead of proceeding to
Facts: the barangay or police, went to his brother, Sherman
Beltran, in Bauan, Batangas, and the bext day, to his
On November 3, 1999, appellant was indicted sister in Lipa City. It took him three long days to
in an Information for Murder allegedly committed as surrender to the police authorities. Moreover, the flight
follows: That on or about October 25, 1999 at around of appellant and his acts of hiding until he was
10:00 o’clock in the evening at Velasquez Road, Brgy. apprehended by the barangay officials are
Sta. Rita, Batangas City, Philippines and within the circumstances highly inconsistent with the spontaneity
jurisdiction of this Honorable Court, the above-named that characterizes the mitigating circcumstance of
accused, while armed with a bolo, a deadly weapon, voluntary surrender.
with intent to kill and with the qualifiying circumstance
of treachery, did then and there, willfully, unlawfully Andrada v. People (GR No. 135222)
and feloniously attack, assault and hack with the said
bolo, suddenly and without warning one Norman
Conception y Habla while the latter was unarmed and Facts:
completely defenseless, thereby hitting him on the
different parts of his body, which directly caused the In an Information dated January 7, 1987, the
victim’s death. When arraigned on November 9, 1999, Office of the City Prosecutor of Baguio City charged
appellant pleaded “Not Guilty” to the charge therein. petitioner with Frustrated Murder committed as follows:
Thereafter, trial ensued. That on or about the 24 th day of September 1986, in
the City of Baguio, Philippines and within the
Appellant, on his defense admitted that he jurisdiction of this Honorable Court, the above-named
hacked Norman with a bolo but insisted that he did the accused with intent to kill, with evident premeditation
same in self-defense. Furthermore, appellant also and with treachery, did then and there willfully,
claimed that Norman is taller than him; that he was unlawfully, and feloniously attack, assault and hack
forced to kill Norman because the latter insulted him one Arsenio Ugerio on the head twice with a bolo
and his mother; and that he was on his way to Bauan thereby inflicting the latter: hacking wound, head,
City to surrender to police when he was apprehended resulting in (1) skull and scalp avulsion vertex; (2)
by the barangay officers in Lipa City. depressed comminuted skull fracture, right parieto
occipital with significant brain laceration; operation

32 | P a g e
done; craciectomy; vertex debridement; craniectomy; purpose, thereby hitting and inflicting upon the said
right parieto occipital; dural repair; debridement, thus Marlo Casiong with fatal wounds on the different parts
performing all the acts of execution which would of his body which caused his death shortly thereafter.
produce the crime of Murder as a consequence
thereof, but nevertheless, the felony was not Appellant surrendered to the police authorities
committed by reason of causes independent of the will on August 18, 1994 while his other co-accused remain
of the accused, that is, by the timely medical at-large. When arraigned on September 28, 1994,
attendance extended to Arsenio Ugerio which appellant, with the assistance of counsel, entered a
prevented his death. When arraigned on February 9, plea of not guilty to the crime charged. Thereafter, trial
1987, petitioner, with the assistance of counsel de ensued.
parte, pleaded “Not Guilty” to the crime charged.
Thereafter, trial ensued. The RTC rendered its Decision finding the
accused Ricky Quimzon guilty beyond reasonable
Petitioner interposed self-defense and invoked doubt of the crime of Murder. On appeal, the Court of
the mitigating circumstance of voluntary surrender. Appeals affirmed the RTC’s Decision. Hence, this
petition.
The RTC rendered its Decision finding the
accused Peter Andrada guilty beyond reasonable Issue:
doubt of the crime of Frustrated Murder. On appeal,
the Court of Appeals affirmed the RTC’s Decision. Whether or not the appellant Ricky Quimzon is
Hence, this petition. entitled to the mitigating circumstance of voluntary
surrender?
Issue:
Decision:
Whether or not the accused Peter Andrada is
entitled to the mitigating circumstance of voluntary It appears in the Commitment Order, dated
surrender? August 14, 1994, issued by the Municipal Trial Judge
of the MTC of Burauen, Leyte, that appellant
“voluntarily surrendered to SPO1 Josefino Agustin of
PNP Burauen, Leyte on August 18, 1994.” An
Decision: examination of the records reveals that it can not be
considered as a mitigating circumstance. For the
Evidence for the prosecution shows that mitigating circumstance of voluntary surreder to be
petitioner, after attacking the victim, ran away. He was appreciated, the accused must satisfactorily comply
apprehended by responding police officers in the with three requisites: (1) he has not been actually
waiting shed at the corner of Cambas Road and arrested; (2) he surrendered himself to a person in
Magsaysay Avenue. For voluntary surrender to be authority or the latter’s agent; and (3) the surrender is
appreciated, the surrender must be spontaneous, voluntary. There must be a showing of spontaneity
made in such a manner that it shows the interest of the and an intent to surrender unconditionally to the
accused to surrender unconditionally to the authorities, authorities, either because the accused acknowledges
either because he acknowledges his guilt or wishes to his guilt or wishes to spare them the trouble and
save them the trouble and expenses that would be expense concominant to his capture.
necessarily incurred in his search and capture. Here,
the surrender was not spontaneous. The surrender of appellant was far from being
spontaneous and unconditional. The warrant of arrest
People v. Quimzon (G.R. No. 133541) is date June 17, 1992 and all the accused, including
appellant, remained at-large, which prompted the
Executive Judge of the RTC of Palo, Leyte to achieve
Facts: the case. It took appellant two years before he finally
surrendered to the police. In between said period,
In an Information dated July 28, 1992, appellant, through counsel, filed a Motion to Fix Bail
appellant, Ricky Quimzon and three oher persons, Bond without surrendering his person to the jurisdiction
namely Salvacion Lascarom, Canoto Cabero and of the trial court. Records do not reveal that the
Edgardo Detona were charged with the crime of motion had been acted upon by the trial court. This
murder allegedly committed as follows: That on or act of appellant may be considered as a condition set
about the 7th day of March 1992, in the Municipality of by him before he surrenders to proper authorities, thus
Burauen, Province of Leyte, Philipines, and within the preventing his subsequent act of surrendering from
jurisdiction of this Honorable Court, the above-named being considered as a mitigating circumstance.
accused, conspiring, confederating and helping one
another with treachery and abuse of superior strength, Confession of Guilt
with intent to kill, did then and there willfully, unlawfully,
and feloniuosly attack, assault, strike, stab and wound People v. Montinola (G.R. Nos. 131856-57)
one Marlo Casiong with short bolos locally known as
“pisao” which accused provided themselves for the

33 | P a g e
Facts: retroactively the special aggravating circumstance of
use of unlicensed firearm under Section 1 of P.D. No.
On 18 November 1996, William Montinola, 1866, as amended by R.A. No. 8294, the imposable
armed with an unlicensed Cal .380 Pistol Llama penalty would be death. Conformably with our ruling in
deliberately, willfully and criminally with violence People v. Valdez, insofar as the new law would
against or intimidation of persons, with intent of gain, aggravate the crime of robbery with homicide and
take and carry away cash amount of P67,500.00 increase the penalty from reclusion perpetua to death,
belonging to Jose Eduardo Reteracion. Montinola shot it would not be given retroactive application, lest it
the victim on the neck, killing Reteracion. Two criminal would acquire the character of an ex post facto law.
cases were filed against Montinola and he was later on Hence, we shall not appreciate that special
sentenced to reclusion perpetua for robbery with aggravating circumstance. There being no modifying
homicide and death for illegal possession of firearm. circumstances, the lesser penalty of reclusion
perpetua shall be imposed upon accused-appellant
Issue: WILLIAM.”

Whether the use of an unlicensed firearm on


the killing perpetrated by reason or on occasion of the
robbery may be treated as a separate offense or as an
aggravating circumstance in the crime of robbery with
homicide?
People v. Dawaton (G.R. No. 146247)
Decision:
Facts:
Sec. 1 of P.D.1866 provides that if homicide or Edgar Dawaton was found guilty by the trial
murder is committed with the use of an unlicensed court of murder qualified by treachery and was
firearm, the penalty of death shall be imposed. Said sentenced to death. On 20 September 1998, Leonidas
Presidential Decree was however, amended by R.A. Lavares and several companions, including Dawaton
8294, while Montinola’s case was still pending. R.A. were drinking in the house of the accused’s uncle.
8294 provides that if homicide or murder is committed Already drunk, Leonidas Lavares decided to sleep
with the use of an unlicensed firearm, such use of an while the accused and his companions continued
unlicensed firearm shall be considered as an drinking. Dawaton awakened Lavares by stabbing him
aggravating circumstance. The Court held “In recent at the base of the neck. Dawaton continued stabbing
cases, we ruled that there could be no separate Lavares until the victim died. Dawaton then ran away
conviction for illegal possession of firearm if homicide to the house of his other relative, where he was later
or murder is committed with the use of an unlicensed on arrested by the police.
firearm; instead, such use shall be considered merely
as an aggravating circumstance in the homicide or Issue:
murder committed. Hence, insofar as the new law will Whether or not the penalty of death imposed
be advantageous to WILLIAM as it will spare him from by the trial court upon the accused was correct?
a separate conviction for illegal possession of firearm,
it shall be given retroactive effect.” Pursuant to the Decision:
third paragraph of Section 1 of P.D. No. 1866, as No. The Supreme Court held that the trial
amended by R.A. No. 8294, use of an unlicensed court erred in not considering the alternative
firearm is a special aggravating circumstance in the circumstance of intoxication in favor of the accused.
homicide or murder committed. “At any rate, even “Under Art. 15 of The Revised Penal Code, intoxication
assuming that the aggravating circumstances present of the offender shall be considered as a mitigating
in the commission of homicide or murder may be circumstance when the offender commits a felony in a
counted in the determination of the penalty for robbery state of intoxication, if the same is not habitual or
with homicide, we cannot appreciate in this case the subsequent to the plan to commit said felony.
special aggravating circumstance of use of an Otherwise, when habitual or intentional, it shall be
unlicensed firearm mentioned in the third paragraph of considered as an aggravating circumstance.The
Section 1 of P.D. No. 1866, as amended by R.A. No. allegation that the accused was drunk when he
8294. Such law was not yet enacted when the crime committed the crime was corroborated by the
was committed by WILLIAM; it cannot, therefore, be prosecution witnesses. The accused and his drinking
given retroactive effect for being unfavorable to him.” companions had consumed four (4) bottles of gin at
The Court further held “Under Article 294 of the the house of Esmeraldo Cortez, each one drinking at
Revised Penal Code, as amended by R.A. No. 7659, least a bottle. It was also attested that while the four
robbery with homicide is punishable by reclusion (4) shared another bottle of gin at the house of Amado
perpetua to death, which are both indivisible penalties. Dawaton, it was the accused who drank most of its
Article 63 of the same Code provides that in all cases contents.” The Court further stated that “Under Art. 63,
in which the law prescribes a penalty composed of two par. 3, of The Revised Penal Code, in all cases in
indivisible penalties, the greater penalty shall be which the law prescribes a penalty composed of two
applied when the commission of the deed is attended (2) indivisible penalties, such as in this case, when the
by one aggravating circumstance. If we would apply commission of the act is attended by a mitigating

34 | P a g e
circumstance and there is no aggravating reclusion temporal medium, as maximum, and to pay
circumstance, the lesser penalty shall be applied. the costs?
Since no aggravating circumstance attended the killing
but there existed the mitigating circumstance of Decision:
intoxication, the accused should be sentenced only to
the lesser penalty of reclusion perpetua.” No. The Supreme Court held that the trial
Maria Garalde court correctly found petitioner guilty of violation of
2008-0326 §2(c) of P. D. No. 533, otherwise known as the Anti-
Cattle Rustling Law of 1974. However, it erred in
imposing the penalty of 10 years and 1 day of prision
Similar and Analogous Circumstances mayor, as minimum, to 12 years, 5 months and 11
days of reclusion temporal medium, as maximum. The
Canta v. People (G.R. No. 140937) trial court apparently considered P. D. No. 533 as a
special law and applied §1 of the Indeterminate
Sentence Law, which provides that "if the offense is
Facts: punished by any other law, the court shall sentence
the accused to an indeterminate sentence, the
Narciso Gabriel owns a cow that was passed maximum term of which shall not exceed the maximum
on from one person to another and each person was fixed by said law and the minimum shall not be less
responsible for the care and custody of the said cow. than the minimum term prescribed by the same."
At the time the cow got lost, it was under the care and However, as held in People v. Macatanda,P. D. No.
custody of Gardenio Agapay. Agapay took the cow in 533 is not a special law. The penalty for its violation is
the mountain of Pilipogan, 40 meters away from his in terms of the classification and duration of penalties
hut, at around 5:00 in the afternoon. When he came prescribed in the Revised Penal Code, thus indicating
back to get the cow at past 9 in the evening, the cow that the intent of the lawmaker was to amend the
was gone. However, Aagapay saw footprints that led Revised Penal Code with respect to the offense of
to the house of Filomeno Vallejos. Vallejos told theft of large cattle. In fact, §10 of the law provides:
Agapay that Exuperancio Canta took the cow.
The provisions of Articles 309 and 310 of Act
Agapay and Maria were instructed by Narciso No. 3815, otherwise known as the Revised
to get the cow and on their way to Florenitno Canta’s Penal Code, as amended, pertinent provisions
house, they saw Exuperancio. The latter told them of the Revised Administrative Code, as
that if it was really Narciso who was the owner of the amended, all laws, decrees, orders, instructions,
cow, he should get it himself. Exuperancia rules and regulations which are inconsistent with
accompanied the two to his father’s house and both this Decree are hereby repealed or modified
recognized the cow but Florentino was not home. accordingly.
Exuperancio told Maria and Agapay that he would call
them the next day to talk about the matter with his There being one mitigating circumstance and
father. Exuperancio never called. The matter was no aggravating circumstance in the commission of the
reported to the police and Narciso and Exuperancio crime, the penalty to be imposed in this case should be
were called for investigation. Exuperancio admitted fixed in its minimum period. Applying the Indeterminate
taking the cow but claims that he was the real owner of Sentence Law, in relation to Art. 64 of the Revised
the cow and that it was lost on December 3, 1985. Penal Code, petitioner should be sentenced to an
However, Narciso presented a certificate of ownership indeterminate penalty, the minimum of which is within
issued on March 9, 1986, signed by the municipal the range of the penalty next lower in degree, i.
treasurer, in which the cow was described as two e.,prision correccional maximum to prision mayor
years old and female. Then, the petitioner also medium, and the maximum of which is prision mayor in
presented a Certificate of Ownership of Large Cattle its maximum period.
dated February 27, 1985 and a statement executed by
Franklin Telen, who was the janitor at the treasurer's
office of the municipality, that he executed the Art. 14: Aggravating Circumstances
certificate of ownership in favor of Exuperancio. The
trial court rendered its decision finding petitioner guilty Classes of Aggravating Circumstances
of the offense charged. Exuperancio filed a Motion for
reconsideration but was denied by the Court of People v. Evina (405 SCRA 152)
Appeals and affirmed the trial court's decision.

Issue: Facts:
Gerardo Gavina was serve sentence
Whether or not the lower courts were correct of Reclusion Pertpetua for raping certain Ms.
in sentencing Exuperancio to ten (10) years and one Maritess Catcharo. Based on the given facts,
(1) day of prision mayor, as minimum, to twelve (12) Gerardo took advantage of the time when the
years, five (5) months, and eleven (11) days of victim’s mother was not around. He would
likely forced Maritess to have carnal

35 | P a g e
knowledged against her will and even poked a court rendered a decision finding the petitioner guilty of
knife at her while doing the deed in the victim’s the crime of Homicide and Frustrated homicide but not
dwelling and threthened the victim to kill her guilty of violation of COMELEC RES. 2958.
family should she tell her parents what
happened. On November 13, 1991 when the Issue:
appellant arrived at the Catcharro residence
he proceeded inside the bedroom of Maritess, Whether or not violation of COMELEC RES.
the latter ran out of the bedroom and told her 2958 may be considered as Special aggravating
mother not to leave her because her Papa circumstances which will negate consideration of
Gerry might raped her again. Surprised by mitigating circumstances of voluntary surrender?
what he heard, the following day Maritess was
brought to Tacloban City Medical Center for a Decision:
check-up and found to have lacerations to the
victims genitalia. Contrary to the facts above, With the passage of Republic Act. No. 8294 on
appellant claimed that the night of the incident 6 June 1997, the use of an unlicensed firearm in
he was working as porter until 10 PM, thus it murder or homicide is now considered as a SPECIAL
cannot be said that he committed the crime aggravating circumstance and not a generic
accused of him. Based on the information aggravating circumstance.68 Republic Act No. 8294
submitted, aggravating circumstances of use applies to the instant case since it took effect before
of weapon and dwelling were not alleged. the commission of the crimes in 21 April 1998.
Therefore, the use of an unlicensed firearm by the
Issue: petitioner in the instant case should be designated and
Whether or not aggravating appreciated as a SPECIAL aggravating circumstance
circumstances proved during trial but was not and not merely a generic aggravating circumstance.
alleged in the information may be considered?

Decision:
The supreme court held in the
negative. Although the special aggravating Katrina Garcia
circumstance of the use of a weapon and the 2006-0127
aggravating circumstance of dwelling were
proven, these aggravating circumstances
cannot be considered in fixing the penalty People v. Mendoza (327 SCRA 695)
because they were not alleged in the
information as mandated by Rule 110,
Sections 8 and 9 of the Revised Rules of Facts:
Criminal Procedure.  Although the crimes
charged were committed before the effectivity Efren Mendoza was charged with the crime of
of the said rule, nevertheless, the same should murder for killing Anchito Nano. In this case Efren
be applied retroactively being favorable to the alleged that Anchito Nano arrived at their house and
appellant. upon arrival it started to destroy the house and that the
Katrina Garcia her wife was shouting for help. Efren immediately look
2006-0127 for something to protect his family but found a bolo.
He approached Anchito but the latter tried to hacked
him but he was able to hacked him first on the right
People v. Palaganas (501 SCRA 533) side of his neck resulting to the death of the victim.
Thereafter Mendoza went to Municipal Hall of Vinzon
and voluntarily surrendered to the police. He claimed
Facts: that it was self defense. The autopsy revealed that
location of the wounds found on the body of the victim
On January 16, 1998 brothers Servillano and came from the back of the victim’s body. The court
Michael Ferrer went to Tidbits Videoke bar singing and ruled rejecting appellant’s self defense. This court
drinking beer. On the same evening Jaime Palaganas finds that the accused was not in imminent danger of
and Ferdinand Palaganas and Virgilio Bautista arrived. death or great bodily harm, an attempt to defend
The two groups occupied separate tables. After the himself by means which appeared unreasonable by
Ferrer’s singing Jaime Palaganas started singing and using a long bolo is unjustifiable. Hence this appeal.
was joined by Tony Ferrer who sang loudly and in
mocking manner. This insulted Jaime and soon a fight Issue:
ensued between Ferrer’s and Palaganas. Ferdinand
ran towards his house and sought help from his Whether or not voluntary surrender was offset
brother Fuijeric, the latter went outside however he by the aggravating circumstances of treachery?
was stoned by the Ferrer brothers. As they were
continuously stoned the appellant Ferdinand suddenly Decision:
pulled the trigger with the gun in his hands. The trial

36 | P a g e
The Supreme Court held in the negative. A minimum to reclusion temporal in its medium period as
qualifying circumstance changes the nature of the maximum.
crime. A generic aggravating circumstance, on the
other hand, does not affect the designation of the
crime; it merely provides for the imposition of the Kristine Gonzales
prescribed penalty in its maximum period. Thus, while 2008-0192
a generic aggravating circumstance may be offset by a
mitigating circumstance, a qualifying circumstance
may not. 32 People v. Tac-an (G.R. No. 76338-39)
Treachery in the present case is a qualifying, not a
generic aggravating circumstance. Its presence served
to characterize the killing as murder; it cannot at the Facts:
same time be considered as a generic aggravating
circumstance to warrant the imposition of the Renato Tac-anand Francis Escanowere close
maximum penalty. Thus, it cannot offset voluntary friends being classmates in high school and members
surrender. of the local Bronx gang. Francis withdrew from the
Kristine Gonzales gang on the advice of his mother who saw that Renato
2008-0192 carried a handgun on his visits to their home. Things
started turning sour between the two, and came to a
head on Dec 14, 1984. After an earlier altercation on
that day, Renato went home and got his gun. He
In Contempt or With Insult to the Public Authorities entered the Mathematics class under Mr. Damaso
Pasilbas in Rm15 and shouted for Francis. After
People v. De Mesa (G.R. No. 137036) locating the victim he fired at him but missed. He was
later able to hit him in the head as he was running to
the door with his classmates to escape. After this,
Facts: Renato paced outside in the hallway. A teacher
unknowing that Renato was the culprit, asked him for
Barangay Chairman Patricio Motas of Sta. help unwittingly informing him that Francis was still
Cruz Putol, San Pablo City was pronounced dead on alive. Renato immediately re-entered the room and
arrival on October 15, 1996 at San Pablo City District saying "So, he is still alive. Where is his chest?"
Hospital. The autopsy report showed that the cause of Standing over Francis sprawled face down on the
death was shock and hemorrhage due to gunshot classroom floor, Renato aimed at the chest of Francis
wounds at the back of the victim. and fired once more. The bullet entered Francis' back
below the right shoulder, and exited on his front chest
Hernando De Mesa was found guilty beyond just above the right nipple.
reasonable doubt for the crime of murder by the
Regional Trial Court of San Pablo City. He was Tac-an was charged with illegal possession of
sentenced to suffer the penalty of Reclusion Perpetua, firearms under P.D. No. 1866. An amended
pay the costs and to indemnify the heirs of the victim. information for murder was subsequently filed
Treachery, nighttime, in contempt of or with assault to aggravated by the use of illegal possession of
public authorities, were appreciated by the trial court firearms.
as aggravating circumstances attending the case
thereby qualifying the crime committed to murder. Issues:

Whether or not illegal possession of a firearm


Issue:
is a special aggravating circumstance in crimes of
homicide and murder?
Whether or not the trial court erred in
determining the nature of the crime committed and the
Decision:
corresponding penalty to be imposed?
No.Under an information charging homicide or
Decision:
murder, the use of an unlicensed firearm is not an
aggravating circumstance nor can it be used to
Yes. The prosecution failed to positively prove
increase the penalty for the second offense of
the presence of anyqualifying aggravating
homicide or murder to death or reclusion perpetua.
circumstance whereby the crime committed is only
The character of the instrument used in taking or
homicide for which the imposable penalty provided by
destroying human existence is not one of those
the Revised Penal Code is Reclusion Temporal.
included in the enumeration of aggravating
circumstances under Article 14 of the Revised Penal
Being the case, Indeterminate Sentence Law
Code.
may now be applied and absent any aggravating nor
mitigating circumstance, the penalty that may be
On the other hand, under an information for
imposed is prision mayor in its medium period as
unlawful possession of a firearm or ammunition, P.D.

37 | P a g e
1866 authorizes the increase of the imposable penalty From there, they went home. The 3 policemen,
for unlawful possession if the unlicensed firearm was Fortuna, Garcia, and Pablo, were charged with robbery
used to destroy human existence. Though it is not one and were found guilty of having conspired in
of the enumerated aggravating circumstances in committing the crime with intimidation of persons.
Article 14 of the Revised Penal Code, it may still be
considered to increase the penalty imposed because Issue:
of the explicit provision of the said special law.
Whether or not abuse of public position should
be taken as an aggravating circumstance by the mere
fact that the accused were police officers?

Decision:

The Supreme Courted held that the lower


courts failed to appreciate the aggravating
circumstance of “abuse of public position.”
Being police officers, it placed them in a position terrify
the Montecillos to surrender their money as bail. It
was on the account of their authority that convinced
the Montecillos that they had committed a crime and
that they would be taken to the police station. Had
they not been police officers, they would have not
convinced the Montecillos into giving them their
money.
Lourizza Genabe
2008-0154

People v. Villamor (G.R. Nos. 140407-08)

Facts:

On November 25, 1995, brothers Jerry Velez


and Jelord Velez were on their way home on board a
Lourizza Genabe motorcycle. A motorcycle was speeding behind them
2008-0154 and as they were about to cross the bridge, they heard
gun shots firing behind them. As they turned around,
Jerry identified PO3 Renato Villamor and Jessie
Abuse of Public Position Maghilom riding the motorcycle behind them. Shots
were fired at them and Jerry sustained wounds on the
Fortuna v. People (G.R. No. 135784) abdomen and elbow while Jelord died on the spot. The
trial proceeded against PO3 Villamor while Maghilom
was still at large. During trial, the Trial Court found the
Facts: PO3 Renato Villamor guilty of having commited
Murder aggravated by the circumstance of taking
On July 21, 1992, siblings Diosdada Montecillo advantage of his public position.
and Mario Montecillo were standing at the corner of
Mabini and Harrison Streets. A mobile patrol car Issue:
stopped in front of them and a policeman alighted.
The policeman frisked Mario and took Mario’s belt. He Whether or not the Trial Court properly applied
motioned Mario to enter the car. Mario obeyed and the aggravating circumstance of taking advantage of
was followed by Diosdada. While inside the car, the public position?
policemen told Mario that he would be brought to the
Bicutan police station where he would be interrogated, Decision:
mauled and heckled for carrying a deadly weapon.
They told the Montecillos that the bailbond for carrying The Supreme Court ruled that the aggravating
a deadly weapon was P12,000. The Montecillos were circumstance of “taking advantage of public position”
asked how much they had and then Diosdada was under paragraph 1 of Article 14 of the Revised Penal
asked to alight from the car. The driver followed her, Code was improperly applied.
took P1,500 from her wallet and instructed her to tell
the others that she only had P3,500. Inside the car, A public officer must use the influence that is
they were told to put all her money on the box. The vested in his office as a means to realize the purpose
Montecillos were told to get off at Harrison Plaza. of the crime to be appreciated as an aggravating

38 | P a g e
circumstance. The question “Did the accused abuse Nighttime, Uninhabited or Obvious Place or Band
his office to commit the crime” must be asked in order
to appreciate this circumstance as an aggravating People v. Villanueva (G. R. No. 135330)
circumstance.

No proof was shown that Villamor took Facts:


advantage of his position of being a policeman when
he shot Jelord Velez. Neither was his influence, For automatic review by the Supreme Court is
prestige or ascendancy used in killing Velez. Even the judgment of 12 May 1998, of the Regional Trial
without occupying a public position, the accused could Court, Branch 15, Cebu City, in Criminal Case No.
have committed the crime. CBU-46026-A, finding accused-appellant Camilo
Lourizza Genabe Villanueva (hereafter Camilo) guilty of the crime of
2008-0154 rape committed on the victim, Nia Gabuya (hereafter
Nia), and sentencing him to suffer the extreme penalty
of death and to pay Nia moral damages in the amount
People v. Magayac (G.R. No. 126043) of P50,000.

In an amended Information dated 16 January


Facts: 1998, Camilo was charged with rape as defined and
penalized in R.A. No. 8353. The accusatory portion of
On February 11, 1994, Jimmy Lumague, Tino the indictment states:
Magayac and Manuel Magayac, were preparing for
fishing along with other persons. Tino Magayac, That at midnight of 4 December 1997, in Cebu
pushed Jimmy for no reason. When Jimmy asked City, Philippines, and within the jurisdiction of this
why, Tino proceeded to hit Jimmy at the back. Hours Honorable Court, the accused, with deliberate intent,
later, Tino hit Jimmy at the stomach and Manuel with force and intimidation upon person, did then and
proceeded to hit Tino as well. The fight, however, was there willfully, feloniously and unlawfully have carnal
intervened. The next day, Jimmy and Manuel knowledge with Nia Gabuya, a minor of only 11 years
exchanged blows. They were, again, separated from of age and step-daughter of the said accused, against
each other. On February 12, Manuel, while carrying a the latter’s will. CONTRARY TO LAW.
long rifle, approached Jimmy. As the Jimmy was
trying to leave, he was shot by Manuel right on his Issues:
stomach. Jimmy fell on the ground and was shot at
the back several times. Afterwhich, Manuel Whether the private complainant Nia Gabuya
surrendered to the PC Mobile Force. An was really raped by the accused Camilo Villanueva on
information for Murder with the qualifying circumstance 4 December 1997 at around midnight?
of treachery, evident premeditation and taking
advantage of public position as a member of the Whether the testimony of the private
CAFGU. The Trial Court found him guilty of Murder complainant Nia Gabuya is not tainted with material
aggravated by cruelty and taking advantage of public inconsistencies and grievous falsity?
position, appreciated by the mitigating circumstance of
voluntary surrender. Whether the testimony of the private
complainant is enough to convict the accused for a
Issue: crime punishable by death?

Whether or not the circumstance of taking


advantage of public position should be appreciated as Decision:
an aggravating circumstance considering the facts of
the case? The issue of credibility raised in the three
assigned errors should be resolved against Camilo.
Decision:
Nia clearly testified that Camilo raped her. She
The Supreme Court held that in the recounted the details of her harrowing experience in a
commission of the offense, there was no aggravating credible, convincing and straightforward manner.
circumstance, specifically “abuse of public position”.
Considering the facts of the case, Manuel was The prosecution was able to establish with
a member of the CAFGU and the weapon used to moral certainty the fact of penetration, although
shoot Jimmy was a government issued M-14 rifle. incomplete. In order that the crime of rape may be said
These, however, do not necessarily prove that Manuel to be consummated, the successful penetration by the
took advantage of his public position as a member of rapist of the female’s genital organ is not
the CAFGU when the crime of murder was committed. indispensable. Penile invasion, it has often been held,
necessarily entails contact with the labia and even the
briefest of contacts under circumstances of force,
intimidation or unconsciousness, even without

39 | P a g e
laceration of the hymen, is deemed to be rape in our punished in the new Article 266-A of the Revised Penal
jurisprudence. It would, in fine, be enough in a Code.
conviction for rape that there is an entrance of the
male organ within the labia of the pudendum of the People v. Ancheta (G.R. No. 70222)
female organ. Neither the penetration of the penis
beyond the lips of the vagina nor the rupture of the
hymen is indispensable to justify conviction. Facts:

Consequently, the finding that Nia’s hymen is Juan Ancheta was charged in the Regional
intact does not disprove that rape was committed. Trial Court of Aparri, Cagayan, with the crime of
Even the fact that there was no reddening or robbery with arson, committed in conspiracy with two
hematoma in the external genitalia does not render the other persons who could not be tried with him because
occurrence of rape improbable. The doctrine is well they were then at large. He asks for a reversal of the
settled that the absence of external injuries does not decision convicting him of the crime of arson and
negate rape.Even Camilo’s claim that the sperm found sentencing him to the maximum penalty of reclusion
in the vagina of NIA was not his because he has perpetua plus civil indemnity in the sum of P40,000.00
undergone vasectomy, is inconsequential. The for the properties burned.
absence of spermatozoa is not an essential element of
rape. This is because in rape, the important On 25 August 1980, at about 11 o'clock in the
consideration is not the emission of semen but the evening, Ancheta and his two companions awakened
penetration of the female genitalia by the male organ. Teresa Gorospe, forced their entry into her house,
demanded the amount of P1,000.00, and burned her
On the issue of inconsistencies and house when the money was not delivered. Later, while
discrepancies, these things on minor matters neither the house was in flames, the Ancheta, brandishing a
impair the essential integrity of the prosecution’s bolo, prevented the people from approaching and
evidence as a whole nor reflect on the witness’ putting out the fire by warning them that he had thirty
honesty. Such inconsistencies, which may be caused companions.
by the natural fickleness of the memory, even tend to
strengthen rather than weaken the credibility of the Issues:
witness because they erase any suspicion of
rehearsed testimony. Whether or not there was conspiracy between
and among the accused?
Camilo has moral ascendancy over Nia, being
the common-law spouse of her mother and the man Whether or not the penalty imposed on
who acted as her father since she reached the age of Ancheta was proper?
reason. Nia’s tender age and Camilo’s custodial
control and domination over her had rendered her so Decision:
meek and subservient to his needs and desires, thus,
becoming an easy prey to Camilo’s lecherous On whether or not there was conspiracy
advances. Moreover, Camilo threatened her with a between and among the accused. The Supreme Court
knife. agreed that there was a conspiracy among the
accused-appellant and his two companions when they
For rape to exist it is not necessary that the forcibly entered the house of Teresa Gorospe and
force or intimidation employed be so great or of such burned it after their demand for P1,000.00.
character as could not be resisted. It is only necessary
that the force or intimidation be sufficient to A conspiracy exists when two or more persons
consummate the purpose which the accused had in come to an agreement concerning the commission of a
mind. Intimidation must be viewed in the light of the felony and decide to commit it, whether they act
victim’s perception and judgment at the time of the through the physical volition of one or all, proceeding
rape and not by any hard and fast rule. It is enough severally or collectively. It is settled that conspiracies
that it produces fear that if the victim does not yield to need not be established by direct evidence of acts
the bestial demands of the accused, something would charged, but may and generally must be proved by a
happen to her at the moment or thereafter, as when number of indefinite acts, conditions, and
she is threatened with death if she reports the incident. circumstances which vary according to the purpose to
It is this form of intimidation which explains why there be accomplished. The very existence of a conspiracy
are no traces of struggle which would indicate that the is generally a matter of inference deduced from certain
victim fought off her attacker. acts of the persons accused, done in pursuance of an
apparent criminal or unlawful purpose in common
Under the Anti-Rape Law of 1997, any between them.
physical overt act manifesting resistance against the
act of rape in any degree from the offended party, or The conspiracy having been established, it
where the offended party is so situated as to render should follow that the accused-appellant is as guilty as
her/him incapable of giving valid consent, may be his companions of the crime of arson, even if it be
accepted as evidence in the prosecution of the acts conceded that he was not the one who actually poured

40 | P a g e
the kerosene and ignited it to burn Teresa Gorospe's
house. When there is a conspiracy, the act of one is The Motion has merit.
the act of all and visits equal guilt upon every
conspirator. Baroy’s Birth Certificate -- the authenticity of
which was confirmed by the NSO -- outweighs the
On whether or not the penalty imposed on other evidence submitted to prove his date of birth. “A
Ancheta was proper. Under Article 321 of the Revised birth certificate is the best evidence of a person’s date
Penal Code, the penalty of reclusion temporal to of birth.”
reclusion perpetua shall be imposed "if the offender
shall set fire to any building, farmhouse, warehouse, The earlier evidence submitted by appellant
hut, shelter, or vessel in port, knowing it to be occupied during the trial did not conclusively prove his age.
at the time by one or more persons. .... " However, since the OSG did not object to the belated
appreciation of Annex “A” and left the matter to the
The aggravating circumstance of nighttime sound discretion of this Court, we resolve to rule in
was correctly appreciated because it was sought by favor of the accused.
the defendants to facilitate the commission of the
offense and their subsequent escape. Evident If the accused alleges minority and the
premeditation should also have been applied because prosecution does not disprove his claim by contrary
the offenders had deliberately plotted the crime, as evidence, such allegation can be accepted as a fact.”
early as 9 o'clock of the night in question, or two hours
before they actually burned the house. Based on his Birth Certificate, it is clear that
Baroy was only fourteen (14) years old when he
With these aggravating circumstances and no committed the crime of rape. Hence, a reconsideration
mitigating circumstances to offset them, the proper of the Court’s 9 May 2002 Decision is proper.
penalty as imposed by the trial court is reclusion
perpetua. The civil indemnity of P40,000.00 is allowed, Article 68 of the Revised Penal Code provides
but the costs of the suit shall be adjudged in toto that “when the offender is a minor x x x under fifteen
against the accused-appellant and not to be shared, as years x x x a discretionary penalty shall be imposed,
ordered by the trial court. but always lower by two degrees at least than that
prescribed by law for the crime which he committed.”
People v. Baroy (G.R. Nos. 137520-22) The penalty prescribed by law for the crime committed
by Baroy is reclusion perpetua to death. The penalty
two degrees lower is prision mayor.Additionally, Baroy
Facts: is entitled to the benefits granted by the Indeterminate
Sentence Law.
The Supreme Court, in its Decision
promulgated on 9 May 2002, affirmed the conviction of Recidivism
both appellants for three counts of rape with the use of
a deadly weapon. The penalty imposed upon them by People v. Dacillo (G.R. No. 149368)
the trial court was, however, reduced from death to
reclusion perpetua for each count of rape, because
aggravating circumstances had neither been alleged in Facts:
the Information nor sufficiently proven during the trial.
Appellant Dacillo together with Joselito Pacot
Appellant Alfredo Baroy has since then filed a were indicted for murder in an information and that the
Motion for a partial reconsideration of the Court’s commission of the foregoing offense was attended by
Decision. He claims that he is entitled to the privileged the aggravating circumstance of abuse of superior
mitigating circumstance of minority and, hence, to a strength.
penalty two degrees lower than reclusion perpetua. He
presented various pieces of conflicting documentary The case against appellant’s co-accused,
and testimonial evidence during the trial. He now prays Joselito Pacot, was provisionally dismissed for lack of
for the consideration and giving weight to his Birth sufficient evidence to identify him with
Certificate as the best evidence of his age. His Birth certainty.Appellant was arraigned on February 21,
Certificate shows that he was born on 19 January 2001 and, assisted by counsel, pleaded not guilty. Pre-
1984, while the crimes in question were committed on trial was conducted on March 1, 2001 and trial ensued
March 2, 1998. thereafter.

Issue: When the body was discovered in the evening


of February 11, 2000, appellant immediately left for
Whether or not Baroy's Certificate of Live Birth Cebu City, arriving there the next day, February 12,
sufficiently proves his minority when he committed the 2000. He stayed in Cebu City until his arrest the
crimes? following year.

Decision:

41 | P a g e
On May 31, 2001, the trial court rendered known as Meling, and by the latter’s common law
judgment finding appellant guilty of murder and husband, accused Elmedio Cajara also known as
imposed upon him the supreme penalty of death. The Elming. Upon being told by Meling that they would be
Court finds the accused Francisco Dacillo, guilty going to Sulod to get copra, Marita went with Meling
beyond reasonable doubt of the crime of murder for and Elming to the couple’s house in Sitio Catuhaan in
the death of Rosemarie Tallada, as defined and Barangay Serum. Since then until 30 May 1994 Marita
penalized under Art. 248 of the Revised Penal Code, stayed with Meling and Elming together with their two
as amended. Considering the aggravating (2) small children in a house consisting of only one
circumstance of recidivism with no mitigating room without any partition.
circumstance to offset the same, he is hereby
sentenced to the extreme penalty of death. In the evening of 30 May 1994 complaining
witness Marita Cajote slept at one end of the room with
Issue: the two (2) children, with Meling and Elming at the
other end. At about two o’clock the following morning
Whether or not it is necessary, in recidivism as Marita was awakened by the weight of accused who
an aggravating circumstance, to be alleged in the was already on top of her. The accused who was
information? holding a bolo told her to keep quiet or he would kill
her. He then placed his bolo aside and held Marita’s
Decision: hands with his right hand. With his left hand accused
lowered Marita’s pants as well as her panty down to
The Court, however, finds that the trial court her knees. Marita shouted for help but her sister
erred in imposing the death penalty on the ground that Meling just wrapped her head with their mosquito net
appellant admitted during re-cross examination that he and pretended to be asleep. Marita struggled
had a prior conviction for the death of his former live-in continuously against the advances of the accused but
partner. The fact that appellant was a recidivist was he was much stronger, while she was getting weak.
appreciated by the trial court as a generic aggravating The accused first inserted his fingers into Marita’s
circumstance which increased the imposable penalty private part and later succeeded in inserting his penis
from reclusion perpetua to death. into her vagina. Meling then pulled Elming away from
Marita and hit Elming in the eye. Elming boxed Meling
In order to appreciate recidivism as an on the mouth and kicked her when she fell on the floor.
aggravating circumstance, it is necessary to allege it in Elming went back to Marita and continued with his
the information and to attach certified true copies of beastly acts. By this time, Marita was already too weak
the sentences previously meted out to the to resist. Elming inserted his fingers first and then his
accused.This is in accord with Rule 110, Section 8 of penis into her private organ. The older of the two (2)
the Revised Rules of Criminal Procedure which states: children of Meling cried. Meling who was holding her
youngest child helplessly watched the accused rape
SEC. 8. Designation of the offense. - The her younger sister.
complaint or information shall state the
designation of the offense given by the statute, The trial court convicted him as charged and
aver the acts or omissions constituting the sentenced him to death. The Office of the Solicitor
offense, and specify its qualifying and General, in its brief, belittles the accused for failing to
aggravating circumstances. If there is no show any compelling or justifiable reason to set aside
designation of the offense, reference shall be his conviction for rape and his penalty of death, citing
made to the section or subsection of the Art. 335 of The Revised Penal Code, as amended by
statute punishing it. RA 7659.

The aggravating circumstance of recidivism Issue:


was not alleged in the information and therefore
cannot be appreciated against appellant. Hence the Whether or not the accused is guilty of
imposable penalty should be reduced to reclusion Qualified Rape.
perpetua.
Decision:

Reiteracion The Solicitor General is correct in finding the


accused guilty of rape. The bare denial of the accused
People v. Cajara (G.R. No. 122498)
and his common-law wife cannot overcome the
categorical testimony of the victim. Denial when
unsubstantiated by clear and convincing evidence is
Facts: negative and self-serving evidence which deserves no
greater evidentiary value than the testimony of a
On 18 May 1994 16-year old Marita Cajote, a credible witness on affirmative matters. No woman,
resident of Manila, arrived in Basey, Samar, and especially of tender age, would concoct a story of
stayed with her sister Marie. The following day, Marita defloration, allow an examination of her private parts
was fetched by another sister, Merly Tagana also and thereafter pervert herself by being subjected to a
42 | P a g e
public trial if she was not motivated solely by the desire Facts:
to have the culprit apprehended and punished. The
Court likewise agrees with the finding of the trial court On January 2, 2001, Edna, one hired as a
that Marita’s positive identification of the accused as housemaid by Roberto Separa Sr. was accused of
the person who raped her was given in a categorical, setting fire the house of his employer resulted in the
straightforward and spontaneous manner which destruction of his employer’s house and the death of
rendered it worthy of faith and belief. six persons including his employer Roberto Separa
Sr., some seven adjoining residential houses, were
Contrary to the ruling of the trial court and the also razed by fire. 
stand of the Solicitor General, the accused can only be
convicted of simple rape punishable by reclusion She was apprehended by the Barangay
perpetua. It was error for the trial court to impose the Chairman and was brought to the Barangay Hall. She
penalty of death. Although the circumstance of was then identified by a neighbor, whose house was
relationship by affinity within the third civil degree was also burned, as the housemaid of the Separas and
alleged in the Information, evidence for the prosecution upon inspection, a disposable lighter was found inside
clearly showed the lack or absence of such accused-appellant’s bag. Thereafter, accused-
circumstance to qualify the rape because the accused appellant confessed to the Barangay Chairman.
and Merly Tagana, sister of the victim Marita Cajote,
were mere common-law husband and wife and were On January 9, 2001, an information was filed
not legally married at the time of the rape. The before the RTC of Manila, charging the accused-
accused and the victim cannot be said to be related by appellant with the crime of Arson with multiple
affinity within the third civil degree at the time of the homicide. The RTC as well as the Court of Appeals
commission of the crime.Neither can the accused be finds the accused guilty beyond reasonable doubt of
convicted of qualified rape on the basis of the the crime of Arson with multiple homicide.
circumstance that the rape was committed in full view
of the relatives of the victim within the third degree of Issue:
consanguinity because this qualifying circumstance
was not pleaded in the Information or in the Complaint Whether or not Edna Malngan was guilty of
against the accused. the crime of destructive arson or simple arson?

The records show that the crime was Decision:


aggravated by reiteracion under Art. 14, par. 10, of
The Revised Penal Code, the accused having been The crime committed by the accused-appellant
convicted of frustrated murder in 1975 and of is Simple Arson and not Arson with Multiple Homicide.
homicide, frustrated homicide, trespass to dwelling, The Supreme Court ruled that there is no complex
illegal possession of firearms and murder sometime in crime of Arson with Multiple Homicide. There are two
1989 where his sentences were later commuted to laws that govern the crime of arson where death
imprisonment for 23 years and a fine of P200,000.00. results therefrom – Article 320 of the Revised Penal
He was granted conditional pardon by the President of Code and Section 5 of Presidential Decree 1613,
the Philippines on 8 November 1991.Reiteracion or quoted hereunder, to wit:
habituality under Art. 14, par. 10, herein cited, is
present when the accused has been previously Revised Penal Code
punished for an offense to which the law attaches an Art. 320.
equal or greater penalty than that attached by law to Destructive Arson – xxxx If as
the second offense or for two or more offenses to a consequence of the
which it attaches a lighter penalty. As already commission of any of the acts
discussed, herein accused can be convicted only of penalized under this Article,
simple rape and the imposable penalty therefor is death results, the mandatory
reclusion perpetua. Where the law prescribes a single penalty of death shall be
indivisible penalty, it shall be applied regardless of the imposed.
mitigating or aggravating circumstances attendant to Presidential Decree
the crime, such as in the instant case. No. 1613
Sec. 5. Where
The Decision of the trial court convicting the Death Results from Arson – if
accused ELMEDIO CAJARA alias Elming of Qualified by reason of or on the
Rape is MODIFIED to the effect that he is convicted occasion of the arson death
instead only of Simple Rape and is sentenced to suffer results, the penalty of
the penalty of reclusion perpetua. reclusion perpetua to death
shall be imposed.

By Means of Inundation, fire, etc. Both laws provide only one penalty for the
commission of arson, whether considered destructive
People v. Malngan (G.R. No. 170470) or otherwise, where death results therefrom. The

43 | P a g e
reason is that arson is itself the end and death is After trial, the court a quo convicted appellants
simply the consequence. of the complex crime of Murder with Multiple
Attempted Murder for having conspiring, confederating
The case falls under simple arson since from a and mutually helping one another, with intent to kill and
reading of the body of the information it can be seen by means of treachery and with the use of an
that it states that “the accused, with intent to cause explosive.
damage, xxx deliberately set fire upon the two-storey
residential house, xxx that by reason and on the Issue:
occasion of the said fire, xxx which were the direct
cause of their death xxx.” It is clear that her intent was Whether or not the use of explosive qualifies the
merely to destroy her employer’s house through the crime to murder?
use of fire.
Whether or not appellants conspired to kill the
When fire is used with the intent to kill a victims?
particular person who may be in a house and that
objective is attained by burning the house, the crime is Decision:
murder only. When the Penal Code declares that killing
committed by means of fire is murder, it intends that Yes, the killing by means of explosives qualifies
fire should be purposely adopted as a means to that the crime to murder. The information alleges that both
end. There can be no murder without a design to take treachery and the “use of explosive attended the
life. In other words, if the main object of the offender is crime.
to kill by means of fire, the offense is murder. But if the
main objective is the burning of the building, the Since both circumstances can qualify the killing
resulting homicide may be absorbed by the crime of to murder under Article 248 of the Revised Penal
arson. The latter being the applicable one in this case. Code, the Supreme Court held that when the killing is
perpetrated with treachery and by means of
People v. Comadre (G.R. No. 153559) explosives, the latter shall be considered as a
qualifying circumstance.  Not only does
jurisprudencesupport this view but also, since the use
Facts: of explosives is the principal mode of attack, reason
dictates that this attendant circumstance should qualify
At around 7:00 o’clock in the evening of the offense instead of treachery which will then be
August 6, 1995, Robert Agbanlog, Jimmy Wabe, Gerry relegated merely as a generic aggravating
Bullanday, Rey Camat and Lorenzo Eugenio were circumstance.
having a drinking spree on the terrace of the house of
Robert’s father, Jaime Agbanlog. Jaime was seated on No, there was no conspiracy. The undisputed
the banister of the terrace listening to the conversation facts show that when Antonio Comadre was in the act
of the companions of his son. of throwing the hand grenade, George Comadre and
Danilo Lozano merely looked on without uttering a
As the drinking session went on, Robert and single word of encouragement or performed any act to
the others noticed appellants Antonio Comadre, assist him.
George Comadre and Danilo Lozano walking. The
three stopped in front of the house. While his A conspiracy must be established by positive
companions looked on, Antonio suddenly lobbed an and conclusive evidence. It must be shown to exist as
object which fell on the roof of the terrace. Appellants clearly and convincingly as the commission of the
immediately fled by scaling the fence of a nearby crime itself.  Mere presence of a person at the scene
school. of the crime does not make him a conspirator for
conspiracy transcends companionship.
The object, which turned out to be a hand
grenade, exploded ripping a hole in the roof of the The evidence shows that George Comadre and
house. Robber Agbanlog and his companions were hit Danilo Lozano did not have any participation in the
by shrapnel and slumped unconscious on the floor. commission of the crime and must therefore be set
They were all rushed to the hospital for medical free.  Their mere presence at the scene of the crime as
treatment. However, Robert Agbanlog died before well as their close relationship with Antonio are
reaching the hospital for wounds sustained which the insufficient to establish conspiracy considering that
grenade explosion inflicted. Robert’s companions they performed no positive act in furtherance of the
sustained shrapnel injuries. crime. There being no conspiracy, only Antonio
Comadre must answer for the crime.
The appellants were arrested the following day
but denied any participation in the incident, claimed Craft, Fraud or Disguise
they were elsewhere when the incident occurred and
that they had no animosity towards the victims People v. Labuguen (G.R. No. 127849)
whatsoever.

44 | P a g e
Facts: perpetua to death Applying Article 63 of the same
Code, the imposable penalty under the premises is
The deceased Bonifacio Angeles was death in view of the presence of the aggravating
engaged in buying cows and selling them to the public circumstances of craft and fraud and the absence of
market. One day, the accused Vivencio Labuguen any mitigating circumstance.
went to him and told him that he knows of three big
cows for sale and that the place where they are is Four members of the Court are steadfast in
near. Believing on such declaration, he took money their adherence to the separate opinion expressed in
from his cabinet at his house amounting to P40,000 People vs. Echegaray that Republic Act No. 7659 is
and then drove in his motorcycle with the accused to unconstitutional insofar as it prescribes the death
see the cows. On their way to see the cows, they have penalty. However, they bow to the majority opinion that
been seen together by several witnesses who later on the aforesaid law is constitutional and therefore, the
identified them in court as the victim and the accused penalty prescribe thereunder has to be imposed.
respectively. The accused according to the witness
was wearing a jacket and with a handkerchief tied on Abuse of Superior Strength
his forehead. One of the witnesses, a driver of a
minibus testified that while driving on his way to his People v. Amodio (G.R. No. 177356)
destination, he saw a man behind the ‘talahibs’ and he
noticed that he was wiping something from his head
and right face. It was the same man whom his Facts:
conductor identified as the one who stopped their bus
and rode on it. His conductor testified further that he On June 10, 2003 at about 3:00 a.m., Richard
noticed that the man’s jacket was soaked with blood Avila Roda, an Assistant Manager of Nognog Videoke
including his pants and that he did not talk when asked Restaurant in Quezon City, went out of the restaurant
where he was headed to and instead just gave his to invite customers. He noticed that three of the
fare. The conductor even noticed that there was a lot attackers, whom he later identified as accused-
of money on the breast side pocket of his jacket and appellants Amodia, Marino, and Lo-oc, were regular
that one bill was even falling. The man then alighted customers of their restaurant. He saw Lo-oc hold the
from the minibus after reaching his destination without shoulders of the victim while Marino and Amodia took
saying any word. Later that afternoon, a news broke turns in beating the victim.As a result of the beating,
out that a man’s body was found dead in the middle of the victim fell on the ground where Roda immediately
the ricefield. He was later on identified as Bonifacio approached the victim and saw blood oozing out of the
Angeles. Based on the strength of the testimony of the back of his head. One of the maulers was about to
witnesses, complaint and information were filed deliver another blow on the victim but Roda was able
against Vivencio and the Regional Trial Court found to stop him, thereafter the appellants then went inside
him guilty of the crime of Robbery with Homicide and the restaurant and drank one bottle of beer each.But,
sentenced with the penalty of death. The case was Roda did not immediately report the incident because
brought to the Supreme Court for automatic review. he was threatened by accused-appellants who were
still hanging around the area.

Later, in the early morning of the same day,


there were already some barangay tanods and police
officers investigating the incident.The victim, later
Issue: identified as Jaime Bartina, was then brought to the
Quezon City General Hospital and died at around 5
Whether or not the court has correctly o'clock in the afternoon of June 10, 2003.
appreciated the employment of generic aggravating
circumstance of fraud and craft in the commission of Upon the advice of a person from the La Loma
the crime even if not alleged in the information? Police Station, Roda went to Camp Karingal in Quezon
City to report what he had witnessed.The police then
Decision: filed an investigation report which became the basis for
the filing of an Information against accused-appellants.
Though not alleged in the Information, the
generic aggravating circumstances of fraud and craft However, accused-appellants pleaded not
were properly appreciated by the trial court. Craft guilty to the charge against them by denying
involves intellectual trickery and cunning on the part of involvement in the death of the victim and averred alibi
the offender. When there is a direct inducement by as their defense.
insidious words or machinations, fraud is present. By
saying that he would accompany the victim to see the The Regional Trial Court rendered a decision
cows which the latter intended to buy, appellant was finding accused-appellants guilty beyond reasonable
able to lure the victim to go with him. doubt of the crime of murder.
The case was appealed to the Court of
Under Article 294 of the Revised Penal Code, Appeals which in its decision affirmed the trial court’s
the penalty for Robbery with Homicide is reclusion decision.

45 | P a g e
However, when appellant presented, he
Issue: declared that the shooting was unintentional.

Whether or not the killing was qualified by the The court disbelieved appellant’s claim of
circumstance of abuse of superior strength? accidental shooting whereby convicting the appellant
based on the evidence of the parties.Furthermore, the
Decision: court concluded that abuse of superior strength
attended the commission of the crime.
No.The qualifying circumstance of abuse of .
superior strength had not been sufficiently proved. To Issue:
appreciate the attendant circumstance of abuse of
superior strength, what should be considered is Whether or notthe killing was attended by
whether the aggressors took advantage of their abuse of superior strength to qualify the crime as
combined strength in order to consummate the murder?
offense. Mere superiority in number is not enough to
constitute superior strength. There must be clear proof
that the assailants purposely used excessive force out Decision:
of proportion to the defense available to the person
attacked. Yes.The prosecution sufficiently proved the
qualifying circumstance of abuse of superior strength.
In this case, although the victim was Abuse of superiority is present whenever there is
unquestionably outnumbered, it was not shown that inequality of forces between the victim and the
accused-appellants deliberately applied their combined aggressor, assuming a situation of superiority of
strength to weaken the defense of the victim and strength notoriously advantageous for the aggressor
guarantee the execution of the crime. Notably, and selected or taken advantage of by him in the
accused-appellants took turns in boxing the victim. commission of the crime.
When the victim fell, the prosecution witness was able
to hold him, preventing accused-appellants from In the present case, the victim was a woman
further hurting him. Then accused-appellants simply with a smaller build. She was unarmed. Appellant was
turned away. To be sure, had accused-appellants a fifty-one-year-old male, in the prime of his life, and
really intended to use their superior strength to kill the armed with a deadly weapon. The killing indubitably
victim, they would have finished off the victim, and constitutes an instance of abuse of superior strength,
probably even the lone prosecution eyewitness. hence the offense is qualified to murder, and not
merely homicide.Thus, the qualifying circumstance of
abuse of superior strength, as alleged in the
People v. Jamon (413 SCRA 282) information, attended the fatal shooting of Victoria
Tacla.

Facts: People v. Calpito (416 SCRA 491)

Information was filed dated September 15,


1998 against appellant charging him with murder for Facts:
shooting Victoria Tacla at her left chest with the use of
a gun, thereby inflicting upon said victim serious and That on or about the 21st day of November,
mortal wounds which were the direct and immediate 1990, appellant Francisco Calpito armed with a deadly
cause of her untimely death. weapon, with intent to gain did, then and there willfully,
unlawfully and feloniously by means of violence and
Upon arraignment, appellant pleaded not intimidation on the person of Florentina Villas rob, take
guilty. Thereafter, trial on the merits ensued. and carry away a shoulder bag containing cash in the
amount of P15,000 and jewelries amounting to
The prosecution presented as eyewitness Pilar P30,000 belonging to Florentina Villas.
Evangelista Tacla, the appellant’s wife and the victim’s
own mother as well as Dr. Wilfredo E. Tierra, Medico The appellant attack and stab with the said
Legal Officer of the National Bureau of Investigation weapon Florentina Villas and Israel Montilla inflicting
(NBI), also testified. wounds on Florentina Villas which caused her death
and a wound on Israel Montilla which necessitated
On the witness stand, Pilar said that on July 9, medical attendance on him for a period of 5-7 days
1998, at around 9 o’clock in the morning, she had a and which incapacitated him from performing his usual
heated altercation with appellant.Appellant warned work for the same length of time.
Pilar that once they moved out, she could never set
foot in her daughter’s house again and he added in a However, appellant entered a plea of not guilty
threatening manner, with his eyes bulging that if Pilar and waived pre-trial. But on June 15, 1993, appellant
came back to Victoria’s house, he would beat her up. was re-arraigned and after being appraised of the

46 | P a g e
consequences of the nature of his offense, he changed inasmuch as the attack was preceded by a quarrel and
his plea to one of guilty. heated discussion.

The court a quofinding the charge of Robbery Issue:


with Homicide unsubstantiated by evidence, convicted
appellant of the crime of murder. Whether or not treachery must be appreciated
as an aggravating circumstance?
Appellant, thereafter, filed a Motion for
Reconsideration arguing that the trial court erred in Held:
convicting him of Murder instead of Homicide and in
failing to apply the mitigating circumstance of minority. Yes. There is treachery when the offender
commits any of the crimes against persons, employing
The court denied the motion and affirmed means, methods or forms in the execution thereof
appellant’s conviction for murder. which tend directly and especially to ensure its
execution, without risk to himself arising from any
Issue: defense which the offended party might make. For
treachery to be appreciated, the prosecution must
Whether or not the abuse of superior strength prove: a) that at the time of the attack, the victim was
qualified the killing to murder? not in a position to defend himself, and b) that the
Decision: offender consciously adopted the particular means,
method or form of attack employed by him.
Yes.A perusal of the facts of the case readily
reveals that abuse of superior strength attended the The essence of treachery is a deliberate and
crime.  In several cases, this Court has ruled that this sudden attack, affording the hapless, unarmed and
circumstance depends on the age, size and strength of unsuspecting victim no chance to resist or to escape.
the parties. It is considered whenever there is a While it is true that the victim herein may have been
notorious inequality of forces between the victim and warned of a possible danger to his person, since the
the aggressor, assessing a superiority of strength victim and his companion headed towards their
notoriously advantageous for the aggressor which the residence when they saw the group of accused-
latter selected or took advantage of in the commission appellants coming back for them after an earlier
of the crime. In a recent case, it was held that an quarrel just minutes before, in treachery, what is
attack made by a man with a deadly weapon upon an decisive is that the attack was executed in such a
unarmed and defenseless woman constitutes an manner as to make it impossible for the victim to
abuse of the aggressor’s superior strength. The retaliate.
circumstance must apply with more reason in the
present case, where the abuse of superior strength is In the case at bar, Mateo did not have any
evident from the notorious disparity between the chance of defending himself from the concerted
relative strength of the victim, a 74-year-old unarmed assault of his aggressors, even if he was forewarned
woman, and the assailant, a young man armed with a of the attack. Mateo was obviously overpowered and
knife. helpless when accused-appellants’ group numbering
around eight, ganged up and mauled him. More
importantly, Mateo could not have actually anticipated
Treachery the sudden landing of a large concrete stone on his
head. The stone was thus treacherously struck.
People v. Piedad (393 SCRA 488) Neither could the victim have been aware that Lito
came up beside him to stab his back as persons were
beating him from every direction. Lito’s act of stabbing
Facts: the victim with a knife, inflicting a 15-centimeter-deep
wound shows deliberate intent of using a particular
On the night of April 10, 1996, victim Mateo means of attack. Considering the location of the
Lactawan, and his friend Andrew were drinking beer injuries sustained by the victim and the absence of
when he got involved in a fist fight with other people defense wounds, Mateo clearly had no chance to
drinking alcohol in the nearby store. That was when defend himself. In view of the foregoing, treachery was
Luz, Mateo’s wife arrived in the scene of the crime and correctly appreciated by the trial court.
saw that a group of men were attacking his husband.
Among the other aggressors who continuedly boxed
Mateo who’s already lying on the ground, Luz saw People v. Piliin (515 SCRA 207)
Niel struck Mateo on the head with a stone, and Lito
stabbed Mateo on the back, thereby inflicting traumatic
head injuries and a stab wound which eventually led to Facts:
Mateo’s death. Niel Piedad claims that the attack on
the victim was made upon an impulse of the moment On 19 November 1997, Rodrigo arrived at the
and was not the product of deliberate intent; while Lito gate of his house aboard on an owner-type jeep he
Garcia contends that treachery cannot be appreciated was driving. His wife, Norma Zayenis (Norma), who

47 | P a g e
was inside the house at the time, went out to open the Amadeo, a witness declared that on the night
gate. When Rodrigo was about to park his jeep, a of July 23, 1997 he and the accused had a drinking
man, later identified as Piliin, suddenly approached spree in the latter’s house. Moments later, appellant
him, poked his gun, and fired at him, hitting the left and his live-in partner Virginia had a heated argument.
side of his neck. Rodrigo fell unconscious and the man Accused Ilo kicked her several times.Ilo rushed to the
quickly ran away. By reason of the gunshot wound, the kitchen, got hold of an old frying pan and struck
victim thereafter died. Piliin confesses killing Rodrigo Virginia with it. She fell on the floor. Amadeo tried to
and implicated Yu and Caballes as his co-perpetrators. placate his friend but was rebuffed anew. Ilo got hold
However, After trial, appellant was found guilty for of a stone used as tripod in cooking and smashed
murder. The two other accused, Yu and Caballes were Virginias head with it. Consequently Virginia died.
acquitted for insufficiency of evidence. Thereafter the trial court rendered a decision finding
the Ilo guilty of Murder. On appeal, Appellant argues
During the stage of appeal, Piliin argues that that the injuries inflicted by him on the victim were
the prosecution failed to establish the existence of spur-of- the- moment reflexes during a passionate
treachery. According to him, the witness failed to see lovers quarrel, spawned by jealousy. He avers that the
the inception of the attack because she was in the act prosecution failed to prove that the killing of Virginia by
of opening the gate for her husband when the latter the Ilo was the product of a preconceived plan. He
was shot. She lacked knowledge of the attending further contends that his actuations were triggered by
circumstances prior to the shooting incident. Hence, the provocation emanating from the victim herself.
the trial court’s finding of treachery becomes Hence, appellant contends that he is guilty only of
speculative. homicide and not of murder.

Issue: Issue:

Whether or not treachery must be appreciated Whether or not the trial court erred in finding
as an aggravating circumstance? that treachery attended the killing of the victim ?

Decision: Decision:

Yes. There is treachery when the offender NO.Treachery is not presumed. The
commits any of the crimes against persons, employing circumstances surrounding the murder must be proved
means, methods or forms in their execution, without as indubitably as the crime itself. To constitute
risk to himself arising from the defenses which the treachery, two conditions must be present, namely: (1)
offended party might make. To establish treachery, two the employment of means of execution that gives the
elements must concur: (1) that at the time of the person attacked no opportunity to defend or to
attack, the victim was not in a position to defend retaliate; and (2) the deliberate or conscious adoption
himself, and (2) that the offender consciously adopted of the means of execution. The Court held that
the particular means of attack employed. The essence treachery cannot be appreciated if the assailant did not
of treachery is the unexpected and sudden attack on make any preparation to kill the victim in such a
the victim which renders the latter unable and manner as to insure the killing or to make it impossible
unprepared to defend himself by reason of the or difficult for the victim to defend herself. The
suddenness and severity of the attack. Appellant’s wife prosecution must prove that the killing was
witnessed the incident from its inception up to its premeditated or that the assailant chose a method or
consummation. mode of attack directly and especially to facilitate and
insure the killing without danger to himself.  The
In this case, the victim was about to park his essence of treachery is that the attack is deliberate
car when appellant suddenly appeared and shot him and without warning done in a swift and unexpected
without any warning. The attack was so sudden that manner of execution affording the hapless, unarmed
the latter had no opportunity to repel it or defend and unsuspecting victim no chance to resist or escape.
himself. It can readily be inferred that the manner of There is no treachery where the attack is neither
the attack adopted by appellant manifested treachery. sudden nor preconceived and deliberately adopted but
Furthermore, as correctly observed by the Solicitor just triggered by the sudden infuriation on the part of
General, the weapon used and the nature of the injury the offender. To establish treachery, the evidence
inflicted, which pertained to the lone gunshot fatally must show that the offender made some preparation to
wounding appellant, established that appellant kill the victim in such a manner as to insure the
deliberately and consciously adopted the particular execution of the crime or to make it impossible or
mode of attack to ensure the commission of the difficult for the person attacked to defend himself. The
offense with impunity. mode of attack must be planned by the offender and
must not spring from the unexpected turn of events.
People v. Ilo (392 SCRA 326) There is no treachery when the killing results from a
verbal altercation between the victim and the assailant
such that the victim was forewarned of the impending
Facts: danger. 

48 | P a g e
The prosecution failed to discharge its burden. establish their identity and participation beyond
The prosecution failed to adduce evidence as to the reasonable doubt.
relative positions of appellant vis--vis the victim. Taking
into account the sequential continuity and rapidity of Issue:
the events resulting in the death of Virginia, it cannot
be gainsaid that appellant made preparations to kill Whether or not excising of penis amounts to
Virginia and adopted a mode of attack as to make it ignominy that can aggravate the offense charged?
impossible or difficult for her to defend herself.
Decision:
Ignominy
NO. For ignominy to be appreciated, it is
People v. Salazar (G.R. Nos. 148712-15) required that the offense be committed in a manner
that tends to make its effect more humiliating, thus
adding to the victim’s moral suffering. Where the victim
Facts: was already dead when his body or a part thereof was
dismembered, ignominy cannot be taken against the
On December 28, 1999, at 6:00PM, two armed accused.
men suddenly entered Barnachea residence in
Barangay Calumbaya, Bauang, La Union. The two In this case, the information states that
ordered a 12-year old boy, Jessie E. Barnachea, to Victorino’s sexual organ was severed after he was
drop the floor by hitting him in the back with the butt of shot and there is no allegation that it was done to add
a long gun. They hurriedly proceeded to the living ignominy to the natural effects of the act. We cannot,
room and shot Jessie’s uncle, Victorino Lolarga, and therefore, consider ignominy as an aggravating
continued shooting in the kitchen hitting his mother circumstance.
Carmelita Barnachea, his brother Felix Barnachea, Jr.,
and his cousin Rubenson Abance. SC sustained the conviction of Cachola and
Amay but the rest of the six appellants were acquitted
His eldest brother, Robert E. Barnachea, who for the crime charged for insufficiency of evidence.
then was in his uncle’s house, noticed a stainless jeep,
with blue rim and marking "fruits and vegetables People v. Bumidang (G.R. No. 130630)
dealer," and with the description of the "El Shaddai"
parked in front of the fence of their house. Also, the Facts:
jeep did not go unnoticed by the neighbors, Russel
Tamba and Francisco Andrada. On September 29, 1996, at 2:00AM, accused
Bumidang loudly called Melencio Imbat (father of the
The incident was immediately reported to the victim) to open the latter’s door. Melencio, an
police and at around 7:45 p.m., the jeep was octogenarian who was sleeping at that time, hurriedly
intercepted at a checkpoint set up in the highway by opened the door as the accused threatened to kill
the police force in Aringay, La Union. On board were them if the door was not opened. The accused entered
the eight appellants. No firearms were found in the and asked the old man to bring him upstairs where he
vehicle. The jeep and the eight appellants were and his 56-year old unmarried daughter Gloria was
thereafter brought to the Aringay police station and sleeping. When they were in the room, the accused
then turned over to the Bauang police. Jessie was able got a spear at the side of Melencio’s bed and ordered
to identify two of the eight appellants by the name of the latter to lie in a prone position as he headed the
Cachola and Amay as the two assailants who entered daughter’s bed. Gloria arose and screamed for help
the house. The next day a paraffin test was conducted but his old father was in no strength to help her and
on the appellants. remained in a prone position as told by the accused.

The Death Certificates attest to the gruesome The accused approached Gloria and poked the
and merciless killings. Carmelita sustained one spear at her. She recognized him because he was
gunshot wound on her head and three on her body; lighting the room with a flashlight. The accused
Felix, Jr., two gunshot wounds on his head and on his ordered her to stand up and removed her pajama, with
body, and stab wounds on his chest and arms; the panty going along with it. While the accused was
Victorino, two gunshot wounds on his head, three on removing her clothes, she sat and struggled. The
his body, and with his penis excised;Rubenson, one accused then removed his short pants and became
gunshot wound on his head and a stab wound that completely naked. He used the flashlight to examine
lacerated his liver. her genital. He placed the spear beside her and
whenever she attempted to move, he would point the
RTC convicted all the eight appellants but the spear at her. The accused then went on top of her,
Office of the Solicitor General (OSG) recommended inserted his penis into her pudenda, held her breasts
the affirmance of the conviction for murder of and kissed her until he became sated. The accused
appellants Cachola and Amay, and the acquittal of the threatened the father and daughter that he will kill
other appellants for failure of the prosecution to them if they will report it to the authorities. Then he
went to the door and left after satisfying his lust.

49 | P a g e
in the presence of Gloria's old father. These facts
Despite the threats, Gloria reported the incident to clearly show that Bumidang deliberately wanted to
a Kagawad who handed a note to be given to the further humiliate Gloria, thereby aggravating and
authorities (security). The accused was immediately compounding her moral sufferings. Ignominy was
arrested. appreciated in a case where a woman was raped in
the presence of her betrothed or of her husband or
On October 1, 1996, Gloria submitted herself to a was made to exhibit to the rapists her complete
vaginal examination of Dr. Quines, confirming a nakedness before they raped her.
laceration of the hymen at 6:00 o'clock but no
spermatozoa were obtained. The laceration was about SC sustained RTC’s decision finding Baliwang
3 to 5 days old at the time of the examination. guilty of rape with the use of a deadly weapon and
sentencing him to suffer the penalty of death.
On 8 October 1996, a complaint for rape was filed
before the MCTC of Villaverde-Quezon, Nueva People v. Siao (G.R. No. 126021)
Vizcaya, and found a prima facie case against
Bumidang. The records were forwarded to the Office of
the Provincial Prosecutor. Facts:

When it reached RTC, it rendered a decision On May 27, 1994, at about 3:00PM, accused-
finding the accused guilty of rape with the use of a appellant Rene Siao, in his residence, ordered Reylan
deadly weapon under Art. 335 of the Revised Penal Gimena, his family’s 17-year old houseboy, to pull
Code and was sentenced to death by lethal injection. Estrella Raymundo, their 14-year old housemaid, to
the women’s quarters. Once inside, appellant Siao
Issue: pushed her to the wooden bed and asked her to
choose one among a pistol, candle or a bottle of sprite.
Whether or not dwelling, nighttime and Appellant lit the candle and dropped the melting candle
ignominy shall be appreciated as aggravating on her chest. Estrella was made to lie down on her
circumstances of the crime of rape? back on the bed w/ her head hanging over one end.

Decision: Appellant then poured sprite into her nostrils


as she was made to spread her arms w/ his gun
YES. The trial court correctly appreciated the pointed to her face. Appellant Siao then tied her feet
aggravating circumstance of dwelling. There was a and hands w/ an electric cord or wire as she was made
clear violation of the sanctity of the victim's place of to lie face down on the bed. As Siao pointed his pistol
abode when Gloria, who apparently did not give any at her, he ordered Estrella to undress and commanded
provocation, was raped in her own house. Dwelling is her to take the initiative on Gimena. Not understanding
considered an aggravating circumstance primarily what he meant, appellant motioned to her poking the
because of the sanctity of privacy the law accords to gun at her temple. Gimena was then ordered to
human abode. remove his shorts. He did not do so but only let his
penis out. Appellant Siao spread the arms of Estrella
Nighttime is an aggravating circumstance when and made her lie down spread-eagled. She felt dizzy
(1) it is especially sought by the offender; (2) it is taken and shouted for help twice. Siao then ordered Gimena
advantage of by him; or (3) it facilitates the to rape Estrella. At first, Gimena refused because he
commission of the crime by ensuring the offender's has a sister. However, Siao said that if they would not
immunity from capture. obey, he would kill both of them.

In this case, other than the fact that the crime Both Gimena and Estrella were forced and
was committed at about 2:00 o'clock in the morning, intimidated at gunpoint by Siao to have carnal
nothing on the record suggests that Bumidang knowledge of each other. They performed the sexual
deliberately availed himself or took advantage of act because they were afraid they would be killed. Siao
nighttime nor proved that Bumidang used the darkness commanded Gimena to rape Estrella in 3 diff positions
to facilitate his evil design or to conceal his identity. (i.e. missionary position, side-by-side and dog position
as narrated vividly in the case), pointing the handgun
The aggravating circumstance of ignominy shall at them the whole time. Thereafter, Siao warned them,
be taken into account if means are employed or “If you will tell the police, I will kill your mothers.”
circumstances brought about which add ignominy to
the natural effects of the offense; or if the crime was Appellant Siao, for his defense, denies the
committed in a manner that tends to make its effects whole event. He asserts that she retaliated through
more humiliating to the victim, that is, add to her moral this accusation because Estrella herself was accused
suffering. of stealing many of his family’s personal effects.

In this case, it was established that Bumidang


used the flashlight and examined the genital of Gloria Issue:
before he ravished her. He committed his bestial deed
50 | P a g e
Whether or not ignominy, as an aggravating Rufino Mallari and Joseph Galang when the latter
circumstance of the crime of rape, is attendant to admonished the former not to drive fast while passing
justify the award of exemplary damages? by the latter’s house. To end the situation Joseph,
together with his brothers, who were also present at
Decision: that time, asked for apology from Rufino. However, the
conflict did not end there because when dusk came
YES. The RTC overlooked and did not take and while Joseph was watching basketball game with
into account the aggravating circumstance of ignominy his wife, Rufino arrived with some companions and
and sentenced accused-appellant to the single attacked Joseph with bladed weapons. They chased
indivisible penalty of reclusion perpetua. It has been him and when Joseph was able to run away, Rufino
held that where the accused in committing the rape pursued him with the use of the Isuzu Canter Elf truck.
used not only the missionary position i.e. male When he caught up with him, he bumped him which
superior, female inferior but also the dog position as resulted in his instant death. The doctor who
dogs do, i.e. entry from behind, as was proven in the conducted the medico-legal inspection of the cadaver
case, the aggravating circumstance of ignominy testified that Joseph’s cause of death was “crushing
attended the commission thereof. injury on the head secondary to vehicular accident”.
The trial court found Rufino liable with murder and
Still, SC respected RTC’s finding of facts and sentenced with the penalty of death after considering
found any inconsistencies in the witnesses’ the qualifying circumstance of use of motor vehicle in
testimonies inconsequential considering that they committing the crime. The case was brought to the
referred to trivial matters w/c have nothing to do w/ the Supreme Court pursuant to the requirement of
essential fact of the commission of rape that is carnal automatic review of cases penalized with death
knowledge through force and intimidation. Ergo, even penalty based on Article 47 of the Revised Penal
if it was pointed out that in all 3 positions, Gimena Code. Rufino argued that the use of a motor vehicle
ejaculated 3x in a span of less than 30 mins, w/c does was only incidental, considering that he resorted to it
not conform to common experience, rape was still only to enable him to go after Joseph after he failed to
present from the evidence because rape is not the catch up with the latter.
emission of semen but the penetration of the female
genitalia by the male organ. Penetration, however Issue:
slight, and not ejaculation, is what constitutes rape.
Moreover, even if the house was occupied by many Whether or not the qualifying circumstance of
people at the time of the crime, rape was still use of motor vehicle was correctly appreciated by the
committed because lust is no respecter of time and trial court in imposing the death penalty?
place. And Estrella’s and Gimena’s decision not to flee
proves only the fear and intimidation that they were
under because Siao was after all their “amo” or
employer who threatened to kill them or their family if Decision:
they did not succumb to his demands.
The evidence shows that Rufino deliberately
The governing law is Art 335 RPC as used his truck in pursuing Joseph. Upon catching up
amended by RA 7659 w/c imposes the penalty of with him, Rufino hit him with the truck, as a result of
reclusion perpetua to death, if committed w/ the use of which Joseph died instantly. It is therefore clear that
a deadly weapon. Siao is further ordered to pay the the truck was the means used by Rufino to perpetrate
offended party moral damages, w/c is automatically the killing of Joseph.
granted in rape cases w/o need of any proof, in the
amount of PhP50K. Furthermore, the presence of the The case of People v. Muñoz cited by Rufino
aggravating circumstance of ignominy justifies the finds no application to the present case. In the said
award of exemplary damages pursuant to Art 2230 case, the police patrol jeep was merely used by the
CC. Judgment affirmed w/ modification of damages accused therein in looking for the victim and in carrying
awarded. the body of the victim to the place where it was
dumped. The accused therein shot the victim, which
Aid of Minor or By Means of Motor Vehicles caused the latter’s death. In the present case, the truck
itself was used to kill the victim by running over him.
People v. Mallari (404 SCRA 170)
Under Article 248 of the Revised Penal Code,
a person who kills another “by means of a motor
Facts: vehicle” is guilty of murder. Thus, the use of motor
vehicle qualifies the killing to murder. The penalty for
Based on the accusatory portion of the murder is reclusion perpetua to death. Since the
Information filed against Rufino Mallari, he was penalty is composed of two indivisible penalties, we
accused of hitting and bumping one Joseph Galang shall apply Article 63(3) of the Revised Penal Code,
with an Isuzu Canter Elf truck on or about July 7, 1996. which reads:
The evidence for the prosecution showed that the said 3. When the commission of the act is attended
incident was preceded by an altercation between by some mitigating circumstances and there is
51 | P a g e
no aggravating circumstance, the lesser
penalty shall be applied.
Issue:
In the present case, the aggravating
circumstances of evident premeditation and treachery, Whether or not the aggravating circumstance
which were alleged in the information, were not of use of motor vehicle should be considered in this
proved. What was proved was the mitigating case?
circumstance of voluntary surrender through the
testimonies of Rufino and Myrna, which were not Decision:
rebutted by the prosecution.
The indictment against accused-appellant is
In view of the absence of an aggravating murder attended by the use of motor vehicle. The use
circumstance and the presence of one mitigating of a motor vehicle qualifies the killing to murder if the
circumstance, reclusion perpetua, not death, should be same was perpetrated by means thereof. Appellant's
the penalty to be imposed on Rufino. claim that he merely used the motor vehicle, Kia Ceres
van, to stop the victim from escaping is belied by his
People v. Enguito (326 SCRA 508) actuations. By his own admission, he testified that
there was a police mobile patrol near the crossing.
Accused-appellant could have easily sought the
Facts: assistance of the police instead of taking the law into
his own hands. Moreover, accused-appellant already
On or about September 22, 1991, Felipe noticed the deceased trying to jump out of the
Requerme was driving a motorela, together with his motorela but he still continued his pursuit. He did not
wife Rosita and another passenger, Engr. Wilfredo stop the vehicle after hitting the deceased[16] who was
Achumbre, who is the deceased in this case. The hit when he (Achumbre) was at the railing of the
deceased was picked up by them on their way home Marcos bridge. Accused-appellant further used the
and requested them to bring him to his house. While vehicle in his attempt to escape. He was already more
on their way, a white vehicle, which was later on than one (1) kilometer away from the place of the
identified as a Ceres Kia automobile bearing Plate No. incident that he stopped his vehicle upon seeing the
722, intentionally hit and pushed the motorela that they police mobile patrol which was following him.
were riding and violently kept pushing it causing it to
turn around facing the direction from where it came Appellant contends that he should have been
from and fell on its right side. Rosita testified that convicted of the crime of homicide with two (2)
while she was struggling out of the motorela she mitigating circumstances of acting in passion and
noticed that the white vehicle went up the elevated voluntary surrender; and had the charge been
catwalk or pathway pursuing Achumbre who was hit homicide he could have pleaded guilty. We find that
when he was already at the railing (barandilla). Then these mitigating circumstances cannot be appreciated
she observed that the white vehicle drove away in his favor. Accused-appellant was allegedly "still very
without even caring to see what happened to them. angry" while he was following, bumping and pushing
The spouses/victims were brought to the police station the motorela which was in front of him. He was
while the Achumbre was brought to the hospital who previously mauled by the deceased and he was
was declared dead on arrival. It was later on found out allegedly rendered unconscious by the blows inflicted
upon investigation that said incident was predicated on on him. When he regained consciousness, he claims
the earlier fight which transpired between Achumbre that he wanted to look for a policeman to report that he
and the driver of the motor vehicle, Thadeos Enguito, was mauled. Clearly, accused-appellant's state of mind
the accused in this case. As a result of the death of after he was mauled and before he crushed Achumbre
Achumbre, his wife filed a criminal complaint against to death was such that he was still able to act
the accused. The Regional Trial Court found him guilty reasonably. In fact, he admitted having seen a police
with the crime of Homicide with Less Serious Physical mobile patrol nearby but instead, he chose to resort to
Injuries, taking into consideration the aggravating the dastardly act which resulted in the death of
circumstance of use of motor vehicle which was Achumbre and in the injuries of the spouses
alleged in the information. On appeal to the Court of Requerme. For passion to be considered as a
Appeals, the latter modified the crime to Murder due to mitigating circumstance, facts must be proved to show
the aggravating circumstance. The accused went to causes sufficient to produce loss of self-control and to
the Supreme Court imputing error on the decision of overcome reason. The turmoil and unreason which
the Court of Appeals with respect to the declaration of naturally result from a quarrel or fight should not be
the crime of Murder against him on the ground that he confused with the sentiment or excitement in the mind
did not intentionally choose the motor vehicle he was of a person injured or offended to such a degree as to
driving as a means of committing the offense, and that deprive him of his sanity and self-control.
at most, the vehicle was the only available means to
stop the deceased from escaping. He argued that it The mitigating circumstance of voluntary
was his intention to apprehend and surrender the surrender cannot be appreciated. Evidence shows that
deceased to the police for his previous act of mauling accused-appellant was further pursued by the police.
him but in the process, he killed the deceased. Appellant himself testified that he stopped his vehicle
52 | P a g e
just after the police mobile stopped but admitted According to the the witness, Jacalne, he was
having "stopped farther than the police mobile". SPO3 informed that one Dino Guerrero was inside the house
Catiil further testified that appellant did not surrender nearby. Dino Guerrero came out with his hands
but only stopped his vehicle when its right tire was extended forward. SPO1 Emilio Taracatac immediately
already flat. His testimony was corroborated by PO3 frisked and handcuffed him. Before Dino was
Makiling who was patrolling the portion of Marcos handcuffed, according to the witness, he said that it
Bridge. He testified that he saw the vehicle being was his son who had killed the victim. Thereafter, Dino
driven by accused-appellant already destroyed and the was brought to the police station for custodial
right portion of the vehicle a little bit lower as it was investigation.
running flat. Clearly, accused-appellant could have
eluded arrest but his situation became futile when his Further, Jacalne testified that appellant
vehicle suffered a flat tire. Orlando Guerrero, Jr., was not at the scene of the
crime during their investigation. But upon their return to
The foregoing notwithstanding, the existence the police station, appellant was already there. [11
or non-existence of a mitigating circumstance in the Appellant admitted killing the victim, according to
case at bar will not affect the penalty to be imposed Jacalne, by clubbing the victim first with the wooden
pursuant to Article 63 of the Revised Penal Code. The stick, and then cutting his head and his penis with a
crime committed by accused-appellant is the complex knife.
crime of murder with less serious physical injuries.
Under Article 48 of the Revised Penal Code, the Another withness, Ireneo Acierto, appellants
penalty for a complex crime shall be the maximum brother-in-law, testified that while he was resting in his
period of the penalty for the most serious crime. The house at past 11:30 in the morning of July 7, 1997, he
crime was committed in 1992 where the penalty for the heard someone screaming. When he looked out from
crime of murder, which is the most serious crime, was his window, he saw that the person screaming was his
reclusion temporal in its maximum period to death sister-in-law, Ana. He went out of the house and went
under Article 248 of the Revised Penal Code. The near the porch of the Guerreros, where he saw
death penalty being the maximum period of the penalty Ernesto Ocampos head about to be severed by
for murder should be imposed for the complex crime of appellant. When the head was cut off, appellant placed
murder with less serious physical injuries considering the same on the right side of the victims trunk. After
that under Article 63, an indivisible penalty cannot be that, appellant cut off Ernestos penis. Ireneo noticed
affected by the presence of any mitigating or that while the head was being severed, the victim was
aggravating circumstance. And, consonant with the lying down on the floor, but not moving. Ireneo then
ruling in People vs. Muñoz that Article III, Section 19 told appellant, That is enough, bayaw. Stop it.
(1) of the 1987 Constitution did not change the period According to the witness, his wife Ana was also
of the penalty for murder except only insofar as it saying, That is enough, Manong. Appellant angrily
prohibits the imposition of the death penalty and turned to Ireneo, telling him not to interfere or else he
reduces it to reclusion perpetua, the Court of Appeals might also be implicated. Ireneo hurriedly went away
was correct in imposing the penalty of reclusion after that. Ireneo did not see his father-in-law, Dino, at
perpetua. the time of the incident and did not know where Dino
was.

Cruelty The trial court convicted Orlando Guerrero, Jr.


of murder while his father Dino was acquitted.
People v. Guerrero (389 SCRA 389)
Issue:
Facts: Whether or not the court a quo gravely erred in
appreciating the qualifying circumstance of cruelty
Orlando Guerrero, Jr., also known as Pablo, and/or outraging and scoffing the corpse in order to
together with his father Orlando Guerrero, Sr., classify the killing as murder despite failure of the
nicknamed Dino, was accused of murder. The prosecution to allege the same in the information
accuseds, conspired, confederated and mutually
helped one another, with deliberate intent to kill and Decision:
with evident premeditation and treachery, did then and
there willfully, unlawfully and feloniously and without The information alleges the qualifying
justifiable cause, attack, assault, club, beheaded and circumstances of (1) treachery and (2) evident
cut off the penis of the victim Ernesto Ocampo, which premeditation. It also states that there was cruelty in
caused his death thereafter, to the damage and the perpetration of the crime, where there was
prejudice of his lawful heirs. deliberate and inhuman suffering of the victim and the
offender had scoffed at the victims corpse.
Upon arraignment, both pleaded not guilty.
Orlando interposed self-defense while his father, Dino, On treachery and evident premeditation, the
denied any complicity in the killing. trial court found that the evidence adduced by the
prosecution fell short of the requirements of the law. [
53 | P a g e
we hold that in the present case, the trial court did not purposely chosen to facilitate the commission of the
err when it found neither treachery nor evident crimes. Recidivism was also considered because the
premeditation. However, the trial court found there was accused "admitted that in 1963 he was convicted of
cruelty as well as outraging or scoffing at the corpse, murder and frustrated murder for which he was
thus, qualifying the crime to murder. sentenced to serve 8 years to 14 years imprisonment.
As a matter of fact, he was granted parole and at the
time of the incident he was a parolee.
Simangan v. People (434 SCRA 38)
ISSUE:

Facts: On February 10, 1980 at about Whether or not the trial court erred in
8 pm, Simangan and four other men concluding that there was treachery?
wearing fatigues knocked on the door
of the store owned by Ernesto and Decision:
Sofronia. The couple was having
dinner with their daughter Lorna. The record shows that the appellant had a
Simangan asked Ernesto to guide previous misunderstanding with Ligaya Santos; and
them on the road as they were not that when he suddenly attacked her he was with a
familiar. Ernesto agreed, he then group of armed men so that there was present not only
ordered his houseboy Romeo to the element of surprise but also the advantage of no
accompany him in guiding the group risk to himself.
of Simangan. The next morning,
Romeo reported to Sofronia that Aleviosa was certainly present.
Ernesto is dead. Ernesto was found
near a creek, he sustained 10 stab
wounds. Art. 15: Alternative Circumstances
 
Issue: Relationship

Whether or not Simangan is People v. Calongui (G.R. No. 170566)


guilty beyond reasonable doubt.
 
Decision: Facts:

Yes. The testimonies of Calonqui was found guilty for two counts of
Romeo and Sofronia are credible. rape. On January 1, 1998 about 2 am in Tagbong,
Thus, Simangan’s conviction is Camarines Sur, Calonqui was able to rape the 13 year
affirmed. It is found that Simangan old girl Maricel in the latter’s house. On September 26,
stabbed Ernesto 10 times, three of 1998 at about three in the morning, the accused again
which were fatal. But the number of raped the victim. Both rape incidents were witnessed
stab wounds does not qualify as an by the brother of Maricel.
aggravating circumstance against
Simangan for it must be proven that Issue: 
Simangan intended to exacerbate the
suffering of Ernesto. Nigh time is also Whether or not the aggravating circumstances
not appreciated as it was included in of dwelling and relationship be appreciated against
the original information. Calonqui and the latter circumstance as an alternative
circumstance?
People v. Catian (374 SCRA 514)

Decision: 
Facts:
Calonqui and Maricel live under the same
PEDRO CATACUTAN alias "Pedro Duling" shelter as they are first cousins. At the time of the
was accused of murder and frustrated murder. incident, both are living in the same house and in the
same room. Therefore, the supposed aggravating
The trial court adjudged the accused guilty of circumstance of dwelling cannot be appreciated as
murder qualified by treachery for the death of Ligaya there was no trespass to the sanctity of the house of
Santos. For the injury sustained by Renato Licup, the the victim on the part of Calonqui, while the
accused was adjudged guilty of attempted murder only aggravating circumstance of relationship is likewise
because Licup would not have died even without cannot go against Calongui, even as an alternative
medical attention. Nocturnity was not considered circumstance, as being first cousins is not within the
because the site of the shooting was well lighted and concept contemplated in Article 15 of the Revised
for lack of evidence to indicate that nighttime was
54 | P a g e
Penal Code. However, his conviction is nonetheless
affirmed. On August 12, 1998, the provincial prosecutor
filed with the Regional Trial Court, Surigao del Sur,
Branch 29, an Information for murder against accused
People v. Marcos (G.R. No. 132392) George Cortes y Ortega. Accused admitted that he
stabbed Edlyn and enter the plea of guilty of the said
crime. The prosecution presented evidence to prove
Facts: the presence of intoxication as aggravating
circumstances. The accused on the other hand
Prosecution, with the testimony of a lone eye- presented evidence proving the alternative mitigating
witness, who happened to be the son of the victim, of intoxication.
along with the admission of guilt, found Cesar Marcos
guilty beyond reasonable doubt of Murder for the killing On September 2, 1998, the trial court rendered
of his elder brother, Virgilio, as aggravated by the decision finding accused guilty beyond reasonable
qualifying circumstance of evident premeditation. doubt of the crime of Murder, and sentence to suffer
During Appeal, the Solicitor General insisted that the penalty of Death.
since the accused is a brother of the victim, the
alternative circumstance of relationship must be Issue:
considered in determining the imposable penalty.
Whether or not the crime committed by the
Issue: accused was aggravated by reason of intoxication?

Whether or not the alternative circumstance of Decision:


relationship shall be considered in the imposition of the
proper penalty? Ordinarily, intoxication may be considered
either aggravating or mitigating, depending upon the
Decision: circumstances attending the commission of the crime.
Intoxication has the effect of decreasing the penalty, if
In order that the Alternative Circumstance of it is not habitual or subsequent to the plan to commit
relationship may be taken into consideration in the the contemplated crime; on the other hand, when it is
imposition of the proper penalty, Paragraph 2 of Article habitual or intentional, it is considered an aggravating
15 of the Revised Penal Code provides that the circumstance. A person pleading intoxication to
offended party must either be the (a) spouse, (b) mitigate penalty must present proof of having taken a
ascendant, (c) descendant, (d) legitimate, natural or quantity of alcoholic beverage prior to the commission
adopted brother or sister, or (e) relative by affinity in of the crime, sufficient to produce the effect of
the same degree, of the offender. obfuscating reason. At the same time, that person
Relationship is Mitigating in Crimes against Property, must show proof of not being a habitual drinker and
while it must be considered as Aggravating in Crimes not taking the alcoholic drink with the intention to
against Persons where the offended party is a relative reinforce his resolve to commit the crime.
of a higher degree than the offender or when in the
same degree or level, as in brothers. Accused argues that in the absence of any of
the aggravating circumstances alleged in the
Therefore, the Alternative Circumstance of information and considering that there was one
relationship shall be considered as Aggravating.  mitigating circumstance attendant, that of plea of
guilty, the penalty imposable is not death but reclusion
Intoxication perpetua. The Solicitor General agrees with the
accused that "the only aggravating circumstance
People v. Marquita (G.R. No. 137050) present was treachery which qualified the killing to
murder and that there were two mitigating
circumstances of plea of guilty and intoxication, not
Facts: habitual. The penalty shall be reclusion perpetua, not
death, in accordance with Article 63 in relation to
On June 24, 1998, at about eleven o'clock in Article 248 of the Revised Penal Code, as amended by
the evening, Junilla Macaldo was sitting on a bench Republic Act No. 6759
outside her house. While thus seated, Edlyn Gamboa
came to her asking for the whereabouts of Yen-yen People v. Mondigo (G.R. No. 167954)
Ibuña. Junilla noticed that Edlyn was followed by
accused George Cortes. Junilla then instructed Edlyn
to go upstairs of the house. When Edlyn was about to Facts:
go upstairs, accused followed her and successively On September 27 1998, appellant, Damaso
stabbed her several times. Junilla tried to help Edlyn, Delima (Damaso), Damaso’s son Delfin and three
but was overpowered by the accused. Junilla shouted other unidentified individuals were having a drinking
for help and the accused run away. She was brought spree in Ligas, Malolos, Bulacan. Anthony (damaso’s
to the Babano Medical Clinic, where she expired. another son) joined the group. At around 6:00 p.m.,

55 | P a g e
Mondigo, using a "jungle bolo," suddenly hacked twice from his eye-level. Both Josephine and Eugenio
Anthony on the head, causing him to fall to the ground fell to the ground, the former, backwards, and the latter
unconscious. Appellant next attacked Damaso. A landing on top of her.
witness who was in the vicinity, Lolita Lumagi, hearing
shouts coming from the scene of the crime, rushed to Neighbors testified that Neil went out to the
the area and there saw appellant repeatedly hacking street, went between the parked white car and yellow
Damaso who was lying on his back, arms raised to taxicab, aimed the gun at Eugenio and Josephine who
ward off appellant’s blows. Damaso later died from the were at the mango tree, and then asked Castor: "Tay,
injuries he sustained. Anthony sustained wound on his banatan ko na?"; that Castor replied: "Sige, anak,
left temporal area.Appellant was charged before the banatan mo na."
RTC with Murder and Frustrated Murder. TheRTC
found appellant guilty of Murder for the killing of Issue:
Damaso and Serious Physical Injuries for the hacking
of Anthony, mitigated by intoxication. Whether or not the statement made by the
father made him liable as principal by inducement?
Issue:
Whether the trial court erred in giving Decision:
credence of alternative circumstance of intoxication to
mitigate the crime? The Court finds that Castor and Neil conspired
in shooting Eugenio. This finding is inexorable
Decision: because the testimonies of the Prosecution witnesses
The trial court erred in crediting appellant with that Castor returned the gun back to Neil; that he
the circumstance of intoxication as having mitigated instigated Neil to shoot by shouting: "Sige, banatan mo
his crimes because "the stabbing incident ensued in na"; and that Neil then fired his gun twice – were
the course of a drinking spree." For the alternative credible and sufficed to prove Castor’s indispensable
circumstance of intoxication to be treated as a cooperation in the killing of Eugenio. Accordingly,
mitigating circumstance, the defense must show that Castor was as much liable criminally for the death of
the intoxication is not habitual, not subsequent to a Eugenio as Neil, the direct participant in the killing,
plan to commit a felony and the accused’s was.
drunkenness affected his mental faculties. Here, the
only proof on record on this matter is appellant’s While Castor was indeed heard to have
testimony that before Damaso, Anthony, and Delfin shouted "Huwag," this cannot be considered as
attacked him, he drank "about 3 to 4 bottles of beer." reliable evidence that he tried to dissuade Neil from
The low alcohol content of beer, the quantity of such firing the gun. It was established by credible testimony
liquor appellant imbibed, and the absence of any that he handed back the gun to Neil and urged him to
independent proof that appellant’s alcohol intake shoot the Refugio spouses. Josephine Refugio plainly
affected his mental faculties all negate the finding that stated on cross-examination that Castor shouted
appellant was intoxicated enough at the time he "Huwag" while inside the car grappling for possession
committed the crimes to mitigate his liability. of the gun, and not when Neil was aiming the gun at
the spouses.

Arts. 16-20: Persons Criminally Liable for Felonies As concluded by the trial court, the
circumstances surrounding Castor’s utterance of
Principals "Huwag!" shows beyond doubt that Castor shouted the
same, not to stop Neil from firing the gun, but to force
People v. Batin (GR No. 177223) him to leave the use of the gun to Castor. These
circumstances only confirm the conspiracy between
Facts: the Batins in committing the crime: after the Batins
grappled for the gun and Castor shouted "Huwag,"
Eugenio’s wife, Josephine Refugio testified Castor finally decided to give the gun to Neil – a
she glanced to her left and saw Neil Batin standing at crystal-clear expression of the agreement of the Batins
the gate to their compound, looking towards her and concerning the commission of a felony.
her husband. A few moments later, Neil went to one of
the parked cars, opened its door, and took a gun from Conspiracy may also be deduced from the
inside. She next noticed Castor going towards Neil as acts of the appellants before, during, and after the
the latter stood at the side of the car and shouting: commission of the crime which are indicative of a joint
"Huwag!" Castor grabbed the gun from Neil. After the purpose, concerted action, and concurrence of
gun was taken from him, Neil just proceeded towards sentiments.Even if we pursue the theory that the
the right rear of the car. Castor followed Neil and defense is trying to stir us to, the results would be the
handed the gun back to him. When she shifted her same. Castor’s argument is that "(h)is alleged
glance from the Batins, Josephine heard Castor utterance of the words ‘Sige, banatan mo na’ cannot
ordering his son: "Sige, banatan mo na." Neil be considered as the moving cause of the shooting
responded by drawing the gun from his waistline, and, therefore, he cannot be considered a principal by
raising and aiming it at her and her husband, and firing inducement.
56 | P a g e
Whether Domingo Vasquez chased the
Inducement may be by acts of command, deceased with a bolo was averred by Luis Luable or
advice or through influence or agreement for whether the accused merely incited his companions in
consideration. The words of advice or the influence the jeepney to kill the deceased as averred by Luisa
must have actually moved the hands of the principal by Abellanosa, is immaterial in the determination of his
direct participation. We have held that words of liability because a conspiracy among the occupants of
command of a father may induce his son to commit a the jeepney has been established.
crime. The moral influence of the words of the father
may determine the course of conduct of a son in cases In order to hold an accused guilty as co-
in which the same words coming from a stranger principal by reason of conspiracy, it must be
would make no impression. There is no doubt in our established that he performed an overt act in
minds that Castor’s words were the determining cause furtherance of the conspiracy, either by actively
of the commission of the crime. participating in the actual commission of the crime,
or by lending moral assistance to his co-conspirators
People v. Vasquez (G.R. No. 123939) by being present at the scene of the crime, or by
exerting moral ascendancy over the rest of the
conspirators as to move them to executing the
Facts: conspiracy."

The appellant drove the passenger jeepney The Supreme Court, likewise, stressed that
with his cohorts on board looking for Luable and where there are several accused and conspiracy has
Geronimo. When the appellant saw the two going in been established, the prosecution need not pinpoint
the opposite direction, the appellant drove the vehicle who among the accused inflicted the fatal wound.
and sideswiped Geronimo. And when Geronimo fled, Where conspiracy has been established, evidence as
the appellant, armed with a bolo, pursued him. When to who among the accused rendered the fatal blow is
the appellant failed to overtake the victim, he returned not necessary. All the conspirators are liable as co-
to the passenger jeepney and drove it to where his principals regardless of the intent and character of
cohorts ganged up on the victim. The appellant urged their participation because the act of one is the act of
them on to kill Geronimo. Thereafter, he left the scene all.
along with his cohorts, leaving the hapless Geronimo
mortally wounded. Article 8 of the Revised Penal Code provides
that there is conspiracy when two or more person
After trial, the court rendered judgment agree to commit a felony and decide to commit it.
acquitting Ramon, but convicting the appellant of Conspiracy need not be proven by direct evidence. It
murder for the killing of Geronimo, and attempted may be inferred from the conduct of the accused
homicide for attempting to kill Luis. before, during and after the commission of the crime,
showing that they had acted with a common purpose
The appellant avers that he and his brother and design. Conspiracy may be implied if it is proved
Ramon had no motive to kill Geronimo. The appellant that two or more persons aimed by their acts towards
contends that the witnesses for the prosecution were the accomplishment of the same unlawful object, each
not in agreement as to who killed Geronimo. The doing a part so that their combined acts, though
appellant noted that according to the testimony of the apparently independent of each other were, in fact,
witness, the appellant stayed in the jeepney and connected and cooperative, indicting a closeness of
merely yelled to his companions who ganged up on personal association and a concurrence of sentiment.
Geronimo, "Sige patayin ninyo, patayin ninyo na, at Conspiracy once found, continues until the object of it
huwag ninyong iwanang buhay!" has been accomplished and unless abandoned or
broken up. To hold an accused guilty as a co-principal
The appellant further posits that the by reason of conspiracy, he must be shown to have
prosecution witnesses were not even in accord as to performed an overt act in pursuance or furtherance of
where Geronimo was stabbed to death. The appellant the complicity. There must be intentional participation
argues that because of the inconsistencies in the in the transaction with a view to the furtherance of the
testimonies of the witnesses of the prosecution, it common design and purpose. Each conspirator is
failed to prove his guilt beyond reasonable doubt of the responsible for everything done by his confederates
crimes charged. Hence, he should be acquitted of the which follows incidentally in the execution of a
said charges. common design as one of its probable and natural
consequences even though it was not intended as part
Issue: of the original design.

Whether the trial court erred in convicting the Responsibility of a conspirator is not confined
appellant when the witnesses testimony didn’t confirm to the accomplishment of a particular purpose of
who chased and stabbed the victims? conspiracy but extends to collateral acts and offenses
incident to and growing out of the purpose intended.
Decision: Conspirators are held to have intended the
consequences of their acts and by purposely engaging
57 | P a g e
in conspiracy which necessarily and directly produces At past 10:00 a.m., appellant brought the concrete
a prohibited result, they are, in contemplation of law, mixture and cast the dead body in cement.
chargeable with intending that result. Conspirators are
necessarily liable for the acts of another conspirator
unless such act differs radically and substantively from Issue:
that which they intended to commit. When a
conspirator embarks upon a criminal venture of Whether or not appellant is liable as a
indefinite outline, he takes his chances as to its principal?
content and membership, so be it that they fall within
the common purposes as he understands them." Decision:

All the foregoing constitutes evidence beyond The rule is that any admission made by a party
cavil of conspiracy between the appellant and the in the course of the proceedings in the same case
principals by direct participation. The appellant is, thus, does not require proof to hold him liable therefor. Such
criminally liable for the death of the victim, although admission may be contradicted only by showing that it
there is no evidence that he did not actually stab the was made through palpable mistake or no such
latter. admission was in fact made. There was never any
such disclaimer by appellant.
People v. Dacillo (G.R. No. 149368)
Moreover, despite appellant’s self-serving,
exculpatory statement limiting his involvement in the
Facts: crime, all circumstances pointed to his guilt. Assuming
for the sake of argument that Pacot was the
Appellant was convicted by the trial court of mastermind, appellant’s admission that he participated
the crime of murder for the death of Rosemarie in its commission by holding Rosemarie’s legs made
Tallada, with aggravating circumstance of recidivism him a principal by direct participation.
with no mitigating circumstance to offset the same, and
sentenced to the extreme penalty of death. Two or more persons taking part in the
commission of a crime are considered principals by
In his defense, appellant admitted complicity in direct participation if the following requisites are
the crime but minimized his participation. Appellant present: 1.) they participated in the criminal resolution
alleged that he only held down Rosemarie’s legs to and 2.) they carried out their plan and personally took
prevent her from struggling and, after the latter was part in its execution by acts which directly tended to
killed by another man he identified as Joselito Pacot, the same end.
he encased the corpse in cement.
Both requisites were met in this case. Two or
He claimed that Pacot, a co-worker at Davao more persons are said to have participated in the
Union Cement Corporation (DUCC), was looking for a criminal resolution when they were in conspiracy at the
house where he and his girlfriend Rosemarie could time of the commission of the crime. To establish
spend the night. He offered his brother’s house which conspiracy, it is not essential that there be proof of the
was under his care. In the evening of February 6, previous agreement and decision to commit the crime,
2000, he and Joselito Pacot brought Rosemarie to the it being sufficient that the malefactors acted in concert
house at Purok No. 3, New Society Village, Ilang, pursuant to the same objective.
Davao City.
It is well-settled that a person may be
After accompanying the couple there, he went convicted for the criminal act of another where,
home to take supper. Later that evening, he returned between them, there is conspiracy or unity of purpose
to the house with the bottle of Sprite Pacot had and intention in the commission of the crime
ordered. When he arrived, Pacot and Rosemarie were charged. Conspiracy need not be proved by direct
already grappling with each other and Pacot was evidence of prior agreement on the commission of the
strangling the girl. He told Pacot to stop but instead of crime as the same can be inferred from the conduct of
heeding him, the latter ordered him to close the door. the accused before, during, and after the commission
Pacot told appellant that he was going to be implicated of the crime showing that they acted in unison with
just the same so he closed the door as ordered and each other pursuant to a common purpose or design.
helped Pacot "(hold) the feet of the woman" as "her
feet kept hitting the walls." Accomplices

The two men stopped only when Rosemarie People v. Roche (G.R. No. 115182)
was already motionless. Pacot wanted to dump the
body into the sea but appellant told him it was low tide.
Appellant then suggested that they entomb the body in Facts:
cement for which Pacot gave appellant P500.Pacot left
the house at dawn the following day, February 7, 2000. An information for the murder of Roderick
Ferol was filed against accused-appellant Restituto

58 | P a g e
Roche and three others, namely, Marcelino Fallore, Caballes borne out of the desire to get even with him
Francisco Gregorio, and one John Doe. The Court for the offense committed against his brother. In no
found that the prosecution evidence has established way can such act be attributed to accused-
beyond reasonable doubt the guilt of accused appellant.Neither can accused-appellant be held liable
Restituto Roche for the crime of murder but could not as an accomplice for the crime charged. The following
make a pronouncement as to the guilt of accused requisites must concur in order that a person may be
Dorico Caballes because he remained at large and considered an accomplice: (a) community of
therefore could not be arraigned.Finding that the design, i.e.,  knowing that criminal design of the
prosecution evidence failed to establish the guilt of principal by direct participation, he concurs with the
accused Francisco Gregorio and Marcelino Fallore, latter in his purpose; (b) he cooperates in the
both accused were acquitted. execution of the offense by previous or simultaneous
acts; and, (c) there must be a relation between the
Issue: acts done by the principal and those attributed to the
person charged as accomplice.
Whether or not the accused-appellant should
held liable for the killing of Roderick Ferol on the There is no evidence to show that accused-
ground of conspiracy? appellant performed any previous or simultaneous act
to assist Dorico Caballes in killing Roderick Ferol. In
Decision: fact, it has not been proven that he was aware of
Dorico Caballes plan to attack and kill Roderick Ferol.
No. In the case at bar, Rogelio Rossel testified Absent any evidence to create the moral certainty
that he did not see Restituto Roche at the time Dorico required to convict accused-appellant, we cannot
Caballes was stabbing Roderick Ferol.Apart from uphold the trial courts finding of guilt. Our legal culture
Helen Amarille and Rodel Ferol, whose testimonies demands the presentation of proof beyond reasonable
are highly suspect, no other witness was presented to doubt before any person may be convicted of any
prove that accused-appellant directly participated in crime and deprived of his life, liberty, or even property.
the commission of the offense or performed an act The hypothesis of his guilt must flow naturally from the
which would show community of purpose with Dorico facts proved and must be consistent with all of them.
Caballes. Even if it is assumed as true that accused-
appellant was responsible for telling Dorico Caballes it Abarquez v. People (G.R. No. 150762)
was Roderick Ferol who had tripped him (Restituto),
this would not suffice to find accused-appellant in
conspiracy with Dorico Caballes. Facts:

For conspiracy to exist, proof of an actual The prosecution charged Abarquez with the
planning of the perpetration of the crime is not a crimes of homicide and attempted homicide alleging in
condition precedent. It may be deduced from the mode the two informations filed that said accused was
and manner in which the offense was perpetrated or conspiring and confederating with one Alberto
inferred from the acts of the accused evincing a joint or Almojuela in the killing of Ricardo Quejong Bello, by
common purpose and design, concerted action and stabbing him twice with a bladed weapon and hitting
community of interest. him with a gun at the back.

In People v. Elijorde, Conspiracy must be The trial court found Abarquez guilty beyond
proved as indubitably as the crime itself through clear reasonable doubt as an accomplice in the crime of
and convincing evidence, not merely by conjecture. To homicide.
hold an accused guilty as a co-principal by reason of
conspiracy, he must be shown to have performed an Abarquez filed an appeal to the Court of
overt act in pursuance or furtherance of the complicity. Appeals. However the Court of Appeals rejected
Hence, conspiracy exists in a situation where at the Abarquez’s allegation that he was merely at the crime
time the malefactors were committing the crime, their scene to pacify the quarreling parties.
actions impliedly showed unity of purpose among
them, a concerted effort to bring about the death of the Abarquez alleges that the prosecution’s
victim. In a great majority of cases, complicity was evidence does not satisfy the test of moral certainty
established by proof of acts done in concert, i.e., acts and is not sufficient to support his conviction as an
which yield the reasonable inference that the doers accomplice. He further alleges that there was a
thereof were acting with a common intent or design. misapprehension of facts and that the trial court and
Therefore, the task in every case is determining the Court of Appeals reached their conclusion based
whether the particular acts established by the requisite entirely on speculation, surmises and conjectures.
quantum of proof do reasonably yield that inference. Abarquez also assails the credibility of the witnesses
against him.
Indeed, there is no proof to show accused-
appellant, together with Dorico Caballes, had resolved Issue:
to attack Roderick Ferol. Instead, we think the assault
on Roderick Ferol was an impulsive act by Dorico
59 | P a g e
Whether or not there is sufficient evidence to Mauricio Gonzaga and Lemuel Compo were
prove that fact that Abarquez was an accomplice in the charged conspiring in the murder of Procopio Dales
killing of Ricardo Bello? .
Based on the statements of Libardo, Gonzaga,
Decision:No. PO3 Pedro Wate, and the post mortem findings the the
Court held the accused Mauricio Gonzaga and Lemuel
Two elements must concur before a person becomes Compo guilty of the crime of murder punished under
liable as an accomplice: Article 248 of the Revised Penal Code and sentenced
each one of them to suffer an imprisonment of
(1) community of design, which means that the Reclusion Perpetua.
accomplice knows of, and concurs with, the
criminal design of the principal by direct Accused Lemuel Compo filed with the trial
participation; and court a notice of appeal. In this appeal, accused-
(2) the performance by the accomplice of appellant imputes a single assignment of error to the
previous or simultaneous acts that are not trial court.
indispensable to the commission of the
crime.Mere commission of an act, which aids Issue:
the perpetrator, is not enough.
Whether or not the testimony of the witness
The cooperation that the law punishes is the Librado that he saw Lemuel carrying an Indian Pana is
assistance knowingly rendered, which cannot exist sufficient to establish the latter is an accomplice to the
without the previous cognizance of the criminal act crime?
intended to be executed. It is therefore required in
order to be liable as an accomplice; that the accused Decision: No.
must unite with the criminal design of the principal by
direct participation. The court held that the prosecution failed to
overcome the constitutional presumption of innocence.
The court held in one case that the mere Basically, accused-appellant Lemuel was convicted
presence of the accused at the crime scene cannot be based on the testimony of the conductor of passenger
interpreted to mean that he committed the crime bus Gilberto Libardo who saw Lemuel carrying an
charged. “Indian Pana” and a flashlight. Without any testimony
positively identifying accused-appellant as the
In convicting Abarquez in this case, the trial assailant nor any evidence directly linking him as the
court and the Court of Appeals relied mainly on the author of the crime, Lemuel Compo cannot be
testimony of Paz. Paz testified that he was held by convicted of the murder of Dales.  The accused-
Abarquez on the shoulders, thus preventing him from appellant deserves an acquittal and must forthwith be
helping Quejong who was grappling with Almojuela. given back his liberty.

Paz’s testimony does not show that Abarquez The testimony of witness Mauricio Gonzaga,
concurred with Almojuela’s criminal design. "Tumigil" states that Lemuel was merely present before the
literally means "stop." Clearly, Abarquez was trying to stabbing incident, holding a flashlight. No other overt
stop Paz from joining the fray, not from helping act was established to prove that Lemuel shared and
Quejong. Paz claims that he was only trying to talk to concurred with the criminal design of Mauricio.  The
Almojuela. However, Paz could not have been merely mere presence of Lemuel, who was not shown to be
talking to Almojuela, as he tried to portray, because armed, at the scene of the crime does not connote
Almojuela was already grappling with Quejong at that conspiracy. 
time. Paz interpreted Abarquez’s action as an attempt
to prevent him from helping Quejong. His interpretation Singularity of purpose and unity in the
was adopted by the trial court and sustained by the execution of the unlawful objective are essential to
Court of Appeals. Yet, in his testimony, Paz admitted establish conspiracy.
that while restraining him, Abarquez was scolding or Mere knowledge, acquiescence, or agreement
reprimanding him and telling him to stop. It was not to cooperate, is not enough to constitute one as a
shown that Abarquez was stopping Paz from helping party to a conspiracy, absent any active participation in
Almojuela. It is more likely that Abarquez was trying to the commission of the crime, with a view to the
stop Paz from joining the fight. Abarquez’s act of trying furtherance of the common design and
to stop Paz does not translate to assistance to purpose.Conspiracy transcends companionship.The
Almojuela. presence and company of Lemuel were not necessary
or essential to the perpetration of the murder.
People v. Compo (G.R. No. 112990)
Neither can Lemuel be considered an
accomplice.  Article 18 of the Revised Penal Code
Facts: provides that an accomplice is one who, not being a
principal, cooperates in the execution of the offense by
previous or simultaneous acts.  To be convicted as
60 | P a g e
such, it is necessary that he be aware of the criminal people, he will kill them. Out of fear, they just followed
intent of the principal and then cooperate knowingly or whatever Tolentino told them.
intentionally by supplying material or moral aid for the
efficacious execution of the crime. On 01 March 1996, however, Jonathan was
The prosecution, however, failed to present arrested for the death of Hernan Sagario. Accused
convincing evidence establishing that accused- Jonathan Fabros and Wilfredo Tolentino both denied
appellant Lemuel knew of the other accused's intent to killing the victim. Instead, they pointed to each other as
kill Dales.  Again, his mere presence at the scene of the one who killed Hernan Sagario. Fabros pointed to
the crime and his flight therefrom with the other Tolentino as the assailant and the latter also fingered
accused are not proof of his participation in the crime. the former as the killer of Sagario.
The quantum of proof required in criminal prosecution
to support a conviction has not  been reached with However, on 14 July 2000, long after the trial
regard to accused-appellant Lemuel. The oft-repeated court's decision had become final and executory on his
truism that the conviction of an accused must rest not part, Wilfredo Tolentino, apparently conscience-
on the weakness of the defense but on the strength of stricken, executed an affidavit admitting sole
the prosecution’s evidence applies.He must, therefore, responsibility for the death of Hernan Sagario and
be acquitted on reasonable doubt. retracted his testimony implicating accused-appellant
Jonathan Fabros.

Accessories The trial court held that the prosecution's


evidence positively identified Wilfredo Tolentino as the
People v. Tolentino (G.R. No. 139179) person who had hit the victim with a piece of wood and
later stabbed him with a bolo. It also ruled that the
killing was qualified by treachery and attended by the
Facts: aggravating circumstance of dwelling.
On February 28, 1996 appellant Jonathan The court a quo  observed that overt and
Fabros and his cousins, Sheila Guilayan and Merwin positive acts of appellant (Jonathan Fabros)
Ledesma, were at their house in Luyahan, Pasonanca, manifested his approval of the killing and the
Zamboanga City when their neighbor Wilfredo concurrence of his acts with those of the other
Tolentino called them. When asked what it was all accused.8 Thus, the RTC concluded that Fabros was a
about, Wilfredo simply motioned to them to come to his co-conspirator and should be held equally responsible
house located just across the road. Once they were for the murder.
inside the house, Wilfredo immediately revealed his Hence, this appeal.
plan to kill Hernan Sagario, Sheila's stepfather.
Wilfredo explained that it was the only way to free Issue:
Sheila's mother - appellant's aunt - of the sufferings
being caused by Hernan. Wilfredo then instructed Whether or not appellant (Jonathan Fabros)
Merwin to go back to the house and get the bolo of should be convicted as an accessory?
Hernan. Merwin obliged, got the bolo, and gave it to
Wilfredo. Thereafter, they were told by Wilfredo to go Decision:
home and wait for Hernan.
Appellant cannot be convicted as an
Around 8:30 in the evening, Hernan arrived. accessory. Article 19 of the Revised Penal Code
He went directly to the kitchen and fixed the bag of rice defines an accessory as one who had knowledge of
he was carrying. Jonathan together with Sheila and the commission of the crime and did not participate in
Merwin, just stayed quiet in the living room.Later, its commission as principal or accomplice, yet took
Wilfredo with a 2"x2" piece of wood in his hand part subsequent to its commission by any of three
entered the house. He then followed Hernan towards modes: (1) profiting oneself or assisting the offender to
the kitchen. When about an armslength away from profit by the effects of the crime; (2) concealing or
Hernan, Wilfredo, immediately walloped Hernan on the destroying the body of the crime, or the effects or
right side of the neck sending the latter unconscious instruments thereof, in order to prevent its discovery;
and falling face down to the ground. Wilfredo and (3) harboring, concealing, or assisting in the
immediately instructed appellant and Merwin to help escape of the principals of the crime, provided the
him bring Hernan out of the house. Lifting Hernan out accessory acts with abuse of his public functions or
of the house, Wilfredo held him by the neck while both when the offender is guilty of treason, parricide,
appellant and Merwin grasped his feet. They then murder, or an attempt to take the life of the Chief
carried Hernan towards the creek, upon reaching the Executive, or is known to be habitually guilty of some
creekside, the three stopped, then Wilfredo other crime. To convict an accused as an accessory,
successively stabbed Hernan on different parts of the the following elements must be proven: (1) knowledge
body causing the latter's instant death. After throwing of the commission of the crime and (2) subsequent
the victim's lifeless body in the creek, the three participation in it by any of the three above-cited
immediately left. Tolentino called Jonathan, Sheila and modes.
Merwin and warned them that if they will tell other

61 | P a g e
Under paragraph 2 of said codal provision, the Garcia, Yul Alvarez, a certain Benjie, a certain Leos
concealment or the destruction of the body of the and a certain Laring as the look-outs who stayed
crime or of the effects or the instruments thereof must outside the Lim compound. He named Toto Garcia as
have been done in order to prevent the discovery of the chief plotter of the crime at bar, and revealed that
the crime. That, precisely, is wanting in the present his neighbor and close family friends, the spouses
case. Leonilo and Beverly Cui, participated in the plan.
Basingan said he was asked to join the plot and was
In his testimony, appellant stated that because assured that he would not be under suspicion because
he was afraid his co-accused would hurt him if he he would be placed at gun point together with the other
refused, he agreed to assist the latter in carrying the members of the Lim household when the crime is
victim towards the river. The fact that appellant left committed. However, he refused to join the plot during
thereafter likewise indicated his innocence of the the December 2, 1990 meeting of the group at the
charge. Verily, he adequately explained his conduct residence of the Cuis in Quiot, Pardo, Cebu City.
prior to the stabbing incident as one born of fear for his Leonilo Cui even invoked their close ties as godfathers
own life. It is not incredible for an eyewitness to a of each other's children but he was unmoved. At the
crime, especially if unarmed, to desist from assisting meeting were Toto Garcia, Mawi Garcia, Edgar, Rey,
the victim if to do so would put the former's life in peril. Sadam and the Cuis.

The presumption of innocence in favor of On December 18, 1990, Basingan executed a


appellant has not been overcome by proof beyond sworn statement reiterating these revelations in
reasonable doubt. Thus, he must be acquitted. writing. Johnny and Rose Lim then formalized their
complaint by executing a Joint Affidavit. Assistant
People v. Cui (G.R. No. 121982) Prosecutor Bienvenido N. Mabanto, Jr. filed an
information for Kidnapping with Ransom against
Basingan, the Cuis, and the members of the group of
Facts: Toto Garcia as identified by Basingan in his sworn
statement.On the same day, Basingan and Leonilo Cui
In the evening of December 5, 1990, ten (10) were arrested. 
armed robbers raided the compound of Johnny and
Rose Lim on Edison Street, Lahug, Cebu City. The On March 14, 1991, Joselito "Tata" Garcia,
Lims, their three (3) children, and the employees of the Hilaria Sarte and her live-in partner, Luis Obeso,
family-owned business, were able to see the faces of referred to by Basingan as "Laring" and "Leos",
the leader Wilfredo alias "Toto" Garcia and two of his respectively, were arrested in the neighboring Negros
men, Mawe Garcia and a certain Edgar. The other Island. The next day, however, Tata Garcia died due to
robbers could not be identified as they had flour sacks "hemorrhage, severe, secondary to gunshot
over their heads. The robbers carted away cash and wounds." Upon presentation of his death certificate,
jewelries worth twenty thousand pesos (P20,000.00). the trial court ordered his name deleted from the
They also blindfolded and forcibly abducted seventeen information.
(17) year old Stephanie, the youngest daughter of the
Lims. They demanded a ransom of one million pesos After preliminary investigation, Prosecutor
(P1,000,000.00) for her release.Johnny Lim turned Manuel J. Adlawan found that the participation of the
over to Toto Garcia the ransom amount in the Cuis was only that of accomplices amended the
afternoon of the next day at an arranged meeting Information downgrading the charge against the Cui’s
place. Stephanie, in turn, was released to her father. as mere accomplices in the kidnapping with ransom of
Stephanie Lim.
Initially, the Lim’s kept the crime a secret. But
on the third day, they reported the kidnapping to the On May 15, 1991, Basingan, the Cuis, Obeso
Philippine National Police Cebu Metropolitan District and Sarte were arraigned and they all pleaded not
Command (Cebu Metrodiscom). The Metrodiscom guilty. On June 27, 1991, Basinga escaped from the
Intelligence Security Team (MIST) conducted an prison. Trial on the merit ensued against the Cuis,
investigation and Johnny Lim identified one of the Obeso and Sarte. Basingan was tried in absentia.
suspects as Toto Garcia.
On February 13, 1992, Beinvenido
Toto Garcia was known as the leader of a Nacario, alias "Rey Nacario", was arrested. On
group of armed robbers called the Baong Gang. The arraignment on April 13, 1992, he pleaded not guilty.
gang's base of operation was pinpointed at Quiot, However, on May 5, 1991, he, too, escaped from
Pardo, Cebu. When the police learned that Eduardo detention and remains at large to this date.
Basingan, hailed from Quiot, Pardo, Cebu City, they On August 18, 1992, the prosecuting fiscal manifested
decided to interrogate him. before the trial court that, per newspaper report, Toto
Garcia had been killed in Davao. Thus, on December
Upon Basingan's interrogation, he identified 6, 1993, the trial court convicted the Cuis, Obeso,
Toto Garcia, Mawe Garcia and Edgar as the three (3) Sarte, Basingan and Nacario. 
who did not wear masks, Sadam and Rey as the two Obeso and Sarte filed their Notice of Appeal on May
(2) who held him and the Lims at gunpoint, and Tata 19, 1994. The Cui’s filed theirs on May 31, 1994. 
62 | P a g e
evening of September 28. 1969, appellant Ricardo
Issue: Verzola went to their house in Barrio Lipcan, Bangued
Abra entered the room where she was sleeping with
Whether or not the Cui’s are liable as her husband, Bernardo Molina, woke her up and had
accessories? carnal knowledge of her; that when Bernardo Molina
Decision: woke up and attempted to rise from the floor, that was
the moment when Verzola clubbed Bernards, hitting
Art. 19 of the Revised Penal Code, as him on the head several times that afterwards, she
amended, penalizes as accessories to the crime those heard the sound of a body being dragged downstairs
who, subsequent to its commission, take part therein and the voice of Verzola saying that he was leaving
by profiting themselves or assisting the offenders to and warning her not to say anything about the incident.
profit by the effects of the crime, without having She looked out of the door and saw her husband
participated therein, either as principals or already lying prostrate at the foot of the stairs. This
accomplices. Conviction of an accused as an statement was sworn to by her before Municipal Judge
accessory requires the following elements: (1) that he Francisco T. Valera.
has knowledge of the commission of the crime; and (2)
that he took part in it subsequent to its commission by On that same morning, appellant Verzola was
any of the three modes enumerated in Article 19 of the picked up by the police and brought to the municipal
Revised Penal Code, as amended. These twin building, and there he also executed a written
elements are present in the case of the Cui’s, and statement admitting that he clubbed the victim several
indubitable proof thereof is extant in the records of the times. Both appellants admit that it was appellant
case. Verzola who inflicted the fatal blows on the victim.

The Court held that the Cui’s profited from the The trial court convicted Verzola as principal
kidnapping of Stephanie Lim and are liable as and Josefina Molina as an accessory to the crime of
accessories. murder.

People v. Verzola (G.R. No. L-35022) Issue:


Facts:
Whether or not assisting the principal in
On September 28, 1969, Bernardo Molina was bringing the body of the deceased to the ground will
clubbed to death by Ricardo Verzola in the presence of make one an accessory to the crime?
appellant Josefina Molina inside Molina's house at
Barrio Lipcan, Bangued, Abra. The body of the victim Decision:
was subsequently carried by the two appellee to the
ground and left at the foot of the stairs. Appellant An accessory does not participate in the
Verzola then went to his house, changed his clothes criminal design, nor cooperate in the commission of
and threw his bloodstained sweater undershirt and the felony, but with knowledge of the commission of
underwear, including the piece of wood be used in the crime, he subsequently takes part in three (3)
clubbing the deceased, inside their toilet. Afterwards, ways: (a) by profiting from the effects of the crime; (b)
he went to the municipal building and reported to the by concealing the body, effects or instruments of the
police authorities that Bernardo had died in an crime in order to prevent its discovery; and (c) by
accident. The police authorities together with the assisting in the escape or concealment of the principal
Municipal Health Officer, the Municipal Judge and a of the crime, provided he acts with abuse of his public
photographer went to Lipcan to conduct the functions or the principal is guilty of treason, parricide,
investigation. They found the body of the deceased murder, or an attempt to take the life of the Chief
Bernardo Molina sprawled at the foot of the bamboo Executive or is known to be habitually guilty of some
ladder. Blood had oozed from the mouth, nose and other crime.
ears. There were bloodstains on the floor of the
bedroom of the house, on the mat, as well as on the Even if she assisted her co-appellant without
beddings of the deceased. The bloodstains led to the duress, simply assisting Verzola in bringing the body
bamboo ladder where some of the stains could be down the house to the foot of the stairs and leaving
found on the steps of the ladder. When questioned by said body for anyone to see, cannot be classified as an
the police, Josefina revealed that the assailant of her attempt to conceal or destroy the body of the crime.
husband was Ricardo Verzola. The concealing or destroying of the body of the crime,
the effects or instruments thereof, must be done to
Upon her request, she was brought to the prevent the discovery of the crime. In the case at bar,
Office of the Chief of Police of Bangued, where at the body was left at the foot of the stairs at a place
about 2:00 o'clock in the morning of September 29, where it was easily visible to the public. Under such
1969 she gave a written statement narrating the circumstances there could not have been any attempt
circumstances surrounding the incident in question on the part of Josefina to conceal or destroy the body
and pointing to appellant Verzola as the assailant of of the crime. Thus, Josefina Molina is acquitted.
her husband. In that extra-judicial statement, she
stated that immediately after 10:00 o'clock in the
63 | P a g e
Accessories Exempt from Criminal Liability Respondent Judge issued an order granting the
motion to quash on the ground of lack of jurisdiction
People v. Mariano (G.R. No. L-40527) but did not rule on the other grounds invoked in the
motion to quash.

Facts: Issue:

That on or about and during the period from Whether or not Mariano can be held liable for
May 11 and June 8, 1971, in San Jose del Monte, estafa?
Bulacan, the said accused Hermogenes Mariano,
being then appointed as Liaison Officer by the then Decision:
incumbent Municipal Mayor, Constantino Nolasco,
acting for and in behalf of the municipality and The Supreme Court ruled that Respondent
authorized to receive and be receipted for US excess court gravely erred when it ruled that it lost jurisdiction
property of USAID/NEC for the use and benefit of said over the estafa case against respondent Mariano with
municipality, received from the said USAID/NEC the the filing of the malversation charge against Mayor
following items with a total value of $717.50 or Nolasco before the Military Commission. Estafa and
P4,797.35, involving the duty of making delivery of malversation are two separate and distinct offenses
said items to the said Municipal Mayor, but the said and in the case now before the SC the accused in one
accused Hermogenes Mariano once in possession of is different from the accused in the other.
the said items and far from complying with his
aforesaid obligation and in spite of repeated demands, The conferment of jurisdiction upon courts or
did then and there wilfully, unlawfully and feloniously, judicial tribunals is derived exclusively from the
with grave abuse of confidence and with deceit, constitution and statutes of the forum. Thus, the
misappropriate, misapply and convert to his own question of jurisdiction of respondent Court of First
personal use and benefit the said items valued at Instance over the case filed before it is to be resolved
$717.50 or P4,797.35, belonging to the said on the basis of the law or statute providing for or
USAID/NEC, to the damage and prejudice of the said defining its jurisdiction. That, We find in the Judiciary
owner in the said sum of $717,50 or P4,797.35. On Act of 1948 where in its Section 44 (f) it is provided
February 19, 1975, Hermogenes Mariano thru his that Courts of First Instance shall have original
counsel Filed a motion to quash the Information on the jurisdiction In all criminal cases in which the penalty
following grounds: provided by law is imprisonment for more than six
months,or a fine of more than two hundred pesos.The
1. That the court trying the cause has no offense of estafa  charged against respondent Mariano
jurisdiction of the offense charged or of the person of is penalized with arresto mayor  in its maximum period
the defendant; to prision correccional in its minimum period, or
2. That the criminal action or liability has been imprisonment from four (4) months and one (1) day to
extinguished; two (2) years and four (4) months. By reason of the
3. That it contains averments which , if true, penalty imposed which exceeds six (6) months
would constitute a legal excuse or justification. imprisonment, the offense alleged to have been
committed by the accused, now respondent, Mariano,
In his motion to quash, Mariano claimed that falls under the original jurisdiction of courts of first
the items which were the subject matter of the instance.
Information against him were the same items for which
Mayor Constantino A. Nolasco was indicted before a The above of course is not disputed by
Military Commission under a charge of malversation of respondent Judge; what he claims in his Order is that
public property, and for which Mayor Nolasco had his court exercises concurrent jurisdiction with the
been found guilty and that inasmuch as the case military commission and because the latter tribunal
against Mayor Nolasco had already been decided by was the first to take cognizance of the subject matter,
the Military Tribunal, the Court of First Instance of respondent court lost jurisdiction over it .That
Bulacan had lost jurisdiction over the case against him. statement of respondent court is incorrect. In People
On March 14, 1975 respondent Judge issued vs. Fontanilla, this Court speaking through then Justice
an Order granting the motion to quash on the ground now Chief Justice Fred Ruiz Castro, categorically
of lack of jurisdiction reasoning as follows: reiterated the settled rule that the jurisdiction of a court
is determined by the statute in force at the time of the
Considering that the Military Commission had commencement of the action. In the case at bar, it is
already taken cognizance of the malversation case rightly contended by the Solicitor General that at the
against Mayor Nolasco involving the same subject time Criminal Case No. SM-649 was filed with the
matter in its concurrent jurisdiction with this Court, the Court of First Instance of Bulacan, that was December
case involving the subject properties had already been 18, 1974, the law in force vesting jurisdiction upon said
heard and decided by a competent tribunal, the Military court was the Judiciary Act of 1948, the particular
Commission, and as such this Court is without provision of which was not affected one way or the
jurisdiction to pass upon anew the same subject other by any Presidential issuances under Martial Law.
matter. (pp. 30-31, rollo, emphasis supplied)

64 | P a g e
The Military Commission is not vested with jurisdiction considering the use of an unlicensed firearm simply as
over the crime of estafa. an aggravating circumstance in murder or homicide.
R.A. 8294 took effect on July 6, 1997. The crime
involved in the case at bench was committed on May
Arts. 21-24: Penalties in General 5, 1991. In view of the amendments introduced by
Republic Act 8294 to Presidential Decree 1866,
Retroactive Effect of Penal Laws separate prosecutions for homicide and illegal
possession are no longer in order. Instead, illegal
People v. Evina (G.R. No. 124830-310) possession of firearms is merely to be taken as an
aggravating circumstance in the homicide case.

Facts: Issue:
Whether or not the accused can be rightfully
Gerardo Evina was found guilty by the convicted of the crime of illegal possession of firearms
Regional Trial Court of Tacloban City (Branch 9) of two separately from the crime of homicide under RA 8294
counts of simple rape and sentenced to suffer the (amending PD 1866).
penalty of reclusion perpetua for each count. The
crime of rape was committed on November 3, 1991 Decision:
and on November 7, 1991, in the City of Tacloban No. As a general rule, penal laws will
against Marites Cacharo while she was sleeping in her generally have prospective application except where
bedroom. Evina tied Marites’s hands with a big the new law will be advantageous to the accused. In
handkerchief and poked a knife at her. This special this case R.A. 8294 will spare accused-appellant
aggravating circumstance of the use of a weapon and Lazaro from a separate conviction for the crime of
the aggravating circumstance of dwelling were both illegal possession of firearm. Accordingly, said law
proven during the trial. However, these were not should be given retroactive application.
alleged in the information.
Accordingly, accused-appellant Lazaro should
Issue: be spared from a separate conviction for the crime of
Illegal Possession of Firearms, which is the subject of
Whether or not the aggravating circumstances the present review. Accused-appellant Lazaro was
be considered in fixing the penalty? hereby acquitted of the said crime and the case was
dismissed.
Decision:

The aggravating circumstances cannot be Heide Olarte-Congson


considered in fixing the penalty because they were not 2007-0316
alleged in the information as mandated by Rule 110,
Sections 8 and 9 of the Revised Rules of Criminal
Procedure. Although the crimes charged were People v. Pacifador (G.R. No. 139405)
committed before the effectivity of the said rule, Facts:
nevertheless, the same should be applied retroactively On October 27, 1988, Arturo F. Pacificador
being favorable to the appellant. Although the then Chairman of the Board of the National Shipyard
aggravating circumstances in question cannot be and Steel Corporation, a government-owned
appreciated for the purpose of fixing a heavier penalty corporation, and therefore, a public officer was
in this case, they should, however, be considered as charged before the Sandiganbayan with the crime of
bases for the award of exemplary damages, violation of Republic Act No. 3019, as amended,
conformably to current jurisprudence. otherwise known as the Anti-Graft and Corrupt
Practices Act. The crime was committed from
December 6, 1975 to January 6, 1976, in Metro Manila
People v. Lazaro (G.R. No. 112090) by Pacificador. After his arraignment, the respondent
filed a Motion to Dismiss on the ground of prescription
of the offense. Sandiganbayan on November 10, 1998
Facts: dismissed the Information against the respondent on
Lazaro was charged, tried and convicted for the ground of prescription. The Urgent Motion for
two separate crimes of illegal possession of Reconsideration of the Solicitor General was denied by
firearms/ammunition and homicide under Section 1 of the Sandiganbayan. Republic Act No. 3019 provides
P.D. No. 1866 which was the governing law at the time for its own prescriptive period. Section 11 of R.A. No.
the crime was committed in 1991. The two separate 3019, as amended by B.P. Blg. 195, provides that the
cases, Criminal Case No. 91-3487 (for homicide) and offenses committed under the said statute shall
Criminal Case No. 91-3483 (for illegal possession of prescribe in fifteen (15) years. It appears however, that
firearm) were not tried jointly, although filed in the prior to the amendment of Section 11 of R.A. No. 3019
same trial court. Republic Act No. 8294 has since by B.P. Blg. 195 which was approved on March 16,
amended P.D. No. 1866 by reducing the penalties for 1982, the prescriptive period for offenses punishable
simple and aggravated forms of illegal possession and under the said statute was only ten (10) years.

65 | P a g e
The trial court convicted Sta. Catalina for the
Issue: crime of Estafa. Aggrieved, he appealed the decision
Whether or not the crime had not yet of the trial court before the Court of Appeals. However,
prescribed as the special law governing the same have the public prosecutor filed a manifestation stating that
been amended increasing the prescriptive period from Ballecer is no longer interested in pursuing his
ten (10) to fifteen (1) years. complaint and the case should be decided based on
Ballecer’s Affidavit of Desistance. The Court of
Decision: Appeals rendered a Decision affirming the judgment of
The longer prescriptive period of fifteen (15) conviction by the trial court. Hence, this instant
years, as provided in Section 11 of R.A. No. 3019 as petition.
amended by B.P. Blg. 195, does not apply in this case
for the reason that the amendment, not being Issue:
favorable to the accused, cannot be given retroactive
effect. Hence the crime prescribed on January 6, 1986 Whether or not the Affidavit of Desistance
or ten (10) years from January 6, 1976. The crime had executed by Ballacer will justify the dismissal of the
already prescribed when the Information in this case action?
was filed with the Sandiganbayan on October 27,
1988.It bears emphasis, as held in a number of cases Decision:
that in the interpretation of the law on prescription of
crimes, that which is more favorable to the accused is The Supreme Court held that an Affidavit of
to be adopted. The said legal principle takes into Desistance is not a ground for the dismissal of an
account the nature of the law on prescription of crimes action, once the action has been instituted in court. In
which is an act of amnesty and liberality on the part of the case at bar, Ballecer made the so-called pardon of
the state in favor of the offender. Sta. Catalina after the institution of the action, almost
two years after the trial court had rendered its decision.
Pardon by Offended Party
The court attaches no persuasive value to a
Sta. Catalina v. People (G.R. No. 167805) desistance especially when executed as an
afterthought. It would be a dangerous rule to reject the
testimony taken before the court of justice simply
Facts: because the witness who had given it later on changed
his mind for one reason or another. Such a rule will
Lorenzo Ballecer entered into a joint business make a solemn trial a mockery and place the
venture with Arnold Sta. Catalina involving importation investigation at the mercy of unscrupulous witnesses.
of Jute sacks from China. Petitioner told that he had a
ready buyer in the Philippines which was willing to buy
the jute sacks at P12.25 per piece. Convinced,
Ballecer ordered one container to Sta. Catalina.

Thereafter, Ballecer and Sta. Catalina Balderama v. People (G.R. No. 147578-85)
proceeded to Citytrust Bank to open a letter of credit.
They were required to post a marginal deposit
amounting to P100, 000. The two went to United Facts:
Coconut Planters Bank to encash a check. After the
encashment, they went back to Citytrust but arrived Rolando Balderama and Rolando Nagal are
after banking hours. Sta. Catalina suggested that the employees of the Field Enforcement Division of LTO.
money be deposited in his account which Ballecer On the other hand, Juan Armamento is the operator of
agreed. SJ Taxi. On July 14, 1992, the team of Flying Squad
flagged down one taxi owned by Armamento. They
Few days after, while preparing the supporting impounded the taxi on the ground that its meter was
documents for the letter of credit, Ballacer found that defective, however, upon inspection and testing by the
there was an overpricing on the cost of the jute sacks. LTO the results showed that the meter was functioning
Realizing that his business venture was losing normally.
proposition he asked Sta. Catalina to return the P100,
000, however, the latter failed to return the money Feeling aggrieved, Armamento filed a
despite repeated verbal and formal demands made by complaint for Bribery and violation of Anti-Graft and
the former. Corrupt Practices Act before the Ombudsman. He
alleged that prior to the impounding of his taxi, the four
Sta. Catalina as defense alleged that there LTO officers had been collecting protection money
was no misappropriation of the money. He further from him in exchange of non-apprehension and non-
claimed that the said money was spent and used for impounding of his vehicles.
the office expenses, salaries and other expenses of
the office which both of the occupy. Eventually, the Office of the Ombudsman filed
with the Sandiganbayan nine (9) Information for

66 | P a g e
violation of Direct Bribery and Anti-Graft and Corrupt
Practices Act. During the pendency of the action, On the other hand, the accused denied such
accused De Jesus died. The cases against him were accusations. He contended that he could not have
dismissed but the hearing proceeded against raped Maricar because he was always in the office. He
Balderama, Nagal and Lubrica. claimed that it was impossible for him to rape his
daughter because there were other people in the
The Sandiganbayan rendered its Decision, house. He further argued that had he raped Maricar,
convicting Balderama, Nagal and Lubrica for the above then she would have not accompanied him to the
violation. They filed a motion for reconsideration but Paranaque Police Station to apply for police clearance.
were denied by the former. Hence, this instant petition.
The trial court rendered its Decision,
Issue: convicting Edgardo of the crime of Rape. Aggrieved,
he appealed his case before the Court of Appeals but
Whether or not Armamento’s affidavit of the latter affirmed the decision of the trial court.
recantation will result to the dismissal of the
complaint? Issue:

Whether or not the voluntary and due


Decision: execution of the Affidavit of Desistance by Maricar is a
ground for the dismissal of the complaint against
The Supreme Court held that the complaint Edgardo?
shall not be dismissed. A recantation or an affidavit of
desistance is viewed with suspicion and reservation.
The court looks with disfavor upon retractions of Decision:
testimonies previously given in court. It is settled that
an affidavit of desistance made by a witness after The Supreme Court held that by itself, an
conviction of the accused is not reliable, and deserves Affidavit of Desistance is not a ground for the dismissal
only scant attention. of an action, once the action has been instituted in
court. A private complainant loses the right or absolute
The rationale for the rule is obvious: affidavits privilege to decide whether the rape charge should
of retraction can easily be secured from witnesses, proceed, because the case was already filed and must
usually through intimidation or for a monetary therefore continue to be heard by the trial court.
consideration. Only when there exist special
circumstances in the case which when coupled with The court attaches no persuasive value to a
the retraction raise doubts as to the truth of the desistance, especially when executed afterthought.
testimony or statement given, can retractions be The unreliable character of this document is shown by
considered and upheld. the fact that it is quite incredible that a victim, after
going through the trouble of having the accused
In this case, there is indubitably nothing in the affidavit arrested by the police, enduring the humiliation of a
which creates doubts on the guilt of the accused physical examination of her private parts and
Balderama and Nagal. recounting her anguish in detail, will suddenly turn
around and declare that she is no longer interested in
pursuing the case.
People v. Dimaano (G.R. No. 168168)
In the case at bar, Maricar repudiated the
affidavit of desistance in open court by stating that no
Facts: lawyer assisted her when she affixed her signature
and had shown her resolve to continue with the
Maricar Dimaano is the daughter of the prosecution of the cases.Arts. 25-45: Penalties
accused Edgardo Dimaano. A complaint was filed by
Maricar charging Edgardo with two counts of Rape and Reclusion Perpetua
one count of attempted rape. Maricar alleged that she
was only 10 years old when her father Edgardo started People v. Novio (G.R. No. 139332)
sexually abusing her. It was only on November of 1995
that she confided the sexual abuses to her mother.
The last sexual assault happened in the afternoon of Facts:
January 1, 1996.
On September 23, 1994, 13-year-old Maricel
Maricar and her mother went to Camp Crame B. Talisay, together with her minor brothers Jun and
upon the advised of a relative. The Medico-Legal Joey slept side-by-side at their store. Their parents
Officer at the PNP Crime Laboratory examined the were caretakers of a beach house and needed to
complainant and found her to have suffered deep sleep there at that time. At 3:00 in the morning, a
healed hymenal lacerations and was in a non-virgin ticklish sensation and stabs of pain in her vagina
state. awakened Maricel. When she woke up, she saw the

67 | P a g e
accused, Noli Novio, naked on top of her. Her duster case. The Court is not impervious to Article 70 of the
was rolled up to her neck and her panty has already Revised Penal Code which pertinently provides that, in
been removed by the accused. Noel Novio was able applying the so-called “three-fold rule,” i.e., that
to penetrate his penis inside Maricel’s Vagina. “(w)hen the culprit has to serve two or more
penalties, . . . the maximum duration of the convict’s
Meanwhile, Maricel’s parents were awakened sentence shall not be more than three-fold the length
by their neighbor and reported to them that a man was of time corresponding to the most severe of the
inside their store. Nenita, Maricel’s mother penalties imposed upon him” – “the duration of
immediately got hold of her bolo and flashlight and perpetual penalties (penal perpetua) shall be
proceeded to their store. Nenita saw a man’s sandals computed at thirty years.” The imputation of a thirty-
at the doorstep. Nenita knocked and called out to year duration to reclusion perpetua in Article 70 is, as
Maricel and ordered her to open the door. Despite this Court recently held, “only to serve as the basis for
repeated demands to open the door, Maricel was not determining the convict’s eligibility for pardon or for the
able to do so. Nenita was able to forcibly open the application of the three-fold rule in the service of
door and beamed the flashlight to Maricel and saw multiple penalties.
Noel Novio on top of her. The accused was holding
the hands of Maricel with his left hand and covered her People v. Zacarias (G.R. No. 138990)
mouth with his right hand. Nenita mounted to hack
Novio with her bolo but the accused immediately took
his jogging pants and ran away leaving his shirt, wallet, Facts:
underwear and sandals. Nenita immediately reported Sergio Pelicano, Sr., on direct examination,
the incident to the barangay and went to the police for testified that on June 23, 1993 at about 12:30 in the
investigation. Maricel submitted herself to medical morning while waiting for his son, he heard a
examination right after the incident. commotion outside his house. When he looked
outside, he saw Christopher Sacay, son of his long-
Noli Novio denied the allegations and argued time friend, being chased by Sammy Zacarias, Rodel
that Maricel and him are sweethearts. The trial court Zacarias, Wally Ticalo and Rene Matugas. The boy
found the accused guilty beyond reasonable doubt for ran towards the Seventh Day Adventist Church.
the crime of rape and sentenced him to 30 years of Pelicano followed the group and when he was only
reclusion perpetua and to indemnify the victim the sum about 10 meters away from the four men, he saw
of Fifty Thousand (P50,000.00) pesos without Rodel Zacarias hold the victim while the rest took turns
subsidiary imprisonment in case of insolvency and to in stabbing and hacking the boy.However, Ticalo
pay the cost of these  proceedings. claimed that on the day of the said incident, he was
Issue: working in the farm and had a drinking spree with the
owner of the farm until 10:00 in the evening. The
Whether or not the trial court was correct in owner of the farm even contended that the town where
imposing the proper penalty for the crime of rape? his farm was was far distant from the town where the
stabbing incident took place. The trial court sentenced
Decision: Ticalo to serve the penalty of reclusion perpetua for
the death of Christopher Sacay
No, the Supreme Court held that the penalty
imposed by the trial court is void. Under Article 335 of Issue:
the Revised Penal Code, as amended by Republic Act Whether or not the court a quo gravely erred in
7659, the prescribed penalty for simple rape is finding Ticalo guilty of the crime charged?
reclusion perpetua. However, the trial court sentenced
the appellant to thirty years of reclusion perpetua. The Decision:
penalty imposed by the trial court is void.Although No, the Supreme Court is not convinced with
under Article 27 of the Revised Penal Code as the contention of Ticalo, however, a word, in passing,
amended by Republic 7659, reclusion perpetua has a about the manner the trial court imposed the penalty.
range of twenty years and one day to forty years, by In the scales of penalties under the Revised Penal
nature, the penalty remains a single and indivisible Code, reclusion perpetua is the penalty immediately
penalty. It cannot be divided into periods or equal higher than reclusion temporal which has a duration of
portions. If the law prescribes reclusion perpetua as a twelve years and one day to twenty years. The
single and indivisible penalty for a felony, the trial court minimum range of reclusion perpetua should then, by
is mandated to impose said penalty, absent any necessary implication, start at 20 years and 1 day
privileged mitigating circumstances conformably with while the maximum thereunder could be co-extensive
Article 63 of the Revised Penal Code. The trial court is with the rest of the natural life of the offender. Article
not authorized to vary the penalty provided for by law 70, however, provides that the maximum period in
either in the character or the extent of punishment regard to the service of sentence shall not exceed 40
inflicted. years. Reclusion perpetua remains to be an indivisible
penalty and, when it is the prescribed penalty, should
There was no need for the trial court to specify be imposed in its entirety, i.e., reclusion perpetuasans
the duration of thirty years of reclusion perpetua a fixed period for its duration, regardless of any
whenever it is imposed as a penalty in any proper mitigating or aggravating circumstance that may have

68 | P a g e
attended the commission of the crime. In prescribing direction, carrying a thin, bloodied knife. Accused
the penalty of reclusion perpetua,its duration in years, Latupan entered the house of Ceferino and started
in fine, need not be specified. chasing Ceferino's wife, who was able to run.

People v. Ramirez (G.R. No. 138261) The house of Emilio Asuncion known as “Emy”
was 100 meters from Ceferino's house. He reached his
house and found his wife, Lilia, dead on the ground
Facts: with several stab wounds on her body and his 3
children (Leo, Jaime, and Jose) wounded. Doctors
On May 23, 1993, at 7:30 in the evening in treated the injuries of Leo and Jaime, However, Jose
Bgy. San Jose, Ormoc City, Montano Banez, while was transferred to another hospital due to seriousness
strolling in the plaza, saw the victim Jonathan ‘Jojo” of his wounds. Jose was declared dead on arrival.
Alkuino. Since Jojo was a former resident of the
barangay, Banez invited him to have a drinking spree On May 25, 1993, at arraignment, accused
in the nearby store. The two sat side-by-side and were pleaded not guilty to the charge of frustrated murder.
exchanging stories when Pedro Ramirez suddenly During the pre-trial conference of the four cases
came up to them. Ramirez hit Jojo on the right side of (Criminal Case No. 379-T, Criminal Case No. 380-T,
his body just below his ribs. Jojo was immediately Criminal Case No. 381-T, Criminal Case No. 382-T),
brought to the hospital and was still alive on arrival but accused offered to change his plea of not guilty to
died the next day due to hypovolemic shock or guilty of the complex crime of double murder and
massive blood loss. The trial court found Ramirez frustrated murder.
guilty of murder and sentencing him to "suffer
imprisonment of forty (40) years reclusion perpetua.” On August 25, 1993, the trial court rendered a
decision finding the accused Latupan guilty beyond
Issue: reasonable doubt of the complex offense of Double
Murder and sentenced him to suffer life imprisonment
Whether or not the trial court was correct in and to indemnify the heirs.
specifying the length of imprisonment in the penalty of
Reclusion Perpetua? Issue:

Decision: Whether or not trial court erred in convicting


accused-appellant of the complex crime of double
No, the Supreme Court held that in sentencing murder?
appellant "to suffer imprisonment of forty (40) years
reclusion perpetua." There was no justification or need
for the trial court to specify the length of imprisonment,
because reclusion perpetua is an indivisible penalty.
The significance of this fundamental principle was laid
down by the Court in People v. Diquit, "Since reclusion
perpetua is an indivisible penalty, it has no minimum,
medium or maximum periods. It is imposed in its
entirety regardless of any mitigating or aggravating
circumstances that may have attended the commission
of the crime. (Art. 63, Revised Penal Code) Reclusion
perpetua is imprisonment for life but the person
sentenced to suffer it shall be pardoned after
undergoing the penalty for thirty (30) years, unless by
reason of his conduct or some other serious cause, he
shall be considered by the Chief Executive as
unworthy of pardon (Art. 27, Revised Penal Code)."

Arts. 46-77: Application of Penalties

Complex Crime

People v. Latupan (G.R. Nos. 112453-56)

Facts:

On April 29, 1991, Ceferino Dagulo heard


shouts of a woman and a child coming from the north.
He saw accused Gerardo Latupan walking in his

69 | P a g e
continuing acts on the part of the accused, not by
Decision: different and separate sets of shots, moved by one
impulse and should therefore be treated as one crime
The trial court, erred in convicting accused- though the series of shots killed more than one victim;"
appellant of the "complex crime of double murder" and and that only one information for multiple murder
separate offenses of serious physical injuries. Article should be filed, to obviate the necessity of trying five
48 of the Revised Penal Code provides: "When a cases instead of one."
single act constitutes two or more grave or less grave
felonies or when an offense is a necessary means for
committing the other, the penalty for the most serious
crime shall be imposed, the same to be applied in its
maximum period."
Issue:
The instant case does not fall under any of the
two mentioned instances when a complex crime is Whether or not there should be one
committed. The killing of Lilia Asuncion and Jose information, either for the complex crime of murder and
Asuncion and the wounding of Jaime and Leo frustrated murder or for the complex crime of robbery
Asuncion resulted not from a single act but from with multiple homicide and frustrated homicide or
several and distinct acts of stabbing. "Where the death should the five indictments remain as they are?
of two persons does not result from a single act but
from two different shots, two separate murders, and Decision:
not a complex crime, are committed."
Four separate crimes of murder and a
Thus, accused-appellant is liable, not for a frustrated murder result from the firing of several shots
complex crime of double murder, but for two separate at five victims. The crimes are not complex. Five
counts of murder, and separate counts of physical information should be filed. There is a complex crime
injuries. where one shot from a gun results in the death of two
or more persons, or where one stabbed another and
People v. Pineda (G.R. No. L-26222) the weapon pierced the latter’s body and wounded
another, or where a person plant’s a bomb in an
airplane and the bomb explodes, with the result that a
Facts: number of persons are killed. When various victicms
expire from separate shots, such acts constitute
On July 29, 1965, the occupants of the home separate and distinct crimes.
of the spouses Teofilo Mendoza and Valeriana
Bontilao de Mendoza in Pugaan City of Iligan, were People v. Sanidad (G.R. No. 146099)
asleep. It was then that guns (rifle, caliber 22)
and paliuntod(homemade gun) were fired in rapid
succession from outside the house. Teofilo Mendoza Facts:
fell dead. Thereafter, defendants below destroyed the
door of the house, entered therein, and let loose On 16 January 1999, Marlon Tugadi, Jun
several shots killing Neceforo Mendoza, all minor Quipay, Raymund Fontanilla, Rolando Tugadi, Pepito
children of the couple and wounding Valeriana Tugadi, Delfin Tadeo, Ricardo Tadeo, Edwin Tumalip,
Bontilao de Mendoza. Bobby Velasquez and Dennis Balueg left Budac,
Tagum, Abra, on board a passenger jeepney driven by
Two of the three defendants in the five criminal Delfin Tadeo to attend a barangay fiesta in the town of
cases: Tomas Narbasa and Tambak Alindo, moved Lagangilang, Abra. When they arrived they joined the
for a consolidation thereof "into one (1) criminal case." residents in a drinking spree that lasted the following
Their plea is that "said cases arose out of the same morning.
incident and motivated by one impulse."
Accused-appellants Jimmel Sanidad, Ponce
Respondent Judge directed the City Fiscal to Manuel alias Pambong and several other residents of
unify all the five criminal cases, and to file one single Lagangilang joined them in drinking. Marlon Tugadi
information in Case 1246. He also ordered that the and accused Jimmel Sanidad were drinking buddies
other four cases, Nos. 1247, 1248, 1249 and 1250 "be and members of the CAFGU before then.
dropped from the docket."
The City Fiscal moved for reconsideration On 17 January 1999, Jimmel Sanidad and his
upon the ground that "more than one gun was used, companions finished drinking and left. Shortly after, the
more than one shot was fired and more than one group of Marlon Tugadi also stopped drinking and
victim was killed." Thereafter, the defense opposed. headed home for Budac, Tagum, Abra, boarding the
same jeepney driven by Delfin Tadeo.
On May 31, 1966, respondent Judge denied
the motion to reconsider. He took the position that the As the jeepney moved closer, the accused in a
acts complained of "stemmed out of a series of classic case of ambuscade suddenly and without

70 | P a g e
warning unleashed a volley of shots at the Delito Continuado
jeepney.Miraculously, almost all of its passengers, with
the exception of Rolando Tugadi, survived the ambush Ramiscal v. Sandiganbayan (G.R. Nos. 169727-28)
and suffered only minor injuries.

Apparently shaken and dazed by their Facts:


terrifying ordeal, the victims hid in a culvert on the side
of the road and did not come out until the police Pursuant to the recommendation of the Senate
arrived at the scene. Apparently shaken and dazed by Blue Ribbon Committee to “prosecute and/or cause
their terrifying ordeal, the victims hid on the side of the the prosecution of Gen. Jose Ramiscal Jr. (Ret), past
road and did not come out until the police arrived at AFP-RSBS President, who had signed the
the scene. unregistered deeds of sale covering the acquisition of
certain parcels of land,” Ombudsman Investigators
An Information for murder with multiple Ricardo Sullano, Rodil Buenaventura and Anatolio
attempted murder and malicious mischief was filed Alejandrino of the Office of the Deputy Ombudsman for
against Jimmel Sanidad, Ponce Manuel the Military conducted a fact-finding investigation. They
alias Pambong, John Doe and Peter Doe. The defense executed a Joint Affidavit-Complaint, stating that
of the accused rested on bare denial and alibi. based on their findings, the following may be charged
The trial court disregarded the defense with falsification of public documents and violation of
interposed by the accused and convicted them of the Section 3(e) and (g) of Republic Act (R.A.) No. 3019:
complex crime of murder and multiple attempted petitioner B/Gen. Jose Ramiscal, Jr., former AFP-
murder, and sentenced them to death. RSBS president; Atty. Meinrado Enrique Bello, Head
of the AFP-RSBS Legal Department in charge of Land
Issue: Acquisition; Capt. Perfecto Enrique Quilicot, AFP-
RSBS Project Officer, Tanauan, Batangas, Land
Whether or not accused-appellants are guilty Acquisition; and Notaries Public Alfredo Nasser and
of complex crime of murder and multiple attempted Manuel Satuito.
murder and imposing upon then the supreme penalty
of death? The matter was further looked into by a panel
of Ombudsman Investigators, which issued on March
Decision: 30, 2001 a Joint Resolution finding probable cause to
file the corresponding Informations for 148 counts of
The Supreme Court fully agreed with the lower violation of Article 315, in relation to Article 171,
court that the instant case comes within the purview of paragraph 4 of the Revised Penal Code, and Section 3
Art. 48 of The Revised Penal Code which, speaking of (e) of R.A. No. 3019 against Meinrado Bello and Atty.
complex crimes, provides that when "a single act Manuel Satuito. However, it was likewise
constitutes two or more grave or less grave felonies, or recommended that the complaint against petitioner be
when an offense is a necessary means for committing dismissed, without prejudice to a thorough fact-finding
the other, the penalty for the most serious crime shall investigation on his liability.
be imposed in its maximum period." In a complex
crime, although two or more crimes are actually After conducting clarificatory hearings, the
committed, they constitute only one crime in the eyes investigating panel issued a Memorandum,
of the law as well as in the conscience of the offender. recommending to the Ombudsman that petitioner be
charged with 148 counts of estafa through falsification
Although several independent acts were of public documents, and one count violation of
performed by the accused in firing separate shots from Section 3(e) of R.A. No. 3019. The Ombudsman
their individual firearms, it was not possible to approved the recommendation of the Panel of
determine who among them actually killed victim Prosecutors. Petitioner and his co-accused filed their
Rolando Tugadi. Moreover, there is no evidence that respective Motions for Reconsideration of the
accused-appellants intended to fire at each and every investigating panel’s June 15, 2004 Memorandum.The
one of the victims separately and distinctly from each Sandiganbayan denied the motion. It likewise denied
other. On the contrary, the evidence clearly shows a the motion for the consolidation of the cases,
single criminal impulse to kill Marlon Tugadi's group as considering that the other cases filed were pending in
a whole. Thus, one of accused-appellants exclaimed in its other divisions.
frustration after the ambush: "My gosh, we were not
able to kill all of them." Where a conspiracy animates Petitioner filed a motion for reconsideration of
several persons with a single purpose, their individual the resolution which was denied again by the
acts done in pursuance of that purpose are looked Sandiganbayan. Motion to Quash was likewise denied.
upon as a single act, the act of execution, giving rise to
a single complex offense. Issue:

Whether or not only one information for estafa


should be filed for all these cases?

71 | P a g e
Held: arraignment on November 13, 1992. The
Sandiganbayan (First Division) denied the motion to
The petition has no merit.The Sandiganbayan, defer the arraignment. Petitioner filed a motion for a bill
for its part, sustained the contention of respondents of particulars. According to petitioner, unless she was
and ruled that the determination of (a) the charge/s furnished with the names and identities of the aliens,
and the person/s against whom the charge is filed are she could not properly plead and prepare for trial.
addressed to the sound discretion of the Prosecutors
based on the facts before them; and (b) the crimes On March 14, 1993, the Sandiganbayan (First
committed by petitioner are separate, and not a single Division) promulgated a resolution, admitting the 32
crime consisting of series of acts arising from a single Amended Informations and ordering petitioner to post
criminal resolution. the corresponding bail bonds. Hence, the filing of the
instant petition.
When required to comment on the motion of
petitioner and his co- accused for a consolidation of Issue:
the charges filed against them before the
Sandiganbayan, the Special Prosecutor objected Whether or not the 32 Amended Informations
thereto, insisting that there were as many crimes may be admitted?
committed by the accused as there were sales
contracts forged by them.  Held:

Indeed, the determination of what charges to The petition is denied.


file and who are to be charged are matters addressed
to the discretion of the Ombudsman, including the The Court find that, technically, there was only
matter of whether the crime perpetrated by petitioner one crime that was committed in petitioner's case, and
and his co-accused under the Informations pending in hence, there should only be one information to be file
the Divisions of the Sandiganbayan constitute delito against her.The 32 Amended Informations charge
continuado or classified as concurso de delitos; what is known as delito continuado or "continued
orinvolve separate crimes under the category of crime" and sometimes referred to as "continuous
concurso real delito involve factual issues. Such crime."
factual issues should be resolved after trial on the
merits, and not in this case. The Court is being tasked The original information charged petitioner
to determine whether the several sales contracts with performing a single criminal act - that of her
executed by petitioner and his co-accused were set approving the application for legalization of aliens not
afoot or triggered by a single impulse and operated by qualified under the law to enjoy such privilege. The
an uninterrupted force however long a time it may original information also averred that the criminal act :
occupy, which, however, is a matter best left to the (i) committed by petitioner was in violation of a law -
determination of the trial court, in this case, the Executive Order No. 324 dated
Sandiganbayan. April 13, 1988, (ii) caused an undue injury to one
offended party, the Government, and (iii) was done on
Santiago v. Garchitorena (G.R. No. 109266) a single day, i.e., on or about October 17, 1988. The
32 Amended Informations reproduced verbatim the
allegation of the original information, except that
Facts: instead of the word "aliens" in the original information
each amended information states the name of the
On May 1, 1991, petitioner was charged in individual whose stay was legalized.
Criminal Case No. 16698 of the Sandiganbayan with
violation of Section 3(e) of R.A. No. 3019, as The 32 Amended Informations aver that the
amended, otherwise known as the Anti-Graft and offenses were committed on the same period of time,
Corrupt Practices Act, allegedly committed by her i.e., on or about October 17, 1988. The strong
favoring "unqualified" aliens with the benefits of the probability even exists that the approval of the
Alien Legalization Program. application or the legalization of the stay of the 32
aliens was done by a single stroke of the pen, as when
On May 24, 1991, petitioner filed with us a the approval was embodied in the same document.
petition for certiorari and prohibition, docketed as G.R. Likewise, the public prosecutors manifested at the
No. 99289-99290 (Santiago v. Vasquez, 205 SCRA hearing the motion for a bill of particulars that the
162 [1992]), to enjoin the Sandiganbayan from Government suffered a single harm or injury.
proceeding with Criminal Case No. 16698 on the
ground that said case was intended solely to harass The Resolution dated March 3, 1993 in
her as she was then a presidential candidate. The Criminal Case No. 16698 of the Sandiganbayan (First
petition was dismissed on January 13, 1992. Division) is affirmed and its Resolution dated March
11, 1993 in Criminal Case No. 16698 is modified in the
On October 27, 1992, the Sandiganbayan sense that the Office of the Special Prosecutor of the
(First Division), of which Presiding Justice Office of the Ombudsman is directed to consolidate the
Garchitorena is a member, set the criminal case for 32 Amended Informations (Criminal Cases Nos. 18371
72 | P a g e
to 18402) into one information charging only one not recovered, and being a habitual delinquent, the
offense under the original case number, i.e., No. additional penalty of two years, four months and one
16698. day of prision correccional.

Habitual Delinquency The accused plead guilty of the crime imputed


to him in the information filed against him. It was
People v. Espina (G.R. No. 43556) alleged in the information that he is a habitual
delinquent, having been convicted by final judgments
of the crimes of theft and qualified theft on January 4,
Facts: 1933 and November 18, 1935 respectively, the date of
his last release being January 10, 1936.
The appellant was charged in the lower court
with the crime of theft of articles valued at P 585.15 Due to his admission of guilt of the crime
and, having pleaded guilty, was sentenced to six imputed to him in the information, it is well settled in
months and one day of prision correccional and, being this jurisdiction that when one pleads guilty of the
a habitual delinquent, to an additional penalty of two crime imputed to him in the information, it is
years, four months and one day of prision understood that he admits all material facts alleged
correccional. The appellant is a recidivist and plead therein, not excluding those alleging his former
guilty to the crime of theft. He is also a habitual convictions of other crimes.
delinquent, this being his third conviction.
Issue:
Issue:
Whether or not the circumstance of recidivism
Whether or not recidivism, as inherent in can be and must be twice taken into consideration, first
habitual delinquency, should still be taken into as an aggravating circumstance, and second as a
consideration in fixing the principal penalty? qualifying circumstance or one inherent in habitual
delinquency?
Decision:
Decision:
Yes, recidivism should still be taken into
consideration in fixing the principal penalty even Yes, recidivism can be considered as an
though it is inherent in habitual delinquency. The aggravating circumstance in determining the principal
appellant in this case is a habitual delinquent, this penalty and as a qualifying circumstance in habitual
being his third conviction. Recidivism, although delinquency.
inherent in habitual delinquency, should still be As to the principal penalty, there is the rule
considered in fixing the principal penalty. There is no that in cases in which the penalty prescribed by law
doubt that the purpose of the law in imposing contains three periods, the courts must take into
additional penalty on a habitual delinquent is to punish consideration, in the application of said penalty, the
him more severely. However, the result would be aggravating or mitigating circumstances established at
otherwise if, for imposing the additional penalty, the trial if they do not appear to be compensated by
recidivism could not be considered as an aggravating other circumstance. It is reiterated in People vs.
circumstance in fixing the principal penalty. In the Melendrez that the aggravating circumstance of
instant case, the mitigating circumstance of voluntary recidivism, even in cases of habitual delinquency,
plea of guilty is present. If the aggravating should be taken into consideration in the application of
circumstance of recidivism is not to be taken into the principal penalty in the corresponding period.
consideration for imposing the additional penalty for
habitual delinquency, the mitigating circumstance The proposition that if recidivism is considered
would require that the penalty prescribed by law be an inherent or qualifying circumstance of habitual
imposed in it minimum period. The imposition of the delinquency it should not be taken into account in the
additional penalty would make the penalty lighter, imposition of the principal penalty, seems to be
instead of more severe, contrary to the purpose of the untenable because it is based upon the erroneous
law. assumption that habitual delinquency is a crime. It is
simply a fact or circumstance which, if present in a
People v. De Jesus (G.R. No. 45198)
given case, gives rise to the imposition of the
additional penalties prescribed therein.
Facts:
As to the additional penalty, if we must rely
upon the spirit and letter of the law, we would say that
Basilio de Jesus y Javier was convicted by the
the purpose of the latter in establishing it was to
Court of First Instance of Manila of the crime of theft of
prevent those for the second time or more commit the
an umbrella and a buri hat valued at P 2.65 committed,
crimes from relapsing thereafter at least during the
according to the information, on April 28, 1936. He was
period fixed thereby. The lower court correctly ruled in
therein sentenced to one month and one day of
imposing the additional penalty.
arresto mayor with accessory penalties, to indemnify
Francis Liwanag the value of the umbrella which was
73 | P a g e
Pending appeal of his conviction, Bayotas died
on February 4, 1992 at the National Bilibid Hospital
Arts. 89-93: Total Extinction of Criminal Liability due to cardio respiratory arrest secondary to hepatic
encephalopathy secondary to hipato carcinoma gastric
Death of the Accused malingering.

De Guzman v. People (G.R. No. 154579) Consequently, the Supreme Court in its
Resolution of May 20, 1992 dismissed the criminal
aspect of the appeal. However, it required the Solicitor
Facts: General to file its comment with regard to Bayotas' civil
On February 8, 1995, in the City of Makati, liability arising from his commission of the offense
petitioner De Guzman, stole several pieces of jewelry charged.
valued at P4,600,000.00 belonging to one Jasmine
Gongora.The trial court rendered its decision finding In his comment, the Solicitor General
de Guzman guilty beyond reasonable doubt and expressed his view that the death of accused-appellant
imposed a penalty of imprisonment, as well as the did not extinguish his civil liability as a result of his
penalties accessory thereto. The Court further finds commission of the offense charged.
the accused De Guzman civilly liable and orders her to
pay the private offended party. On appeal, the CA Issue:
affirmed the conviction but reduced the award of
damages. During the appeal in the Supreme Court, on Whether or not the death of the accused
January 30, 2003, counsel for the petitioner filed a pending appeal of his conviction extinguish his civil
Manifestation informing the Court that the petitioner liability?
passed away on January 13, 2003.The death of the
petitioner resulted from a vehicular accident, as Decision:
indicated in the Certificate of Death attached thereto.
Death of the accused pending appeal of his
Issue: conviction extinguishes his criminal liability as well as
Whether or not the criminal and civil liability of the civil liability based solely thereon. As opined by
the petitioner is extinguished by reason of her death? Justice Regalado, in this regard, "the death of the
accused prior to final judgment terminates his criminal
Decision: liability and only  the civil liability directly arising from
Upon death of the accused pending appeal of and based solely on the offense committed, i.e., civil
his conviction, the criminal action is extinguished liability ex delicto  in senso strictiore."
inasmuch as there is no longer a defendant to stand as
the accused; the civil action instituted therein for Corollarily, the claim for civil liability survives
recovery of civil liability ex delicto is ipso facto notwithstanding the death of accused, if the same may
extinguished, grounded as it is on the also be predicated on a source of obligation other than
criminal.Although both the trial and the appellate delict. Article 1157 of the Civil Code enumerates these
courts found petitioner guilty beyond reasonable doubt, other sources of obligation from which the civil liability
she had the right to appeal her case to this Court of may arise as a result of the same act or omission:
last resort and challenge the findings of the two courts a) Law 
below. The judgment of conviction was pending review b) Contracts
until her untimely demise. It has, therefore, not yet c) Quasi-contracts
attained finality. Thus, pursuant to Article 89 of the d) . . .
Revised Penal Code, it is incumbent upon the Court to e) Quasi-delicts
dismiss the instant petition for review. The Court is
dismissing the case because there is no longer a need Where the civil liability survives, as explained in
to continue with the review of the appeal. The lower Number 2 above, an action for recovery therefor may
court’s decision has thus become ineffectual. be pursued but only by way of filing a separate civil
Ranvylle Albano action and subject to Section 1, Rule 111 of the 1985
2008-0052 Rules on Criminal Procedure as amended. This
separate civil action may be enforced either against
the executor/administrator or the estate of the
People v. Bayotas (G.R. No. 102007) accused, depending on the source of obligation upon
which the same is based as explained above.

Facts: Finally, the private offended party need not fear a


forfeiture of his right to file this separate civil action by
In Criminal Case No. C-3217 filed before prescription, in cases where during the prosecution of
Branch 16, RTC Roxas City, Rogelio Bayotas y the criminal action and prior to its extinction, the
Cordova was charged with Rape and eventually private-offended party instituted together therewith the
convicted thereof on June 19, 1991. civil action. In such case, the statute of limitations on
74 | P a g e
the civil liability is deemed interrupted during the Prescription of Offenses
pendency of the criminal case, conformably with
provisions of Article 1155 of the Civil Code, that should Panaguiton v. DOJ (G.R. No. 167571)
thereby avoid any apprehension on a possible
privation of right by prescription.
Facts:
Applying this set of rules to the case at bench,
we hold that the death of appellant Bayotas Cawili and his business associate Tongson
extinguished his criminal liability and the civil liability borrowed from Panaguiton (petitioner) sums amounting
based solely on the act complained of, i.e., rape. to 1,979,459. They issued checks signed by both of
Consequently, the appeal is hereby dismissed without them to Panaguiton but these were dishonored upon
qualification. presentation.Panaguiton made demands to pay but to
no avail. He formally filed a complaint on August 24,
People v. Abungan (G.R. No. 136843) 1995 for violating BP 22 before the City Prosecutor’s
Office.

Facts: Tongson moved to drop his name from the


case as his signatures were allegedly falsified. Case
On August 4, 1992, at Capulaan, Villasis, against him was dismissed but afterwards upon finding
Pangasinan, the accused conspiring, confederating that Tongson might have indeed signed the checks, the
and mutually helping one another, armed with long chief state prosecutor directed the city prosecutor to
firearms, attack, assault and shoot Camilo Dirilo, [Sr.] y conduct a reinvestigation.Tongson moved for
Pajarito, inflicting upon him wounds on the different reconsideration but denied.
parts of his body which directly caused his death.
In 1999 assistant prosecutor dismissed the
An Information, dated March 9, 1993, was filed complaint for the action has prescribed pursuant to Act
charging appellant Pedro Abungan, together with 3326, which provides for the prescriptive periods of
Randy Pascua and Ernesto Ragonton Jr. (both at statutes without their own (4 years for BP22). She
large), with murder. claims that the filing of the complaint on August 24,
1995 did not interrupt the running of the period as the
The trial court rendered a decision finding law refers to judicial and not administrative proceedings.
Abungan guilty beyond reasonable doubt of the crime
of murder. Issue:

During appeal, in a letter dated August 7, Whether or not the filing of the complaint in the
2000, however, Joselito A. Fajardo, assistant director prosecutor’s office tolled the prescriptive period?
of the Bureau of Corrections, informed the Court that
Appellant Abungan had died on July 19, 2000 at the Decision:
NBP Hospital.
Yes. Filing of the complaint in the prosecutor’s
Issue: office tolls the prescriptive period for violations of
BP22.When Act 3326 was passed into law, preliminary
Whether or not the criminal and civil liability of investigation of cases was done by the justices of
the appellant is extinguished by reason of her death? peace, and not by agents of the executive department
(i.e. prosecutors). Thus, the prevailing rule at that time
Decision: is that prescription is tolled once filed with the justice of
peace (a judicial process). However, since then, the
In the present case, it is clear that, following conduction of a preliminary investigation was moved to
the case of People vs.Bayotas, the death of appellant the function of the executive department.
extinguished his criminal liability. Moreover, because
he died during the pendency of the appeal and before Today, the term proceedings must be
the finality of the judgment against him, his civil liability understood to mean either executive or judicial
arising from the crime or delict (civil liability ex delicto) proceedings. With this interpretation, any type of
was also extinguished. It must be added, though, that investigation may ultimately lead to sufficiently toll
his civil liability may be based on sources of obligation prescription.
other than delict. For this reason, the victims may file a
separate civil action against his estate, as may be To rule otherwise would deprive the injured
warranted by law and procedural rules. party the right to obtain vindication on account of
Brian Bonifacio Dela Cruz delays not under his control. As seen in this case,
2007-0388 various conflicting opinions of the DOJ delayed his
cause. Aggrieved parties who do not sleep on their
right should not be allowed to suffer simply because of
circumstances beyond their control.

75 | P a g e
Recebido v. People (346 SCRA 881) this case is “prision correccional in its medium and
maximum periods and a fine of not more than 5,000.00
pesos.”
Facts:
Under the Revised Penal Code, said penalty is
Sometime in April 1985, Caridad Dorol a correctional penalty in the same way that the fine
mortgaged her property--an agricultural land located in imposed is categorized as correctional. Both the
Bacon, Sorsogon to her cousin Recebido.Dorol and penalty and fine being correctional, the offense shall
Recebido did not execute any mortgage document, but prescribe in ten years. The issue that the petitioner has
instead, the former gave to the latter a copy of the missed, however, is the reckoning point of the
Deed of Sale dated June 16, 1973 which was done by prescriptive period. The petitioner is of the impression
Juan Dorol (father of Caridad). that the ten-year prescriptive period necessarily started
at the time the crime was committed. This is inaccurate.
On September 9, 1990, Caridad Dorol went to the Under Article 91 of the Revised Penal Code, the period
house of Recebido to redeem such property, wherein of prescription shall “commence to run from the day on
Recebido refused to allow claiming that Dorol has which the crime is discovered by the offended party, the
already sold to him the land on 1979. Dorol, on the authorities, or their agents.
other hand, insisted that the transaction between them
was not a sale, but a mere mortgage. Caniza v. People (159 SCRA 16)

Caridad Dorol, then, went to the Office of the


Assessor in Sorsogon and verified the existence of a Facts:
file Deed of Sale dated August 13, 1979 in which she
knew that the property was already registered in OnMarch 20, 1974, Assistant City Fiscal of
Recebido’s name.A comparison of the specimen Manila filedan Information for falsification of
signatures of Caridad’s other documents and that in public documentsallegedly committed on Nov. 5,
the questioned Deed of Sale was done, and NBI 1968 by Caniza.
Document Examiner Antonio Magbojas found out that
in the latter’s signature was falsified. On May 24, 1974, Caniza filed Motion to
Quash sayingthat allegations in the information did
The Office of the Provincial Prosecutor of not constitute an offense, and that the information
Sorsogon filed the information indicting Recebido for contained averments which, if true, would constitute a
Falsification of Public Document with the Regional legal excuse or justification.TheTrial court granted
Trial Court of Sorsogon. Motion to Quash, dismissed case against Caniza.
Fiscal’s Motion for Reconsideration of this Order was
The trial court rendered the decision convicting the denied.
petitioner of the crime and sentenced to an
indeterminate penalty of one (1) year to three (3) years On June 13, 1979, a second
and six (6) months of prision correccional as maximum Informationwas filed charging Caniza with
and to pay a fine of Three Thousand (P3,000.00) substantially the same offense as that charged under
Pesos, with subsidiary imprisonment. the previous information.Caniza moved to quash this
second information on the grounds that 1) the offense
The defense of prescription was raised only during charged had already prescribed, 2)quashal of the first
the motion for reconsideration of the Court of Appeals. Information had been on the merits, 3)the allegations
of the second Information did not constitute and
offense.
Issue:
The judge issued an order denying the
Whether or not the crime charged had already motion to quash. He also denied Caniza’s motion for
prescribed at the time the information was filed? reconsideration.

Decision: Issue:
No. Prescription, although not invoked in the
trial, may, as in this case, be invoked on appeal. Hence, Whether or not the offense charged had
the failure to raise this defense in the motion to quash already prescribed?
the information does not give rise to the waiver of the
petitioner-accused to raise the same anytime thereafter Decision:
including during appeal.
No. 5 years, 4 months, and 16 days had
Nonetheless, we hold that the crime charged elapsed between November 5, 1968 (the date of
has not prescribed. The petitioner is correct in stating commission of the alleged offense) and March 20,
that whether or not the offense charged has already 1974 (date of filing the first information); 4 years, 2
prescribed when the information was filed would months and 12days had elapsed between April 3,
depend on the penalty imposable therefore, which in 1975 (date of denial by the trial court of the Fiscal’s

76 | P a g e
motion for reconsideration) and June 13, 1979 (date After a careful verification and
of filing of the second information). A total of 9 years, evaluation on the claims of the
6 months and 28 days had been consumed by the applicant, the Local Amnesty Board
time the second Information was filed in court. concluded that his activities were done
in the pursuit of his political beliefs. It,
Under Article 90, in relation with Article 172 of thus, recommended on 20 May 1998
the Revised Penal Code, the crime of falsification of the grant of his application for
public document committed by a private individual amnesty.
-the offense with which petitioner Cañiza is presently The Commission, in its
charged - prescribes in ten (10) years. In this respect, deliberation on the application on 22
Article 91 of the Revised Penal Code states further: October 1999, resolved to approve the
“Theperiod of prescription shall commence to run recommendation of the Local Amnesty
fromthe day on which the crime is discovered by Board.
theoffended party, the authorities, or their agents,
andshall be interrupted by the filing of the complaint The Office of the Solicitor
orinformation, andshall commence to run again when General, in its letter dated June 23,
such proceedings terminate without the accused 2000 to the National Amnesty
being convicted or acquitted,or are justifiably stopped Commission, requested information as
for any reason not imputable to him.” to whether or not a motion for
reconsideration was filed by any party,
and the action, if there was any, taken
Amnesty by the NAC.
In his reply dated June 28,
People v. Patriarcha (G.R. No. 135457) 2000, NAC Chairman Tadiar wrote,
among other things, that there has
been no motion for reconsideration
Facts: filed by any party.
On August 16, 1990, an Accused-appellant Jose N.
Information for murder was filed Patriarca, Jr. was granted amnesty
against Jose Patriarca, Jr., alias "Ka under Proclamation No. 724 on May
Django," "Carlos Narra", "Ka Jessie," 17, 1996.
et al., for killing Alfredo Arevalo.
Issue:
Accused-appellant Jose
Patriarca, Jr. was also charged with Whether or not the grant of
Murder for the killing of one Rudy de amnesty in favor of Jose Patriarca, Jr.
Borja and a certain Elmer Cadag - while the various criminal cases filed
under Informations docketed as against him were pending - shall
Criminal Cases Nos. 2665 and 2672, completely extinguished his criminal
respectively. liability?
On January 20, 1998, the Decision:
lower court rendered its decision
convicting the herein accused- Amnesty commonly denotes a
appellant. general pardon to rebels for their
treason or other high political
Thus, Accused-Appellant filed offenses, or the forgiveness which one
his appeal. However, while his appeal sovereign grant to the subjects of
was pending, he applied for amnesty another, who have offended, by some
under Proclamation No. 724 amending breach, the law of nations. Amnesty
Proclamation No. 347, dated March looks backward, and abolishes and
25, 1994, entitled "Granting Amnesty puts into oblivion, the offense itself; it
to Rebels, Insurgents, and All Other so overlooks and obliterates the
Persons Who Have or May Have offense with which he is charged, that
Committed Crimes Against Public the person released by amnesty
Order, Other Crimes Committed in stands before the law precisely as
Furtherance of Political Ends, and though he had committed no offense.
Violations of the Article of War, and
Creating a National Amnesty Paragraph 3 of Article 89 of
Commission." His application was the Revised Penal Code provides that
favorably granted by the National criminal liability is totally extinguished
Amnesty Board. by amnesty, which completely

77 | P a g e
extinguishes the penalty and all its [respondent] and her husband [,] of Vignette
effects. Superstore [,] approached [petitioner] and asked her if
they [could] borrow money to settle some obligations.
In the case of People vs. Having been convinced by them and because of the
Casido, the difference between close relationship of [respondent] to [petitioner], the
pardon and amnesty is given: latter lent the former her money. Thus, every month,
she was persuaded to release P100,000.00 to the
"Pardon is granted by the accused until the total amount reached P1,150,000.00.
Chief Executive and as such it is a
private act which must be pleaded and As security for the P1,150,000.00,
proved by the person pardoned, [respondent] gave [petitioner a open dated Hermosa
because the courts take no notice Savings Bank (HSLB) with the assurance that if the
thereof; while amnesty by entire amount is not paid within one (1) year,
Proclamation of the Chief Executive [petitioner] can deposit the check.
with the concurrence of Congress, is a
public act of which the courts should In June 1997, [petitioner] together with
take judicial notice. Pardon is granted Samson Ching demanded payment of the sums
to one after conviction; while amnesty [above-mentioned], but [respondent] refused to
is granted to classes of persons or acknowledge the indebtedness. Thus, on October 6,
communities who may be guilty of 1977, [petitioner] deposited all aforementioned checks
political offenses, generally before or in the bank of Samson Ching totaling P1,150,000.00
after the institution of the criminal since all the money given by her to [respondent] came
prosecution and sometimes after from Samson Ching. The checks were all returned for
conviction. Pardon looks forward and having been drawn against insufficient funds (DAIF).
relieves the offender from the A verbal and written demand was made upon
consequences of an offense of which [respondent] to pay the amount represented by the
he has been convicted, that is, it bounced checks, but [to] no avail. Hence, a complaint
abolishes or forgives the punishment, for violation of BP 22 was filed against the
and for that reason it does 'not work [respondent].
the restoration of the rights to hold
public office, or the right of suffrage, Issue:
unless such rights be expressly
restored by the terms of the pardon,' Whether respondent remains civilly liable to
and it 'in no case exempts the culprit her for the sum ofP1,150,000. In this connection, she
from the payment of the civil indemnity asserts that respondent obtained loans from her in the
imposed upon him by the sentence' aggregate amount ofP1,150,000 and that these loans
(Article 36, Revised Penal Code). have not been paid?
While amnesty looks backward and
abolishes and puts into oblivion the Decision:
offense itself, it so overlooks and
obliterates the offense with which he is From the standpoint of its effects, a crime has
charged that the person released by a dual character: (1) as an offense against the State
amnesty stands before the law because of the disturbance of the social order and (2)
precisely as though he had committed as an offense against the private person injured by the
no offense." crime unless it involves the crime of treason, rebellion,
espionage, contempt and others (wherein no civil
This Court takes judicial notice liability arises on the part of the offender either
of the grant of amnesty upon accused- because there are no damages to be compensated or
appellant Jose N. Patriarca, Jr. Once there is no private person injured by the crime). What
granted, it is binding and effective. It gives rise to the civil liability is really the obligation of
serves to put an end to the appeal. everyone to repair or to make whole the damage
caused to another by reason of his act or omission,
whether done intentionally or negligently and whether
Arts. 100-103: Civil Liability or not punishable by law.

Nuguid v. Nicdao (G.R. No. 150785) Extinction of penal action does not carry with it
the eradication of civil liability, unless the extinction
proceeds from a declaration in the final judgment that
Facts: the fact from which the civil liability might arise did not
exist.
Accused Clarita S. Nicdao is charged with
having committed the crime of Violation of BP 22 in The basic principle in civil liability ex delicto  is
fourteen (14) counts. The criminal complaints allege that every person criminally liable is also civilly liable,
that sometime in 1996, from April to August thereof, crime being one of the five sources of obligations
78 | P a g e
under the Civil Code. A person acquitted of a criminal encashment of the subject checks. A check is said to
charge, however, is not necessarily civilly free because apply for account only when there is still a pre-existing
the quantum of proof required in criminal prosecution obligation. In the case at bench, the pre-existing
(proof beyond reasonable doubt) is greater than that obligation was extinguished after full payment was
required for civil liability (mere preponderance of made by [respondent]. We therefore find the clear and
evidence). In order to be completely free from civil convincing documentary evidence of payment
liability, a person's acquittal must be based on the fact presented by [respondent] worthy of credence.
that he did not commit the offense. If the acquittal is
based merely on reasonable doubt, the accused may
still be held civilly liable since this does not mean he Subsidiary Civil Liability of Other Persons
did not commit the act complained of. It may only be
that the facts proved did not constitute the offense Nueva Espana v. People (460 SCRA 547)
charged.

Acquittal will not bar a civil action in the Facts:


following cases: (1) where the acquittal is based on
reasonable doubt as only preponderance of evidence Petitioner Nueva Espana was found guilty of
is required in civil cases; (2) where the court declared reckless imprudence resulting into double homicide
the accused's liability is not criminal but only civil in when the passenger bus he was driving rammed into a
nature and (3) where the civil liability does not arise Honda motorcycle driven by Reynard So with Nilo
from or is not based upon the criminal act of which the Castro as passenger resulting into the death of both.
accused was acquitted. An aggravating circumstance was also imposed as
Espana also left the scene of the crime without lending
In this petition, we find no reason to ascribe assistance to the victims.
any civil liability to respondent. As found by the CA,
her supposed civil liability had already been fully During trial, the father of So and the mother of
satisfied and extinguished by payment. The Castro were both called on to testify as to the earning
statements of the appellate court leave no doubt that capacity of the two. So’s father claimed that his son
respondent, who was acquitted from the charges was earning P80,000 a month while Castro’s mother
against her, had already been completely relieved of said that his son was bringing in P8,000 a month. So’s
civil liability. father additionally testified that the funeral expenses
incurred by them was P87,000 while Castro’s mom
Likewise, [petitioner] admitted having received stated that they spent P30,000 for the funeral.
the cash payments from petitioner on a daily basis but
argues that the same were applied to interest As a result the trial court, besides imprisonment,
payments only. It however appears that [petitioner] awarded the following amounts to the heirs of the
was charging [respondent] with an exorbitant rate of victims:
interest on a daily basis. In any event, the cash
payments [made] were recorded at the back of the
 TO THE HEIRS OF THE VICTIM REYNARD SO
cigarette cartons by [petitioner] in her own handwriting
 1)       P2,997,000.00 – indemnity for loss of
as testified to by [respondent] and her employees,
earning capacity of victim
Melanie Tolentino and Jocelyn Nicdao. Indeed, the
daily cash payments marked in evidence as Exhibits 7  2)       14,200.00 – for expenses of the
to 15 reveal that [respondent] had already paid her wake
obligation to [petitioner] in the amount of  3)       20,000.00 – for funeral parlor
P5,780,000.00 as of July 21, 1997 and that she  4)       12,000.00 – for the tomb
stopped making further payments when she realized  5)       53,000.00 – for cost of burial site
that she had already paid such amount.  6)       30,000.00 – for attorney’s fees
 7)       200,000.00 – for moral damages
From the foregoing, it would appear that  8)       100,000.00 – for exemplary damages
[respondent] made a total payment of P6,980,000.00,  P3,429,200.00 – TOTAL AMOUNT
inclusive of the P1,200,000.00 Demand Draft, which is
definitely much more than P1,150,000.00, the amount  TO THE HEIRS OF VICTIM NILO CASTRO
she actually borrowed from [petitioner]. These facts
were never rebutted by [petitioner].
 1)       P1,728,000.00 – indemnity for loss of
earning capacity
Moreover, we find no evidence was presented
 2)       20,000.00 – for funeral expenses
by the prosecution to prove that there was a stipulation
in writing that interest will be paid by [respondent] on  3)       200,000.00 – for moral damages
her loan obligations [as required under Article 1956 of  4)       50,000.00 – for exemplary damages
the Civil Code].  P1,998,000.00 – TOTAL AMOUNT

By and large, the obligation of [respondent]


has already been extinguished long before the
79 | P a g e
The court based the amount of loss of earning To summarize, the heirs of the deceased Reynard So
capacity based on the formula used by the Supreme are entitled to the following:
Court as illustrated:
P   50,000 — civil indemnity ex delicto
As to the civil liability, particularly the      73,000 — actual damages
indemnity for the loss of the earning capacity of the      25,000 — temperate damages    
victims, the formula last enunciated by the Supreme 50,000 — moral damages
Court is:      25,000 — exemplary damages
     30,000 — attorney’s fees
“Net earning capacity (x) = life expectancy x P 253,000 — TOTAL
gross-living expenses annual (50% of gross
annual income)” The heirs of Nilo Castro are also entitled to the
following:
Thusly, since the victim Reynard So was
earning P80,000 a month at the time of his death when P    50,000 — civil indemnity ex delicto
he was thirty (30) years old, his lost earning capacity        50,000 — temperate damages      
should be computed as follows: 50,000 — moral damages
       25,000 — exemplary damages
x = 2 (80 ― 30) x [P960,000.00 ―        30,000 — attorney’s fees
P480,000.00) P  205,000 —  TOTAL
3
The SC meanwhile adopts the pronouncement of
x = 33.4 x P480,000.00 the Court of Appeals regarding the subsidiary liability
x =          x P16,032,000.00 of petitioner’s employer, Vallacar Transit Inc., under
Article 103 of the Revised Penal Code. An employer
may be subsidiarily liable for the employee’s civil
With respect to the victim Nilo Castro, he was
liability in the criminal action if it can be shown that: (1)
earning P8,000.00 a month when he died at the age of
the employer is engaged in any kind of industry; (2) the
twenty-six (26). His lost earnings were:
employee committed the offense in the discharge of
his duties and (3) the accused is insolvent. However,
x = 2 (80 ― 26)    [P96,000.00 ― P48,000.00] subject to prevailing jurisprudence, the subsidiary
3 liability may be enforced only upon a motion for
subsidiary writ of execution against Vallacar Transit,
x = 36 x P48,000.00 Inc. and upon proof that petitioner is insolvent.

x = P1,728,000.00 Pangonorom v. People (455 SCRA 211)

As a result, petitioner appeals to the CA but the


Facts:
appellate court affirmed the decision of the trial court
regarding the damages, Consequently, the CA
Pangonorom was the driver of a passenger
declared that Vallacar Transit Inc., should not yet be
bus owned and operated by MMTC which collided with
held subsidiary liable for the liability of the petitioner as
a Gemini Isuzu car driven by Carlos Berba, resulting
its driver. Thus, this petition for review with the SC.
into the damage of the car and physical injuries
obtained by Berba.
Issue:
Pangonorom was found guilty of reckless
Was the award of damages amounting to P8
imprudence resulting into damage to property and
million proper?
physical injuries.
Decision:
The trial court awarded damages amounting to
P42,000 but was silent as to the subsidiary liability of
The SC modifies the award of damages mostly
MMTC. Consequently, the CA affirmed the decision of
to the fact that loss of earning capacity should be
the trial court and also found MMTC subsidiary liable
properly adduced and supported by competent
for the amount notwithstanding the fact that the
evidence to prove the same. This rule also applies to
judgment of the trial court was silent as to said matter.
the funeral and burial expenses. In the case at bar, the
lower courts based their award for damages solely on
Issue:
the testimony of SO’s father and Castro’s mother, even
though both of them never substantiated the amounts
Did the CA err in not holding MMTC not
claimed with receipts, papers and other evidence. And
subsidiary liable despite the fact that the RTC did not
so the award is modified as follows:
mention anything to that effect?

Decision:
80 | P a g e
same not civilly liable because of the absence of
The SC ruled that even when the dispositive preponderance of evidence to prove liability. QUinto
portion of an RTC decision does not expressly appealed the civil aspect of the decision which the CA
pronounce subsidiary liability of the employer, they are affirmed.
deemed written into the judgment whenever
applicable. Issue:

But, he subsidiary liability of the employer Petitioner comes to the Court and raises the
arises only after conviction of the employee in the following issues: 1) Does extinction of criminal liability
criminal action. In the present case, there exists an carry with it extinction of the civil liability; and 2) was
employer-employee relationship between petitioners, the prosecution able to establish preponderance of
the MMTC is engaged in the transportation industry, evidence.
and Olimpio has been adjudged guilty of a wrongful act
and found to have committed the offense in the Decision:
discharge of his duties. However, there is no proof
here of Olimpio’s insolvency.  The judgment of The civil action based on delict is not
conviction against Olimpio has not attained finality.  extinguished unless the court itself finds that civil
This being so, no writ of execution can issue against liability did not arise. In the case at bar, the trial court
him to satisfy his civil liability.  Only after proof of the was very clear that the prosecution was not able to
accused-employee’s insolvency may the subsidiary establish a preponderance of evidence to find the
liability of his employer be enforced. respondents liable.

In short, there is as yet no occasion to speak As to whether preponderance of evidence


of enforcing the employer’s subsidiary civil liability should have been considered, the trial court and the
unless it appears that the accused-employee’s primary CA was correct in their findings. Preponderance of
liability cannot in the first instance be satisfied because evidence should not be based on the fact that the
of insolvency.  This fact cannot be known until evidence of the defense is weaker. The evidence
sometime after the verdict of conviction shall have presented must be strong enough to SUFFICIENTLY
become final.  And even if it appears prima facie that SUSTAIN THE CAUSE OF ACTION.
execution against the employee cannot be satisfied,
execution against the employer will not issue as a In the case at bar, the prosecution single
matter of course. The procedure for the enforcement of prosecution witness testified that the hematomas on
a judgment will have to be followed.  Once the the alleged victim may have been caused by either
judgment of conviction against Olimpio becomes final hitting with a blunt object or slipping and falling on the
and executory, and after the writ of execution issued hard pavement.
against him is returned unsatisfied because of his
insolvency, only then can a subsidiary writ of execution Even the friend of the deceased testified that
be issued against the MMTC after a hearing set for the drainage was so dark and this was the reason that
that precise purpose.  It is still too early to hold the he did not come with the other boys inside.
MMTC subsidiarily liable with its accused-employee
considering that there is no proof yet of Olimpio’s And so, the possibility of slippage by Wilson
insolvency. was very much a possibility.

Quinto v. Andres (453 SCRA 511) Probation Law (P.D. No. 968)

Francisco v. CA (G.R. No. 108747)


Facts:
Facts:
Petitioner Quinto is the mother of an 11-year
old boy named Wilson who died while going inside a Petitioner, as President and General Manager
drainage with the respondents Andres and Pacheco, of the company, humiliated his employees and blurted
who were also of the same age. out invectives against the latter. He was charged with
multip[le grave oral defamation by 5 of his employees
What was clear according to a witness who who were allegedly the recipient of the said invectives.
was a friend of the victim was that the three of them
(Wilson and the respondents) went inside the drainage He was found guilty of oral defamation in 4 out
filled with water. First to emerge was Pacheco who of 5 cases filed against him. Petitioner elevated the
immediately went home, and then next to come out judgment from the MeTC to the RTC; however, the
was Andres who was already carrying the dead body latter affirmed his conviction, with modification,
of Wilson. accrediting to him the mitigating circumstance of
passion or obfuscation. His appeal to the CA was to no
After being charged with homicide, the trial avail also.
court the respondents not guilty and also found the

81 | P a g e
Issue:

Whether or not petitioner is still qualified to


avail of probation?

Decision: NO

Probation is a mere privilege, not a right. Its


benefits cannot extend to those not expressly included.
Probation is not a right of an accused, but rather an act
of grace and clemency or immunity conferred by the Lagrosa v. People (G.R. No. 152044)
state which may be granted by the court to a
seemingly deserving defendant who thereby escapes Facts:
the extreme rigors of the penalty imposed by law for
the offense of which he stands convicted. Petitioners were found guilty by the RTC, of
violation with Sec. 68 of P.D. No. 705, for having found
The Probation Law should not therefore be in possession of forest products without permit. They
permitted to divest the state or its government of any appealed the decision to the CA, but it affirmed their
of the latter's prerogatives, rights or remedies, unless conviction, with modifications as to the penalty
the intention of the legislature to this end is clearly imposed by the lower court; from 2-8 years to 6
expressed, and no person should benefit from the months to 1 year.
terms of the law who is not clearly within them.
Petitioners applied for probation but was
 That an appeal should not bar the accused denied by the trial court, and subsequently affirmed by
from applying for probation if the appeal is taken solely the CA.
to reduce the penalty is simply contrary to the clear
and express mandate of Sec, 4 of P.D. No. 603, which Issues:
states that no application for probation shall be
entertained or granted if the defendant has perfected Whether or not petitioner can still apply for
the appeal from the judgment of conviction. probation?NO

The penalties imposed by the MeTC were Whether or not Fransisco v. CA applies in this
already probationable. Hence, there was no need to case?YES
appeal if only to reduce the penalties to within the
probationable period. Decision:

The fact that petitioners put the merits of their


 The multiple prison terms are distinct from
conviction in issue on appeal belies their claim that
each other, and if none of the terms exceeds the limit
their appeal was prompted by what was admittedly an
set out in the P.D. No. 603, then he is entitled to
incorrect penalty.  Certainly, the protestations of
probation, unless he is otherwise specifically
petitioners connote a profession of guiltlessness, if not
disqualified.
complete innocence, and do not simply assail the
propriety of the penalties imposed.  
Fixing the cut-off point at a maximum term of
six (6) years imprisonment for probation is based on
For sure, petitioners never manifested that
the assumption that those sentenced to higher
they were appealing only for the purpose of correcting
penalties pose too great a risk to society, not just
a wrong penalty – to reduce it to within probationable
because of their demonstrated capability for serious
range.  Hence, upon interposing an appeal, more so
wrong doing but because of the gravity and serious
after asserting their innocence therein, petitioners
consequences of the offense they might further
should be precluded from seeking probation.
commit. 
By perfecting their appeal, petitioners ipso
Considering that the multiple prison terms
facto relinquished the alternative remedy of availing of
should not be summed up but taken separately as the
the Probation Law, the purpose of which is simply to
totality of all the penalties is not the test, petitioner
prevent speculation or opportunism on the part of an
should have immediately filed an application for
accused who, although already eligible, does not at
probation as he was already qualified after being
once apply for probation, but did so only after failing in
convicted by the MeTC, if indeed thereafter he felt
his appeal.
humbled, was ready to unconditionally accept the
verdict of the court and admit his liability.
Consequently, in appealing the Decision of the MeTC Although it has been suggested that an appeal
to the RTC, petitioner lost his right to probation. For, should not bar the accused from applying for probation
plainly, the law considers appeal and probation if the appeal is solely to reduce the penalty to within
mutually exclusive remedies. the probationable limit may be equitable, we are not

82 | P a g e
yet prepared to accept this proposition, especially Pacita Linghon was the helper of Jovita
given the factual circumstances of this case.  Had the Rodriguez. Pacita, through her brother Macarion, sold
petitioners’ appeal from the decision of the trial court to petitioner Ernesto Linghon several pieces of jewelry
raised the impropriety of the penalty imposed upon stolen from Rodriguez. The Regional Trial Court of
them as the sole issue, perhaps this Court would have Malolos, Bulacan, Branch 22, found petitioner Ernesto
been more sympathetic to their plight.  Unfortunately, Francisco guilty of violating Presidential Decree No.
their misrepresentation has led to their own undoing. 1612, otherwise known as the Anti-Fencing Law,
sentencing him to suffer the penalty of ten (10) years
Vicoy v. People (G.R. No. 138203) and one (1) day of prision mayor maximum, as
minimum, to twenty (20) years of reclusion temporal
Facts: maximum, as maximum, with the accessory penalties
corresponding to the latter, and to pay the
Petitioner was convicted for violation of an corresponding value of the subject pieces of jewelry.
ordinance against peddling fish outside of market, as The petitioner asserts that the prosecution failed to
well as for the crime of resisting and disobeying an prove his guilt for the crime charged beyond
agent of a person in authority. reasonable doubt. He avers that the prosecution failed
to prove that Pacita stole the jewelry subject of the
Petitioner filed an application for probation, but charge, and that Macario sold the said pieces of
subsequently withdrew it and filed a notice of appeal. jewelry to him.
MTCC granted the withdrawal of application for
probation butdenied her notice appeal for being filed Issue:
out of time.The court ordered petitioner to furnish the
City Prosecutor’s Office a copy of her memorandum Whether the Court of Appeals erred in
and the assailed judgement of conviction. Petitioner sustaining the trial court’s decision finding petitioner
failed to do so, and the court dismissed her special guilty beyond reasonable doubt of violation of the (sic)
civil action for certiorari. Presidential Decree No. 1612, otherwise known as the
Anti-Fencing Law?
Issue:
Decision:
Whether or not the RTC erred in dismissing
the petition for certiorari on ground of petitioner’s The essential elements of the crime of fencing
failure to comply with the earlier Order of the same are as follows: (1) a crime of robbery or theft has been
court? committed; (2) the accused, who is not a principal or
accomplice in the commission of the crime of robbery
Decision: NO or theft, buys, receives, possesses, keeps, acquires,
conceals, sells or disposes, or buys and sells, or in any
The fact that the City Prosecutor’s Office has manner deals in any article, item, object or anything of
not yet entered its appearance is no justification to value, which has been derived from the proceeds of
petitioner’s adamant and continued insistence not to the crime of robbery or theft; (3) the accused knew or
comply with a lawful order of the court. Every court has should have shown that the said article, item, object or
the power to enforce and compel obedience to its anything of value has been derived from the proceeds
orders, judgments, and processes in all proceedings of the crime of robbery or theft; and, (4) there is, on the
pending before it. part of the accused, intent to gain for himself or for
another. Macario Linghon testified that he sold the
Section 7, Rule 120, of the Rules on Criminal jewelry to petitioner. “Although the well-entrenched
Procedure is explicit that a judgment in a criminal case rule is that the testimony of a single witness is
becomes final when the accused has applied for sufficient on which to anchor a judgment of conviction,
probation. This is totally in accord with Section 4 of it is required that such testimony must be credible and
Presidential Decree No. 968, which in part provides reliable. In this case, we find the testimony of Macario
that the filing of an application for probation is deemed to be dubious; hence, barren of probative weight.” The
a waiver of the right to appeal. Thus, there was no Court further held “It bears stressing that, in the
more opportunity for petitioner to exercise her right to absence of direct evidence that the accused had
appeal, the judgment having become final by the filing knowledge that the jewelry was stolen, the prosecution
of an application for probation. is burdened to prove facts and circumstances from
Maria Criselda Fojas which it can be concluded that the accused should
2010-0226 have known that the property sold to him were stolen.
This requirement serves two basic purposes: (a) to
prove one of the elements of the crime of fencing; and,
Anti-Fencing Law (P.D. No. 1612) (b) to enable the trial court to determine the imposable
penalty for the crime, since the penalty depends on the
Francisco v. People (434 SCRA 122) value of the property.”

Facts: Tan v. People (313 SCRA 220)

83 | P a g e
Facts: of corpus delicti.The Court held that accused Tan
could not be held guilty because there was no showing
Rosita Lim is the proprietor of Bueno Metal at all that the accused knew or should have known that
Industries; upon inventory, she found that several the very stolen articles were the ones sold to him.
pieces of equiptment were missing. Manuelito Mendez
was a former employee of Lim, who left her
employment before Lim found out that her goods were
missing. Subsequently, Manuelito Mendez was
arrested in the Visayas and he admitted that he and
his companion Gaudencio Dayop stole from the
complainants warehouse some boat spare parts such
as bronze and stainless propellers and brass screws.
Manuelito Mendez asked for complainant’s
forgiveness. He pointed to petitioner Ramon C. Tan as
the one who bought the stolen items and who paid the
amount of P13,000.00, in cash to Mendez and Dayop,
and they split the amount with one another. Tan was
found guilty beyond reasonable doubt of violating the
Anti-Fencing Law of 1979, otherwise known as
Presidential Decree No. 1612, and sentences him to
suffer the penalty of imprisonment of SIX (6) YEARS
and ONE (1) DAY to TEN (10) YEARS of prision
mayor and to indemnify Rosita Lim the value of the
stolen merchandise purchased by him in the sum of
P18,000.00.

Issue:

Whether or not the prosecution has


successfully established the elements of fencing as
against petitioner?

Decision:

Fencing, as defined in Section 2 of P.D. No.


1612 is the act of any person who, with intent to gain
for himself or for another, shall buy, receive, possess,
keep, acquire, conceal, sell or dispose of, or shall buy
and sell, or in any manner deal in any article, item,
object or anything of value which he knows, or should
be known to him, to have been derived from the
proceeds of the crime of robbery or theft. The essential
elements of the crime of fencing are as follows: (1) a
crime of robbery or theft has been committed; (2) the
accused, who is not a principal or accomplice in the
commission of the crime of robbery or theft, buys,
receives, possesses, keeps, acquires, conceals, sells
or disposes, or buys and sells, or in any manner deals
in any article, item, object or anything of value, which
has been derived from the proceeds of the crime of
robbery or theft; (3) the accused knew or should have
shown that the said article, item, object or anything of
value has been derived from the proceeds of the crime
of robbery or theft; and, (4) there is, on the part of the
accused, intent to gain for himself or for another. The
Supreme Court stated that there was no sufficient
proof of the unlawful taking of anothers property. The
theft was not proved because complainant Rosita Lim
did not complain to the public authorities of the
felonious taking of her property. She sought out her
former employee Manuelito Mendez, who confessed
that he stole certain articles from the warehouse of the
complainant and sold them to petitioner. Such
confession is insufficient to convict, without evidence
84 | P a g e
[

You might also like